You are on page 1of 57

EN BANC Veneer Co., Inc.

, and the negotiation came to reality


[G.R. No. 73002. December 29, 1986.] when the Board of Directors of the Acme Plywood &
Veneer Co., Inc., had donated a part of the land
THE DIRECTOR OF bought by the Company from the Infiels for the
LANDS, petitioner, vs. INTERMEDIATE townsite of Maconacon, Isabela (Exh. 'N') on
APPELLATE COURT and ACME PLYWOOD & November 15, 1979, and which donation was
VENEER CO. INC., ETC., respondents. accepted by the Municipal Government of
Maconacon, Isabela (Exh. 'N-1'), during their special
D. Nacion Law Office for private respondent. session on November 22, 1979."
DECISION
NARVASA, J p: The Director of Lands takes no issue with any of these findings except as to the
The Director of Lands has brought this appeal by certiorari from a judgment of applicability of the 1935 Constitution to the matter at hand. Concerning this, he
the Intermediate Appellate Court affirming a decision of the Court of First asserts that, the registration proceedings have been commenced only on July
Instance of Isabela, which ordered registration in favor of Acme Plywood & 17, 1981, or long after the 1973 Constitution had gone into effect, the latter is
Veneer Co., Inc. of five parcels of land measuring 481, 390 square meters, more the correctly applicable law; and since section 11 of its Article XIV prohibits
or less, acquired by it from Mariano and Acer Infiel, members of the Dumagat private corporations or associations from holding alienable lands of the public
tribe. domain, except by lease not to exceed 1,000 hectares (a prohibition not found
in the 1935 Constitution which was in force in 1962 when Acme purchased the
The registration proceedings were for confirmation of title under Section 48 lands in question from the Infiels), it was reversible error to decree registration
of Commonwealth Act No. 141 (The Public Land Act). as amended; and the in favor of Acme.
appealed judgment sums up the findings of the trial court in said proceedings in
this wise: Section 48, paragraphs (b) and (c), of Commonwealth Act No. 141, as amended,
reads:
"1. That Acme Plywood & Veneer Co. Inc.,
represented by Mr. Rodolfo Nazario is a corporation "SEC. 48. The following described citizens of the
duly organized in accordance with the laws of the Philippines, occupying lands of the public domain or
Republic of the Philippines and registered with the claiming to own any such lands or an interest therein,
Securities and Exchange Commission on December but whose titles have not been perfected or
23, 1959; completed, may apply to the Court of First Instance of
the province where the land is located for
2. That Acme Plywood & Veneer Co. Inc., confirmation of their claims, and the issuance of a
represented by Mr. Rodolfo Nazario can acquire real certificate of title therefor, under the Land Registration
properties pursuant to the provisions of the Articles of Act, to wit:
Incorporation particularly on the provision of its
secondary purposes (paragraph (9), Exhibit 'M-1'); xxx xxx xxx

3. That the land subject of the Land Registration (b) Those who by themselves or through their
proceeding was ancestrally acquired by Acme predecessors-in-interest have been in open,
Plywood & Veneer Co., Inc., on October 29, 1962, continuous, exclusive and notorious possession and
from Mariano Infiel and Acer Infiel, both members of occupation of agricultural lands of the public domain,
the Dumagat tribe and as such are cultural minorities; under a bona fide claim of acquisition or ownership,
for at least thirty years immediately preceding the
4. That the constitution of the Republic of the filing of the application for confirmation of title except
Philippines of 1935 is applicable as the sale took when prevented by war or force majeure. These shall
place on October 29, 1962; be conclusively presumed to have performed all the
conditions essential to a Government grant and shall
5. That the possession of the Infiels over the land be entitled to a certificate of title under the provisions
relinquished or sold to Acme Plywood & Veneer Co., of the chapter.
Inc., dates back before the Philippines was
discovered by Magellan as the ancestors of the Infiels (c) Members of the National Cultural minorities who
have possessed and occupied the land from by themselves or through their predecessors-in-
generation to generation until the same came into the interest have been in open, continuous, exclusive and
possession of Mariano Infiel and Acer Infiel; notorious possession and occupation of lands of the
public domain suitable to agriculture, whether
6. That the possession of the applicant Acme disposable or not, under a bona fide claim of
Plywood & Veneer Co., Inc., is continuous, adverse ownership for at least 30 years shall be entitled to the
and public from 1962 to the present and tacking the rights granted in subsection (b) hereof."
possession of the Infiels who were granted from
whom the applicant bought said land on October 29, The Petition for Review does not dispute — indeed, in view of the quoted
1962, hence the possession is already considered findings of the trial court which were cited and affirmed by the Intermediate
from time immemorial; Appellate Court, it can no longer controvert before this Court — the fact that
Mariano and Acer Infiel, from whom Acme purchased the lands in question on
7. That the land sought to be registered is a private October 29, 1962, are members of the national cultural minorities who had, by
land pursuant to the provisions of Republic Act No. themselves and through their progenitors, possessed and occupied those lands
3872 granting absolute ownership to members of the since time immemorial, or for more than the required 30-year period and were,
non-Christian Tribes on land occupied by them or by reason thereof, entitled to exercise the right granted in Section 48 ofthe Public
their ancestral lands, whether with the alienable or Land Act to have their title judicially confirmed. Nor is there any pretension that
disposable public land or within the public domain; Acme, as the successor-in-interest of the Infiels, is disqualified to acquire and
register ownership of said lands under any provisions of the 1973
8. That applicant Acme Plywood & Veneer Co. Inc.,
Constitution other than Section 11 of its Article XIV already referred to.
has introduced more than Forty-Five Million
(P45,000,000.00) Pesos worth of improvements, said Given the foregoing, the question before this Court is whether or not the title that
improvements were seen by the Court during its the Infiels had transferred to Acme in 1962 could be confirmed in favor of the
ocular investigation of the land sought to be latter in proceedings instituted by it in 1981 when the 1973 Constitution was
registered on September 18, 1982; already in effect, having in mind the prohibition therein against private
corporations holding lands of the public domain except in lease not exceeding
9. That the ownership and possession of the land
1,000 hectares.
sought to be registered by the applicant was duly
recognized by the government when the Municipal The question turns upon a determination of the character of the lands at the time
Officials of Maconacon, Isabela, have negotiated for of institution of the registration proceedings in 1981. If they were then still part
the donation of the townsite from Acme Plywood & of the public domain, it must be answered in the negative. If, on the other hand,
they were then already private lands, the constitutional prohibition against their 926, that all the necessary requirements for a grant
acquisition by private corporations or associations obviously does not apply. by the Government were complied with, for he has
been in actual and physical possession, personally
In this regard, attention has been invited to Manila Electric Company vs. Castro- and through his predecessors, of an agricultural land
Bartolome, et al, 1 where a similar set of facts prevailed. In that case, Manila of the public domain openly, continuously, exclusively
Electric Company, a domestic corporation more than 60% of the capital stock of and publicly since July 26, 1984, with a right to a
which is Filipino-owned, had purchased in 1947 two lots in Tanay, Rizal from the certificate of title to said land under the provisions of
Piguing spouses. The lots had been possessed by the vendors and, before Chapter VIII of said Act. So that when Angela Razon
them, by their predecessor-in-interest, Olimpia Ramos, since prior to the applied for the grant in her favor, Valentin Susi had
outbreak of the Pacific War in 1941. On December 1, 1976, Meralco applied to already acquired by operation of law not only a right
the Court of First Instance of Rizal, Makati Branch, for confirmation of title to said to a grant, but a grant of the Government, for it is not
lots. The court, assuming that the lots were public land, dismissed the necessary that a certificate of title should be issued in
application on the ground that Meralco, a juridical person, was not qualified to order that said grant may be sanctioned by the courts,
apply for registration under Section 48(b) of the Public Land Act which allows an application therefor is sufficient, under the
only Filipino citizens or natural persons to apply for judicial confirmation of provisions of section 47 of Act No. 2874. If by a legal
imperfect titles to public land. Meralco appealed, and a majority of this Court fiction, Valentin Susi had acquired the land in
upheld the dismissal. It was held that: question by a grant of the State, it had already ceased
to be of the public domain and had become private
". . ., the said land is still public land. It would cease
property, at least by presumption, of Valentin Susi,
to be public land only upon the issuance of the
beyond the control of the Director of Lands.
certificate of title to any Filipino citizen claiming it
Consequently, in selling the land in question of
under section 48(b). Because it is still public land and
Angela Razon, the Director of Lands disposed of a
the Meralco, as a juridical person, is disqualified to
land over which he had no longer any title or control,
apply for its registration under section 48(b),
and the sale thus made was void and of no effect, and
Meralco's application cannot be given due course or
Angela Razon did not thereby acquire any right. 6
has to be dismissed.
Succeeding cases, of which only some need be mentioned, like Lacaste vs.
xxx xxx xxx
Director of Lands, 7 Mesina vs. Vda. de Sonza, 8 Manarpac vs.
"Finally, it may be observed that the constitutional Cabanatuan, 9 Miguel vs. Court of Appeals 10 and Herico vs. Dar, supra, by
prohibition makes no distinction between (on the one invoking and affirming the Susi doctrine have firmly rooted it in jurisprudence.
hand) alienable agricultural public lands as to which
Herico, in particular, appears to be squarely affirmative: 11
no occupant has an imperfect title and (on the other
hand) alienable lands of the public domain as to ". . . Secondly, under the provisions of Republic Act
which an occupant has an imperfect title subject to No. 1942, which the respondent Court held to be
judicial confirmation. inapplicable to the petitioner's case, with the latter's
proven occupation and cultivation for more than 30
Since section 11 of Article XIV does not distinguish,
years since 1914, by himself and by his
we should not make any distinction or qualification.
predecessors-in-interest, title over the land has
The prohibition applies to alienable public lands as to
vested on petitioner so as to segregate the land from
which a Torrens title may be secured under section
the mass of public land. Thereafter, it is no longer
48(b). The proceeding under section 48(b)
disposable under the Public Land Act as by free
`presupposes that the land is public' (Mindanao vs.
patent. . . .
Director of Lands, L-19535, July 30, 1967, 20 SCRA
641, 644)." xxx xxx xxx
The present Chief Justice entered a vigorous dissent, tracing the line of cases As interpreted in several cases, when the conditions
beginning with Cariño in 1909 2 thru Susi in 1925 3 down to Herico in as specified in the foregoing provision are complied
1980, 4which developed, affirmed and reaffirmed the doctrine that open, with, the possessor is deemed to have acquired, by
exclusive and undisputed possession of alienable public land for the period operation of law, a right to a grant, a government
prescribed by law creates the legal fiction whereby the land, upon completion of grant, without the necessity of a certificate of title
the requisite period ipso jure and without the need of judicial or other sanction, being issued. The land, therefore, ceases to be of the
ceases to be public land and becomes private property. That said dissent public domain and beyond the authority of the
expressed what is the better — and, indeed, the correct, view — becomes Director of Lands to dispose of. The application for
evident from a consideration of some of the principal rulings cited therein. confirmation is mere formality, the lack of which does
not affect the legal sufficiency of the title as would be
evidenced by the patent and the Torrens title to be
The main theme was given birth, so to speak, in Cariño, involving the issued upon the strength of said patent." 12
Decree/Regulations of June 25, 1880 for adjustment of royal lands wrongfully
Nothing can more clearly demonstrate the logical inevitability of considering
occupied by private individuals in the Philippine Islands. It was ruled that:
possession of public land which is of the character and duration prescribed by
"It is true that the language of articles 4 and 5 5 attributes title to those `who may statute as the equivalent of an express grant from the State than the dictum of
prove' possession for the necessary time and we do not overlook the argument the statute itself 13 that the possessor(s) ". . . shall be conclusively presumed to
that this means may prove in registration proceedings. It may be that an English have performed all the conditions essential to a Government grant and shall be
conveyancer would have recommended an application under the foregoing entitled to a certificate of title . . ." No proof being admissible to overcome a
decree, but certainly it was not calculated to convey to the mind of an Igorot chief conclusive presumption, confirmation proceedings would, in truth be little more
the notion that ancient family possessions were in danger, if he had read every than a formality, at the most limited to ascertaining whether the possession
word of it. The words 'may prove' (acrediten), as well or better, in view of the claimed is of the required character and length of time; and registration
other provisions, might be taken to mean when called upon to do so in any thereunder would not confer title, but simply recognize a title already vested. The
litigation. There are indications that registration was expected from all, but none proceedings would not originally convert the land from public to private land, but
sufficient to show that, for want of it, ownership actually gained would be lost. only confirm such a conversion already affected by operation of law from the
The effect of the proof, wherever made, was not to confer title, but simply to moment the required period of possession became complete. As was so well
establish it, as already conferred by the decree, if not by earlier law. . . ." put in Cariño, ". . . (T)here are indications that registration was expected from
all, but none sufficient to show that, for want of it, ownership actually gained
That ruling assumed a more doctrinal character because expressed in more would be lost. The effect of the proof, wherever made, was not to confer title,
categorical language, in Susi: but simply to establish it, as already conferred by the decree, if not by earlier
law."
". . . In favor of Valentin Susi, there is, moreover, the
presumption juris et de jure established in paragraph If it is accepted — as it must be — that the land was already private land to which
(b) of section 45 of Act No. 2874, amending Act No. the Infiels had a legally sufficient and transferable title on October 29, 1962 when
Acme acquired it from said owners, it must also be conceded that Acme had a said period, ipso jure. Following that rule and on the basis of the undisputed
perfect right to make such acquisition, there being nothing in the 1935 facts, the land subject of this appeal was already private property at the time it
Constitution then in force (or, for that matter, in the 1973 Constitution which was acquired from the Infiels by Acme. Acme thereby acquired a registrable title,
came into effect later) prohibiting corporations from acquiring and owning private there being at the time no prohibition against said corporation's holding or
lands. owning private land. The objection that, as a juridical person, Acme is not
qualified to apply for judicial confirmation of title under section 48(b) of the Public
Even on the proposition that the land remained technically "public" land, despite Land Act is technical, rather than substantial and, again, finds its answer in the
immemorial possession of the Infiels and their ancestors, until title in their favor dissent in Meralco:
was actually confirmed in appropriate proceedings under the Public Land Act,
there can be no serious question of Acme's right to acquire the land at the time
it did, there also being nothing in the 1935 Constitution that might be construed
to prohibit corporations from purchasing or acquiring interests in public land to "6. To uphold respondent judge's denial of Meralco's
which the vendor had already acquired that type of so-called "incomplete" or application on the technicality that the Public Land
"imperfect" title. The only limitation then extant was that corporations could not Act allows only citizens of the Philippines who are
acquire, hold or lease public agricultural lands in excess of 1,024 hectares. The natural persons to apply for confirmation of their title
purely accidental circumstance that confirmation proceedings were brought would be impractical and would just give rise to
under the aegis of the 1973 Constitution which forbids corporations from owning multiplicity of court actions. Assuming that there was
lands of the public domain cannot defeat a right already vested before that law a technical error in not having filed the application for
came into effect, or invalidate transactions then perfectly valid and proper, This registration in the name of the Piguing spouses as the
Court has already held, in analogous circumstances, that the Constitution original owners and vendors, still it is conceded that
cannot impair vested rights. there is no prohibition against their sale of the land to
the applicant Meralco and neither is there any
"We hold that the said constitutional prohibition against the application being refiled with
prohibition 14 has no retroactive application to the retroactive effect in the name of the original owners
sales application of Biñan Development Co., Inc. and vendors (as such natural persons) with the end
because it had already acquired a vested right to the result of their application being granted, because of
land applied for at the time the 1973 Constitution took their indisputable acquisition of ownership by
effect. operation of law and the conclusive presumption
therein provided in their favor. It should not be
That vested right has to be respected. It could not be necessary to go through all the rituals at the great
abrogated by the new Constitution. Section 2, Article cost of refiling of all such applications in their names
XIII of the 1935 Constitution allows private and adding to the overcrowded court dockets when
corporations to purchase public agricultural lands not the Court can after all these years dispose of it here
exceeding one thousand and twenty-four hectares. and now. (See Francisco vs. City of Davao)
Petitioner' prohibition action is barred by the doctrine
of vested rights in constitutional law. The ends of justice would best be served, therefore,
by considering the applications for confirmation as
xxx xxx xxx amended to conform to the evidence, i.e. as filed in
the names of the original persons who as natural
The due process clause prohibits the annihilation of
persons are duly qualified to apply for formal
vested rights. `A state may not impair vested rights by
confirmation of the title that they had acquired by
legislative enactment, by the enactment or by the
conclusive presumption and mandate of the Public
subsequent repeal of a municipal ordinance, or by a
Land Act and who thereafter duly sold to the herein
change in the constitution of the State, except in a
corporations (both admittedly Filipino corporations
legitimate exercise of the police power' (16 C.J.S.
duly qualified to hold and own private lands) and
1177-78).
granting the applications for confirmation of title to the
xxx xxx xxx private lands so acquired and sold or exchanged."

In the instant case, it is incontestable that prior to the There is also nothing to prevent Acme from reconveying the lands to the
effectivity of the 1973 Constitution the right of the Infiels and the latter from themselves applying for confirmation of title
corporation to purchase the land in question had and, after issuance of the certificate/s of title in their names, deeding the
become fixed and established and was no longer lands back to Acme. But this would be merely indulging in empty
open to doubt or controversy. charades, whereas the same result is more efficaciously and speedily
obtained, with no prejudice to anyone, by a liberal application of the rule
Its compliance with the requirements of the Public on amendment to conform to the evidence suggested in the dissent
Land Law for the issuance of a patent had the effect in Meralco.
of segregating the said land from the public domain. While this opinion seemingly reverses an earlier ruling of comparatively recent
The corporation's right to obtain a patent for the land vintage, in a real sense, it breaks no precedent, but only reaffirms and re-
is protected by law. It cannot be deprived of that right
established, as it were, doctrines the soundness of which has passed the test of
without due process (Director of Lands vs. CA, 123
searching examination and inquiry in many past cases. Indeed, it is worth noting
Phil. 919)." 15
that the majority opinion, as well as the concurring opinions of Chief Justice
The fact, therefore, that the confirmation proceedings were instituted by Acme Fernando and Justice Abad Santos, in Meralco rested chiefly on the proposition
in its own name must be regarded as simply another accidental circumstance, that the petitioner therein, a juridical person, was disqualified from applying for
productive of a defect hardly more than procedural and in nowise affecting the confirmation of an imperfect title to public land under Section 48(b) of the Public
substance and merits of the right of ownership sought to be confirmed in said Land Act. Reference to the 1973 Constitution and its Article XIV, Section 11,
proceedings, there being no doubt of Acme's entitlement to the land. As it is was only tangential, limited to a brief paragraph in the main opinion, and may, in
unquestionable that in the light of the undisputed facts, the Infiels, under either that context, be considered as essentially obiter. Meralco, in short, decided no
the 1935 or the 1973 Constitution, could have had title in themselves confirmed constitutional question.
and registered, only a rigid subservience to the letter of the law would deny the
WHEREFORE, there being no reversible error in the appealed judgment of the
same benefit to their lawful successor-in-interest by valid conveyance which Intermediate Appellate Court, the same is hereby affirmed, without costs in this
violates no constitutional mandate.
instance.
The Court, in the light of the foregoing, is of the view, and so holds, that the SO ORDERED.
majority ruling in Meralco must be reconsidered and no longer deemed to be
binding precedent. The correct rule, as enunciated in the line of cases already Feria, Yap, Fernan, Alampay, Cruz, Paras and Feliciano, JJ ., concur.
referred to, is that alienable public land held by a possessor, personally or
through his predecessors-in-interest, openly, continuously and exclusively for Gutierrez, Jr., J ., I reiterate my concurrence in Meralco v. Castro-Bartolome,
the prescribed statutory period (30 years under The Public Land Act, as and, therefore, dissent here.
amended) is converted to private property by the mere lapse or completion of
Separate Opinions immemorial, respondent corporation's lawful purchase from them of the land in
1962 and P45 million investments redounding presumably to the welfare and
progress of the community, particularly the municipality of Maconacon, Isabela
TEEHANKEE, C.J., concurring: to which it donated part of the land for the townsite created a vested right which
could not be impaired by the prohibition adopted eleven years later. But as
I am honored by my brethren's judgment at bar that my dissenting opinion in the sufficiently stressed, the land of the Infiels had been ipso jure converted
June, 1982 Meralco and Iglesia ni Cristo cases, 1 which is herein upheld, into private land and they had a legally sufficient and transferable
"expressed what is the better . . . and indeed the correct view." My dissent was title conferred by the conclusive presumption of the Public Land Act (which
anchored on the landmark 1909 case of Cariño 2 through the 1925 case of needed only to be established in confirmation of title proceedings for
Susi3 and the long line of cases cited therein to the latest 1980 case formalization and issuance of the certificate of title) which they lawfully and
of Herico 4 that "it is established doctrine . . . that an open, continuous, adverse validly transferred to respondent corporation.
and public possession of a land of the public domain for the period provided
in the Public Land Act provision in force at the time (from July 26, 1894 In fact, the many amendments to the Act extending the period for the filing of
in Susi under the old law [this period was reduced to 'at least thirty years such applications for judicial confirmation of imperfect and incomplete titles to
immediately preceding the filing of the application for confirmation of title' by alienable and disposable public lands expressly reiterate that it has always been
amendment ofCommonwealth Act No. 141, equivalent to the period of the "policy of the State to hasten the settlement, adjudication and quieting of
acquisitive prescription 5 ]) by a private individual personally and through his titles to [such] unregistered lands," i.e. to recognize that such lands publicly and
predecessors confers an effective title on said possessor, whereby the land notoriously occupied and cultivated under bona fide claim of acquisition or
ceases to be land of the public domain and becomes private property." I hereby ownership have ipso jure been converted into private property and grant the
reproduce the same by reference for brevity's sake. But since we are reverting possessors the opportunity to establish and record such fact. Thus, the deadline
to the old above-cited established doctrine and precedents and discarding for the filing of such application which would have originally expired first on
the Meralco and Iglesia ni Cristo cases which departed therefrom in the recent December 31, 1938 was successively extended to December 31, 1941, then
past, I feel constrained to write this concurrence in amplification of my views extended to December 31, 1957, then to December 31, 1968, further extended
and ratio decidendi. to December 31, 1976 and lastly extended to December 31, 1987. 7

Under the express text and mandate of the cited Act, such possessors "shall
be conclusively presumed to have performed all the conditions essential to a
The cited Act's provision that only natural persons may apply thereunder for
Government grant and shall be entitled to a certificate of title under the
confirmation of title is in effect a technicality of procedure and not of substance.
provisions of this chapter."
My submittal in Meralco, mutatis mutandis, is properly applicable: "The ends of
The Court thus held in Susi that under the presumption juris et de justice would best be served, therefore, by considering the applications for
jure established in the Act, the rightful possessor of the public land for the confirmation as amended to conform to the evidence, i.e. as filed in the names
statutory period "already acquired, by operation of law, not only a right to a grant, of the original persons who as natural persons are duly qualified to apply for
but a grant of the Government, for it is not necessary that certificate of title formal confirmation of the title that they had acquired by conclusive presumption
should be issued an order that said grant may be sanctioned by the courts, an and mandate of the Public Land Act and who thereafter duly sold to the herein
application therefor is sufficient . . . If by a legal fiction, Valentin Susi corporations (both admittedly Filipino corporations duly qualified to hold and own
had acquired the land in question by a grant of the State, it had already ceased private lands) and granting the applications for confirmation of title to the private
to be of the public domain, and had become private property, at least by lands so acquired and sold or exchanged." 8 Indeed, then Chief Justice Enrique
presumption, of Valentin Susi, beyond the control of the Director of Lands [and M. Fernando likewise dissented along the same line from the majority ruling
beyond his authority to sell to any other person]." 6 therein and held: "I dissent insofar as the opinion of the Court would characterize
such jurisdictional defect that the applicant was Meralco, a juridical person
The root of the doctrine goes back to the pronouncement of Justice Oliver rather than the natural persons-transferors, under the particular circumstances
Wendell Holmes for the U.S. Supreme Court in the 1909 case of Cariño (the of this case, as an insurmountable obstacle to the relief sought. I would apply by
Igorot chief who would have been deprived of ancestral family lands by the analogy, although the facts could be distinguished, the approach followed by us
dismissal of his application for registration) which reversed the dismissal of the in Francisco v. City of Davao, where the legal question raised, instead of being
registration court (as affirmed by the Supreme Court) and adopted the liberal deferred and possibly taken up in another case, was resolved. By legal fiction
view that under the decree and regulations of June 25, 1880, "The words 'may and in the exercise of our equitable jurisdiction, I feel that the realistic solution
prove' (acrediten), as well, or better, in view of the other provisions, might be would be to decide the matter as if the application under Section 48(b) were filed
taken to mean when called upon to do so in any litigation. There are indications by the Piguing spouses, who I assume suffer from no such disability." 9 Justice
that registration was expected from all, but none sufficient to show that, for want Vicente Abad Santos, now retired, while concurring in the procedural result,
of it, ownership actually gained would be lost. The effect of the proof,whenever likewise, in effect dissented from the therein majority ruling on the question of
made, was not to confer title, but simply to establish it, as already conferred by substance, and stated his opinion that "the lots which are sought to be registered
the decree, if not by earlier law." have ceased to be lands of the public domain at the time they were acquired by
the petitioner corporation. They are already private lands because of acquisitive
The Court's decision at bar now expressly overturns the Meralco and related prescription by the predecessors of the petitioner and all that is needed is the
cases subsequent thereto which failed to adhere to the aforecited established confirmation of the title. Accordingly, the constitutional provision that no private
doctrine dating back to 1909 and was consistently applied up to June 29, 1982 corporation or association may hold alienable lands of the public domain is
(when the Meralco decision was promulgated). We reaffirm the established inapplicable." 10
doctrine that such acquisitive prescription of alienable public lands takes
place ipso jure or by operation of law without the necessity of a prior issuance To my mind, the reason why the Act limits the filing of such applications to natural
of a certificate of title. The land ipso jure ceases to be of the public domain and citizens who may prove their undisputed and open possession of public lands
becomes private property, which may be lawfully sold to and acquired by for the required statutory thirty-year period, tacking on their predecessors'-in-
qualified corporations such as respondent corporation. (As stressed in Herico, interest possession is that only natural persons, to the exclusion of juridical
supra, "the application for confirmation is a mere formality, the lack of which persons such as corporations, can actually, physically and in reality possess
does notaffect the legal sufficiency of the title.") public lands for the required statutory 30-year period. That juridical persons or
corporations cannot do so is obvious. But when the natural persons have fulfilled
Such ipso jure conversion into private property of public lands publicly held the required statutory period of possession, the Act confers on them a legally
under a bona fide claim of acquisition or ownership is the public policy of the Act sufficient and transferable title. It is preferable to follow the letter of the law
and is so expressly stated therein. By virtue of such conversion into private that they file the applications for confirmation of their title, although they have
property, qualified corporations may lawfully acquire them and there is no lawfully transferred their title to the land. But such procedural failure cannot and
"alteration or defeating" of the 1973 Constitution's prohibition against should not defeat the substance of the law, as stressed in the above-cited
corporations holding or acquiring title to lands of the public domain, as claimed opinions, that the lands are already private lands because of acquisitive
in the dissenting opinion, for the simple reason that no public lands are involved. prescription by the corporation's predecessors and the realistic solution would
be to consider the application for confirmation as filed by the natural persons-
It should be noted that respondent corporation purchased the land from the transferors, and in accordance with the evidence, confirm their title to the private
Infiels on October 16, 1962 under the aegis of the 1935 Constitution which lands so converted by operation of law and lawfully transferred by them to the
contained no prohibition against corporations holding public lands (except a limit corporation. The law, after all, recognizes the validity of the transfer and sale of
of 1,024 hectares) unlike the later 1973 Constitution which imposed an absolute the private land to the corporation. It should not be necessary to go in a round-
prohibition, Even on the erroneous assumption that the land remained public about way and have the corporation reassign its rights to the private land to
land despite the Infiels' open possession thereof as owners from time
natural persons — (as I understand), was done after the decision in still it is conceded that there is no
the Meralco and Iglesia ni Cristo cases) just for the purpose of complying on prohibition against their sale of the land to
paper with the technicality of having natural persons file the application for the applicant Meralco.
confirmation of title to the private land.
and neither is there any prohibition
against the application being refiled with
MELENCIO-HERRERA, J., dissenting:
retroactive effect in the name of the
original owners and vendors (as such
Section 48 of the Public Land Act, in part, provides: natural persons) with the end result of
their application being granted, because
"SEC. 48. The following described citizens of the
of their indisputable acquisition of
Philippines, occupying lands of the public domain or
ownership by operation of law and the
claiming to own any such lands or in interest therein,
conclusive presumption therein provided
but whose titles have not been perfected or
in their favor.
completed, may apply to the Court of First Instance of
the province where the land is located for It should not be necessary to go through all the rituals
confirmation of their claims and the issuance of a at the great cost of refiling of all such applications in
certificate of title therefor, under the Land Registration their names and adding to the overcrowded court
Act, to wit: dockets when the Court can after all these years
dispose of it here and now." (Paragraphing supplied)
(a) . . .
The effect is that the majority opinion now nullifies the statutory provision that
(b) Those who by themselves or through their
only citizens (natural persons) can apply for certificates of title under Section
predecessors in interest have been in open,
48(b) of the Public Land Act, as well as the constitutional provision (Article XIV,
continuous, exclusive, and notorious possession and
Section 11) which prohibits corporations from acquiring title to lands of the public
occupation of agricultural lands of the public domain,
domain. That interpretation or construction adopted by the majority cannot be
under a bona fide claim of acquisition of ownership,
justified. "A construction adopted should not be such as to nullify, destroy or
for at least thirty years immediately preceding the
defeat the intention of the legislature" (New York State Dept. of Social Services
filing of the application for confirmation of title except
v. Dublino [UST 37 L. Ed 2d 688, 93 S Ct 2507; United States v. Alpers, 338 US
when prevented by war or force majeure. These shall
680, 94 L Ed 457, 70 S Ct 352; cited in 73 Am. Jur. 2nd., p. 351).
be conclusively presumed to have performed all the
conditions essential to a Government grant and shall It has also been said that:
be entitled to a certificate of title under the provisions
of this chapter. "In the construction of statutes, the courts start with
the assumption that the legislature intended to enact
(c) . . . an effective law, and the legislature is not to be
presumed to have done a vain thing in the enactment
Article XIV, Section 11, of the 1973 Constitution, in part, provides:
of a statute. Hence, it is a general principle that the
"SEC. 11. . . . No private corporation or association courts should, if reasonably possible to do so interpret
may hold alienable lands of the public domain except the statute, or the provision being construed, so as to
by lease not to exceed one thousand hectares in give it efficient operation and effect as a whole. An
area; nor may any citizen hold such lands by lease in interpretation should, if possible, be avoided, under
excess of five hundred hectares . . ." which the statute or provision being construed is
defeated, or as otherwise expressed, nullified,
It has to be conceded that, literally, statutory law and constitutional provision destroyed, emasculated, repealed, explained away,
prevent a corporation from directly applying to the Courts for the issuance of or rendered insignificant, meaningless, inoperative,
Original Certificates of Title to lands of the public domain (Manila Electric or nugatory. If a statute is fairly susceptible of two
Company vs. Castro-Bartolome, 114 SCRA 799; Republic vs. Villanueva, 114 constructions, one of which will give effect to the act,
SCRA 875; Republic vs. Court of Appeals, 119 SCRA 449; Iglesia ni Cristo vs. while the other will defeat it, the former construction
Hon. Judge, CFI of Nueva Ecija, Br. 1). It is my opinion that the literalism should is preferred, One part of a statute may not be
be adhered to in this case. construed so as to render another part nugatory, or of
no effect. Moreover, notwithstanding the general rule
The reasoning of the majority can be restated in simple terms as follows: against the enlargement of extension of a statute by
construction, the meaning of a statute may be
(a) The INFIELS can successfully file an application
extended beyond the precise words used in the law,
for a certificate of title over the land
and words or phrases may be altered or supplied,
involved in the case.
where this is necessary to prevent a law from
(b) After the INFIELS secure a certificate of title, they becoming a nullity. Wherever the provision of a
can sell the land to ACME. statute is general, everything which is necessary to
make such provision effectual is supplied by
(c) As ACME can eventually own the certificate of implication." (Pliakos vs. Illinois Liquor Control Com.
title, it should be allowed to directly apply 11 III 2d 456, 143 NE 2d 47; cited in 73 Am. Jur. 2d
to the Courts for the Certificate of Title, pp. 422-423).
thus avoiding the circuituous "literal"
requirement that the INFIELS should first
apply to the courts for the titles, and
The statutory provision and the constitutional prohibition express a public policy.
afterwards transfer the title to ACME.
The proper course for the Court to take is to promote in the fullest manner the
The majority opinion, in effect, adopted the following excerpt from a dissent in policy thus laid down and to avoid a construction which would alter or defeat that
Manila Electric Company vs. Castro Bartolome (114 SCRA 799, 823 [1982]. policy.

"To uphold respondent judge's denial of Meralco's In fine, I confirm my adherence to the ruling of this Court in Meralco vs. Hon.
application on the technicality that the Public Land Castro-Bartolome, 114 SCRA 799 [1982] and related cases.
Act allows only citizens of the Philippines who are
||| (Director of Lands v. Intermediate Appellate Court, G.R. No. 73002,
natural persons to apply for confirmation of their title
[December 29, 1986], 230 PHIL 590-615)
would be impractical and would just give rise to
multiplicity of court actions. Assuming that there was
a technical error in not having filed the application for
registration in the name of the Piguing spouses as the
original owners and vendors,
EN BANC city asserts that there is, and for many years has been, a well-defined
[G.R. No. L-4641. March 13, 1911.] boundary, formerly a wall, latterly an iron fence (enverjada), separating from
the public square the land upon which stands the church belonging to the
THE SEMINARY OF SAN CARLOS, by Pedro Julia, seminary, and that the lands mentioned and described in the documents of
Rector of the Seminary of San Carlos of title presented by the seminary lie between the church and the iron fence and
Cebu, petitioner-appellee, vs. THE MUNICIPALITY OF do not, therefore, extend into the plaza.
CEBU, opponent-appellant.
The land in dispute in this case admittedly lies to the south of the
church belonging to the seminary. This church, according to the plans and the
Rafael Del-Pan and Celestino Rodriguez, for appellant.
evidence, is located upon a plot of ground adjoining the public square on its
J.H. Junquera, for appellee.
north side. So that the land in question, if it does not extend into the plaza,
necessarily lies between it and the church. This clearly appears from the plan,
SYLLABUS
Exhibit J, presented by the petitioner and is a fact undisputed in the record.
1. REALTY; INTERPRETATION OF GENERAL DESCRIPTION
The first question to be determined, then, is whether the land in
OF LAND. — Where a tract of land is described generally as "lying on the
question lies within the inclosure, the southern boundary of which is formed
south of the church," such description does not refer to the church building
by the iron fence, or whether it or any portion of it lies beyond said fence,
itself, but rather to the land on which the church stands.
thereby including a portion of the land used as a public square. The problem
2. ID., ANCIENT DOCUMENTS OF TITLE; SPANISH GRANTS. as to the quantity of the land will resolve itself at the same time with the
— Facts considered relative to the location of the land in dispute, and the determination of the present question.
ancient documents of title dated 1783, 1784 and 1826, and held that the legal
The petitioner has introduced in evidence in this case certified
title to the land is in the seminary.
copies of two documents, the one, Exhibit E, bearing date the 12th day of
3. ID.; TO CONSOLIDATE PRESCRIPTION POSSESSION November, 1783, and the other, Exhibit F, of the date the 4th day of May,
MUST BE THAT OF OWNER. — To consolidate prescription the possession 1784. These documents contain the acts and manifestations of the Audencia
must be that of owner, and it must be public, peaceful and uninterrupted. Acts de Manilaperformed and made for the purpose of formally delivering the
of a possessory character done by virtue of a license or mere tolerance on property which, by royal decree previously published, had been transferred
the part of the real owner are not sufficient. from the ownership of the expatriated Jesuits to that of the Seminary of San
Carlos, or its antecessor. These actuaciones present inventories of all the
4. ID.; ID.; EFFECT OF ACKNOWLEDGMENT OF RIGHTS OF
property thus transferred to the seminary, among such property being the
TRUE OWNER. — Any express or implied acknowledgment which the
lands in question. Each one contains a description of such lands. The
possessor makes with regard to the dominant rights of the true owner interrupts
description contained in Exhibit E is:
the possession held for prescriptive purposes and defeats the operating of the
law granting such rights. "Siguese una huerta grande que esta al
lado de la Iglesia hacia el sur, tiene de largo cuarenta y
5. SPANISH GRANTS; ACTS AND STATEMENTS OF THE
nueve brazas castellanas y de ancho treinta y ocho
POLITICO-MILITARY GOVERNOR OF CEBU BIND THE CITY. — The
brazas."
political and military governor of Cebu, being the official by whose acts the
possession of the land in question was begun and maintained on behalf of the The description contained in Exhibit F of that which was intended
City, all acts and words of his done and spoken at the time, giving color to that to be the same property is:
possession, are binding upon the city and are conclusive as to the character
"Ciudad. Primerante. Dos solares vacios
thereof.
por no vivir en ellos persona alguna lindan por la parte
6. ID.; ID. — When, therefore, a question arising between the del norte con la Iglesia que era dichos regulares la que
parties as to the character and purpose of such possession, the said governor al presente sirve de Catedral contiguos a dicha Iglesia;
in a written instrument acknowledges the superior title of the seminary and por la del sur con la Plaza de la Real Fuerza del San
states the purpose of the possession of the city, the is bound thereby Pedro y casa Real; por la del este con la Marino
inmediatos a ellos; y por la del oeste calle en medio
7. ACTING UPON A WRITTEN INSTRUMENT, BOUND BY ITS
para dicha casa Real con dos solares, que asimismo
TERMS. — When such instrument, although not signed by the seminary, also
pertenecian a dichos regulares y en uno de dichos dos
contains a cession of said land to the city for the purposes of a paseo, such
solares, se halla plantada la casa de mi dicho
success on to continue and to be effective so long as the land is used for
castellano Don Manuel Ignacio del Corro. El motivo por
purposes designated, it is binding upon the seminary, the latter having
que dichos dos solares se hallan vacios, es por que los
accepted the instrument, retained it in its possession for many years, and the
habian ocupado los anunciados regulares con una
same having Been executed as far as possible by both parties, according to its
huerta que tenian la que en el dia ya no existe."
terms, and presented in evidence by the seminary itself.
Exhibit E gives the following as indicating the size of said lots:
DECISION
"Diez y nueve solares y medio de a
veinticinco brazas en cuadro cada solar entero que se
MORELAND, J p:
hallan los quince y medio en la plaza de esta referida
cuidad, y las cuatro restantes en la Isla de Tinago."
The petition in this case, made by the Seminary of San Carlos,
asks for the registration of two pieces of land, included in one plan, In dealing with the question whether or not the above description
petitioner's Exhibit J, located in the city of Cebu, alleging as its source of little include a portion of the public square or plaza, we must not lose sight of the
a royal cession from the King of Spain. fact, already noted, that the city of Cebu asserts that the lands described by
these exhibits lie between the church building and the iron fence to the south
The opponent of the registration, the city of Cebu, denies the title of the church, which iron fence marks, as the city alleges, the termination of
of the petitioner and alleges in itself ownership of the land in question, stating petitioner's land and the beginning of the opponent's. Looking at the plan of
that its title is based upon possession thereof of the kind and for the period the lands in question, Exhibit J, we observe that the church building is
required by law to effect a little by prescription. somewhat close to the iron fence (line A B), being in one place less than 4
The petitioner succeeded in the court below and a judgment was meters distant, in another less than 16 meters, and in another less than 21
entered declaring it the owner of said land and ordering the same registered meters. Now, if the limits of the lands lying in front of the church and between
in its name. The opponent made a motion for a new trial upon the ground that it and the iron fence should be continued so that they would extend as far
the decision was clearly against the evidence and was contrary to law. This east and west as does the line A B, and then lines should be drawn to inclose
motion was denied and the opponent duly excepted and perfected its appeal the land thus comprehended between the church and the line A B, there
to this court. would be within that inclosure all of the land which the city claims is described
by the petitioners muniments of title. Simple arithmetic will determine whether
The land in controversy is situated in the city of Cebu. It is the city's contention in this regard is correct or not. The line A B, which is the
claimed by the seminary that it includes a portion of one of the public squares southern boundary of the land thus inclosed, is 87.40 meters long. The north
of that city. Aside from that of the ownership of the land, there are two boundary line of said land is, under the city's contention, the line of the walls
questions raised on the trial and presented for review on this appeal. The first of the church building. From the plan it will be readily seen that said line is
one relates to the quantity of the land claimed by the seminary, and the very broken, following, as the city claims it does, the walls of the church. The
second to its precise location. The claim of the city is that, even admitting strip of land inclosed as above set forth, therefore, varies in width, growing
petitioner's ownership of the land described in its muniments of title, still the narrower as the walls of the church approach the iron fence and wider as they
city must have a judgment in its favor for the reason that, from the description recede from it. At its widest place it can not exceed, as we have before seen,
presented in said muniments, it appears clearly that the land in question does 21 meters. In some places it is less than 4 meters wide, and still others less
not extend into or include any portion of the plaza occupied by the city. The
than 16. This being so, it will be more than fair to the contention of the city to contention shows its weakness. Lastly: The witness for the city, Sr. Rallos,
assume, for the purposes of a mathematical demonstration, that the strip of stated that, in his boyhood, there was, at the south of the church and at a
land comprehended between the church and the iron fence (line A B) is 87.40 considerable distance therefrom a wall which ran along the premises upon
meters long and 21 meters wide. The area of the strip is, therefore, 1,835.4 which the church was located, forming, apparently, the southern boundary
square meters. This is in reality a larger area than any piece of land lying line thereof; that between that wall, the place of which was later taken by the
between the church and the iron fence could possibly have, and, therefore, as iron fence several times referred to, and the fence which surrounded the
we have before said, our assumption is more than fair to the city's contention. monument of Magellan there intervened a distance of from 25 to 30 brazas.
Now, if we take the description of land as found in one of the petitioner's The wall referred to by the witness is clearly not the wall of the church
exhibits wherein it speaks of it being 49 Spanish brazas long and 38 Spanish building. It appears from his testimony also that paseo de Dolores was
brazas wide, we have a parcel of land containing more than 4,000 square located to the south of the wall or iron fence; and that, therefore, the land in
meters, or more than twice the size of the largest parcel that could possibly dispute, which includes said paseo, could not have been located between the
be contained between the church and the iron fence. If, on the other hand, we church building and the iron fence. The same facts appear from the testimony
take the description as it is found in the other exhibit of the petitioner, wherein of other witnesses of the city.
the land is described as two parcels of land, each 25 brazas square, we
Under the evidence, then we must conclude, with the learned
readily see that such a parcel of land could not possibly be contained
trial court, that a portion of the land now occupied by the city of Cebu as a
between the church and the iron fence, as its area would be more than 3,000
public plaza is land described in the petitioner's exhibits, and is so much of
square meters. Inasmuch as the strip of land comprehended between the
said land as is contained in the plan marked petitioner's Exhibit J. As to a
church and the iron fence could not, as we have seen contain more than
paper title to said land so occupied as a public square, the city has presented
1,835.4 square meters, it is, therefore, evident that the land described in
none. To that land it shows no documentary or record title whatever. The
petitioner's exhibits far exceeds in extent that lying between the church and
paper title of the petitioner to such land is entirely unmet and uncontradicted.
the iron fence into what is known as the public plaza, it being admitted that
The city presents no rights to or interest in the same except that acquired by
the iron fence marks the northern boundary of said plaza.
long years of actual occupation. It signifies no source from which comes any
The correctness of the plan, Exhibit J, as prevented by the right or interest and asserts no ability to disclose any. Its ownership is based
petitioner, has not been impugned by the evidence in any manner which is upon prescription solely.
beneficial to the city. There appears to be a large difference between the
Were it not for petitioner's Exhibit K, we would be inclined to hold,
amount of land as described in one of the petitioner's exhibits and that
the record, that the opponent's occupation had ripened into a title by
included within the plan, the latter containing considerably less than the
prescription. We would be disposed to say that the proof's show that the city
amount set out in the description in said exhibit. Apart from this inconsistency,
has occupied the lands in question adversely since the year 1863, and that,
an inconsistency by which the city seems to profit, the correctness of the plan
by reason thereof, it has acquired title under the provisions of the Civil Code
is not questioned in the record. As to the discrepancy existing between the
relating to prescription. In view of that Exhibit, however, we do not find it
amount of land as described in the other exhibit of the petitioner and that
necessary to determine that question, inasmuch as, in our judgment that
presented by the plan, the difference is so slight as to be substantially
exhibit demonstrates conclusively that such occupation was permissive and
negligible. The land described in the plan contains 3, 576 square meters;
not adverse, was under license and not under claim of right, and could not,
while under the description in petitioner's Exhibit F the land contains about
therefore, be made the basis of a prescriptive title.
3,494.4 square meters, a difference of only 82.4 square meters.
Before presenting that exhibit, it should be said that the proofs
Even though what we have already said be taken in its broadest
uncontradictedly demonstrate that the possession by the city of the land in
sense, it does not entirely dispose of the city's contention. Even though it be
question was initiated and maintained, down to the year 1980, when
admitted, as we have found, that the land shown be petitioner's muniments of
the ayuntamiento of Cebu was formed, by the political and military governor
title to have belonged once, to it includes a portion of the public square
of Cebu; that every act of possession and dominion exercised during that
referred to, the query naturally suggests itself, how much of the plaza does it
time over said land was exercised by his orders; that the improvements made
include. While we have already intimated that the amount of the public square
thereon were made by his direction and with provincial moneys; that, so far
included in the description of petitioner's land is so much thereof as is
as appears from the record, not a single act of possession or dominion over
described in the plan Exhibit J, such intimation, embracing, as it does, a
said land was exercised except through him during all the years from 1863
conclusion from facts not presented, needs some explanation to make it
down to 1890. From the latter year to the time of the American occupation,
clear; for, even though it be conceded that such land extends into public
the ayuntamiento, according to the evidence, seems to have maintained the
square, still it will be found to extend thereinto more or less, depending upon
possession.
whether the measurement be taken from the iron fence or from the walls of
the church building. In other words, if the measurement be made from the As to said exhibit: On or about the 18th day of June, 1869, there
iron fence southward, there will be included more of the square than if it be seems to have arisen a controversy over the possession of the land in
made from the walls of the church building. It is the city's contention that, in question. Although, apart from the document when executed, the evidence is
measuring the lands claimed by the petitioner and described in its exhibits, somewhat uncertain and vague as to what actually happened, nevertheless,
such measurement must be taken from the very walls of the church building from what appears, it is a necessary deduction that the seminary, on or about
and not from the iron fence. For this reason the city asserts that it is one of that date, interferred with the possession of the city and requested that the
the vices of Exhibit J that its measurements begin with the iron fence and not matter be adjusted to the end that the rights of the parties in the land might be
the walls of the church, thereby including much more of the public square made clear. The political and military governor of Cebu being, as we have
than would be included if the measurements were taken from the iron fence. seen, the official under whose orders and directions possession of the land
In support of this claim the city refers to the wording of the descriptions of the had been taken and maintained, the officials of the seminary naturally went to
land as shown in the two exhibits, already presented, wherein the land is him for an adjustment. As a result of that adjustment the following document,
spoken of as "una huerta grande que esta al lado de la Iglesia hacia el sur," petitioner's Exhibit K, was executed by the political and military governor of
and also as a parcel of land "linda por la parte del norte con la Iglesia." Cebu, Joaquin Monet:
We do not believe that this contention is sound. In the first place, PETITIONER'S EXHIBIT K.
under the general rules regulating the construction of words and phrases in
cases of this character, the word "church," as used in the description, refers "Don Joaquin Monet y Estevez, military and
to the land upon which the church stands, and not to the church building political governor, with the assistance of the attesting
itself. In the second place, it is unquestioned that, under the description witnesses, etc., does hereby certify, in due form, that
referred to, the north line of the lands in question, as well as the other lines, is part of the Plaza Urbiztondo included in the work which
straight, the lots being each 25 Spanish brazas square. But, as we have is now being done for a public promenade to be known
already seen, the line made by the south wall of the church, which, under the as "Dolores" belongs to the Seminary of San Carlos
city's contention, would be the northern boundary line of the land in question, and the Chaplaincy of the Cathedral, according to the
is extremely broken an crooked. This is clearly shown by the plan Exhibit J, in topographical plan of this city on file in the archives of
which the line of the said south wall presents no less than six angles. It is, the said Seminary, which said land has been ceded by
therefore, impossible that the northern boundary line of the land in question the rector, Jose Casaramona, and the devout parish
be the line of the church wall, the one being a straight line and the other a priest, Leon Esequiel, the persons charged with the
crooked one. Such lines can not be coincident. For the same reason it would preservation of the property belonging to their
be impossible to determine from which the portion of the wall the respective trusts, this as long as the said promenade
measurement of the land in question should be taken. Should it be taken from which embellishes the town exists as such.
the eastern end of the wall, or from the western end, which is about 15 "In testimony thereof, and at the request of
meters farther south, or from the central portion of the wall, which is about 10 the interested parties, I issue these presents in triplicate
meters still farther south? It is apparent that, if the point of departure for at Cebu this eight day of June, one thousand eight
measuring the 25 brazas is to be the wall of the building itself, that point is hundred and sixty-nine,
variable in its location, depending upon which portion of the wall is taken as
the point from which the measurement is made. The mere statement of this (Signed) "JOAQUIN MONET.
(Signed) "LORENZO ESPINA. was executed, it also must be rejected. We are confident that the said
document is a genuine document and that Joaquin Monet was governor of
(Signed) "CALIXTO DEL CAPA. Cebu at the time the same was signed. That he was such governor at such
time is clearly demonstrated by the official "Gaceta de Manila," dated June
The instrument, in our opinion, conclusively characterizes the 22, 1869, in which, on page 1207, appears the name of Joaquin Monet as
occupation of the land in question and renders untenable the position of the political and military governor of Cebu. On that page is seen an official
city that its possession was adverse and under claim of right and should be document formulated and signed by him as such governor on the 5th day of
made the basis of prescriptive title under the Civil Code. That code provides: June 22, 1869. In the said "Gaceta de Manila" of July 22, 1869, at page 164,
"ART. 1941. Possession must be in the appears another official document formulated and signed by said Joaquin
capacity of an owner, public, peaceful, uninterpreted. Monet, as governor of Cebu, which document bears date the 5th day of July,
1869. Both of these writings were official documents made and signed by him
"ART. 1942. Acts of a possessory in the course of the performance of his official duties as political and military
character, performed by virtue of a license, or be mere governor of Cebu. In this connection it must be remembered that the disputed
tolerance on the part of the owner, are of no effect for document, Exhibit K, bears date the 8th day of June, 1869, only three days
establishing possession." after the date borne by said document of June 5th above mentioned. It should
"ART. 1948. Any express or implied also be noted that, in writing the official communications found in Exhibit L,
acknowledgment which the possessor may make with bound volume of official communications, Joaquin Monet was acting as
regard to the right of the owner also interrupts governor interino of the politico-military government of the Visayan Islands. In
possession." signing the disputed document he acted as the political-
military governor (not interino) of Cebu, not the Visayan Islands. Most of the
The law presented by these articles is substantially the same as argument of the learned counsel for the city in relation to the second
that laid down by the American courts. It is there generally held that "any act contention is founded in a failure to observe the distinction between the
of recognition or acknowledgment of a superior title in another during the official character of Joaquin Monet as manifested in Exhibit L and that shown
period of adverse possession, will, as a general rule, amount to an in Exhibit K.
interruption of the continuity of the possession and defeat the operation of the
statute. Exhibit K, however, must be taken as it reads. Its benefits and its
obligations go hand in hand. If the seminary asks the advantages which it
The political and military governor of Cebu, being the official by confers it can not escape the obligations which it imposes. That instrument
whose acts the possession of the land in question was begun and maintained not only gives character to the possession which the city then had and
on behalf of the city, particularly prior to at the time of the execution of the thereafter claimed to have of the land in dispute, rendering that possession
above instrument, all acts and words of his giving color to that possession are incapable of being used as the basis of prescriptive rights, but it also gives
binding upon the city and conclusive as to the guilty thereof. the city the right to continue in that possession as long as it remains of the
The city seeks to evade the legal effects of this document in two kind described therein, namely, "as long as the said promenade which
ways: First, by asserting that it has not been properly proved for admission as embellishes the town exists as such." In other words, it makes, or, at least,
evidence, in that the signature of Joaquin Monet was not shown to be recognizes as already made, a cession of the use of the land in question to
genuine; and, second, be contending that said Joaquin Monet was not, as a the city of Cebu so long as the same continues to be used in the manner in
matter of fact, political and military governor of Cebu at the time that the which it was then being used or for the purposes for which it was then being
document bears date. prepared to be used. There was a legal consideration for this obligation, thus
rendering it enforceable, even though it be urged that the transaction could
Neither of these contentions can be maintained. The document in not have been called a voluntary dedication to public use. Nor may it now be
question having been executed in the year 1869, and having subsequently contended that the church officials who appear to have taken part in the
been generally acted upon as genuine by persons having an interest in the transaction were not authorized and had no power to bind the seminary or the
subject matter involved, and its custody having been presumed satisfactorily church. That instrument has long been record of the church, known to it
explained, its genuineness must be presumed under the provisions of section through all its officials, and has been presented by the church in this litigation
334, paragraph 34, of the Code of Civil Procedure. Moreover, section, 326 as an efficacious and enforceable instrument. Its benefits may not be
and 327 of said Code provide that: interpreted to bind and loose at the same time to the same party.
"SEC. 326. When other evidence of the For these reason it is our opinion that the petitioner has the legal
execution of writing need not be produced. — Where a title to the land in dispute, but that the city of Cebu has the right to the
writing is more than thirty years old, and evidence is possession, occupation and use of said land for the purposes above
given that the party against execution, or where the expressed and presented in said Exhibit K, above quoted, to be so occupied
writing is one produced from the custody of the adverse and possessed so long as said land is dedicated to the uses and purposes
party, and has been acted upon by him as genuine, no therein expressed.
other evidence of the execution need be given.
The judgment of the court below is modified and the land
"SEC. 327. Proof of handwriting. — The described in Exhibit J is hereby ordered registered in the name of the
handwriting of a person may be proved by anyone who petitioner, but such registration is subject to and must affirmatively show the
believes it to be his, and has seen him write, or has rights of the City of Cebu to the possession, occupation and use of said land
seen writing purporting to be his, upon which he has as hereinabove set forth. As modified, said judgment is affirmed.
acted, or been charged, and he has thus acquired
knowledge of his handwriting. Evidence respecting the Arellano, C.J., Mapa, Carson and Trent, JJ., concur.
handwriting may also be given by comparison, made by ||| (Seminary of San Carlos v. Municipality of Cebu, G.R. No. L-4641, [March 13,
the court, with writings admitted or treated as genuine 1911], 19 PHIL 32-46)
by the party against whom evidence is offered, or
proved to be genuine to the satisfaction of the judge.
Where the writing is more than thirty years old, the
comparisons may be made with writings purported to be
genuine, and generally respected and acted upon as
such by persons having an interest and knowing the
fact."
For the purpose of making the comparisons mentioned in the last
section, the petitioner offered in evidence a bound volume of official
communications from the political and military governor of Cebu to the Bishop
of Cebu. Several of said official communications, bearing dates November
20, 1867, December 14, 1867, January 9, 1968, February 13, 1868, February
17, 1868, February 22, 1868, and November 11, 1868, are signed by said
Joaquin Monet as "El gobernador interino" of the politico-military government
of the Visayas. The signatures to those documents are genuine. From a
comparison of the signature on Exhibit K with those on the official
communications, it is manifest, as the learned trial court found, that the
signature on the exhibit is genuine. Moreover, that document, as a whole,
bears every evidence of being genuine.
As to the second contention of the city that Joaquin Monet was
not, in reality, political-military governor of Cebu at the time said instrument
THIRD DIVISION have bound third persons like Lucia Bautista because of the basic civil law principle
of relativity of contracts which provides that contracts can only bind the parties who
[G.R. No. 111027. February 3, 1999.] had entered into it, and it cannot favor or prejudice a third person. This basic principle
applies even if the sales were supposedly concluded at a time prior to the operation
BERNARDINO RAMOS and ROSALIA of the Torrens system of land registration over the properties involved.
OLI, petitioners, vs. COURT OF APPEALS,
RODOLFO BAUTISTA and FELISA 3. ID.; LAND TITLES AND DEEDS; CADASTRAL ACT; ORIGINAL
LOPEZ, respondents. CERTIFICATES OF TITLE ISSUED THEREUNDER SHALL HAVE SAME EFFECT
AS CERTIFICATES ISSUED UNDER LAND REGISTRATION ACT. — Under the
Jose de Luna for petitioners. Cadastral Act, the original certificates of title issued to the original registrant, shall
Jose T. Antonio for private respondents. have the same effect as certificates of title granted on application for registration of
land under the Land Registration Act, because "no title to registered land in
SYNOPSIS derogation to that of the registered owner shall be acquired by prescription or adverse
possession." Pedro Tolentino and petitioners, as the former's alleged successors-in-
On January 9, 1940, Lucia Bautista, predecessor of private respondents, interest, have therefore no valid claim of ownership over the property, particularly
in the cadastral proceedings involving lots 572 and 579 of the Gattaran Cadastre, since petitioners simply failed to substantiate the nature and extent of Tolentino's
was issued OCT Nos. 17811 and 17812. Thirty-six (36) years later, or on January 8, rights and interests over the lots. Such being the case, the conveyances in their favor
1976, petitioners, as buyers of the subject lots, filed an action for reconveyance with were void as the subject properties were lawfully owned by another person.
damages against respondents spouses. They alleged that they bought the lots from
Pedro Tolentino, claimants of the lots evidenced by two "Escritura de Compra Venta." 4. ID.; ID.; LAND REGISTRATION ACT; ONE YEAR PERIOD FROM
Petitioners presented certified copies thereof claiming that the originals were lost in ENTRY OF DECREE TO FILE PETITION FOR REVIEW. — Section 38 of the Land
a fire that gutted the office of their counsel. They failed to present any person who Registration Act provides that a decree of registration duly issued is subject "to the
could have witnessed the execution of the documents and likewise failed to prove right of any person deprived of land or of any estate or interest therein by decree of
that those documents were later registered. The trial court dismissed the complaint. registration obtained by fraud to file in the competent Court of First Instance (now the
It found that petitioner Bernardino Ramos failed to file an answer in the cadastral Regional Trial Court) a petition for review within one year after entry of the decree,
proceedings and also failed to avail of any petition to reopen proceedings, hence, provided no innocent purchaser for value has acquired an interest." The same law
laches had set in. It ruled that a title becomes indefeasible and incontrovertible after provides that upon the expiration of the term of one year, "every decree or certificate
the expiration of one year from entry of the final decree of registration; and that of title . . . shall be imprescriptible."
reconveyance may only take place if the land that is claimed to be wrongfully
5. ID.; PRESCRIPTION OF ACTIONS; ACTION FOR
registered is still registered in the name of the person who procured the wrongful
RECONVEYANCE RESULTING FROM FRAUD PRESCRIBES IN FOUR (4)
registration. The decision was affirmed on appeal by the Court of Appeals.ECcTaS
YEARS. — Under the law, an action for reconveyance of real property resulting from
Unregistered documents bind only the parties thereto and cannot fraud prescribes in four (4) years from the discovery of the fraud. Discovery of the
operate against the whole world because of the basic civil law principle of relativity of fraud must be deemed to have taken place when Lucia Bautista was issued OCT
contracts which provides that contracts can only bind the parties who had entered Nos. 17811 and 17812 because registration of real property is considered a
into it, and it cannot favor or prejudice a third person. Thus, failure to register the "constructive notice to all persons" and it shall be counted "from the time of such
"Escritura De Compra Venta" resulted in the sale binding between the vendee and registering, filing or entering." An action based on implied or constructive trust
the vendor alone and cannot bind their successors-in-interest. prescribes in ten (10) years. This means that petitioners should have enforced the
trust within ten (10) years from the time of its creation or upon the alleged fraudulent
An action for reconveyance of real property resulting from fraud registration of the property. But as it is, petitioners failed to avail of any of the
prescribes in four (4) years from discovery of the fraud. The period is counted from aforementioned remedies within the prescribed periods. With no remedy in view, their
date of issuance of the original certificate of title which the law considers "constructive claims should forever be foreclosed. IaTSED
notice to all persons." An action based on implied or constructive trust prescribes in
ten (10) years. Failure to avail of any of the remedies within the prescribed periods 6. ID.; LAND TITLES AND DEEDS; TORRENS SYSTEM OF LAND
foreclosed their claims. REGISTRATION; HOLDERS OF TITLE, PROTECTED. — Private respondents have
in their favor the law that protects holders of title under the Torrens System of land
Inattention to titled property does not constitute abandonment for holders registration. As this Court so eloquently pronounced in 1915: "Once a title is
of the title have in their favor the protection of the law. HSCcTD registered, the owner may rest secure, without the necessity of waiting in the portals
of the court, or sitting in the 'mirador de su casa,' to avoid the possibility of losing his
SYLLABUS land. TAacIE

1. REMEDIAL LAW; EVIDENCE; ESCRITURA DE COMPRA VENTA, DECISION


AN ANCIENT DOCUMENT; EXISTENCE THEREOF NOT DULY PROVED IN CASE
AT BAR. — The two documents denominated as Escritura de Compra Venta which ROMERO, J p:
were executed in 1939 would have well qualified as ancient documents since they May the heir of the original registrant of parcels of land under the Torrens
were already in existence for more than thirty years in 1976 when the case for System, be deprived of ownership by alleged claimants thereof through acquisitive
reconveyance was initially filed. The original documents, however, were not prescription? aisadc
presented in evidence as these had been apparently lost in the fire that gutted the
office of petitioners' counsel. Under the circumstances, it should have been the duty Impugned in this petition for review on certiorari is the Decision 1 of the
of petitioners therefore to prove the existence of the documents in accordance with Court of Appeals which affirmed in toto that of the Regional Trial Court of Aparri,
Rule 130 of the Revised Rules of Court. It appears that the loss of the two documents Cagayan, Branch VIII, 2 disposing of Civil Case No. VIII-7, an action for
of sale was shown by testimonial evidence of petitioners' counsel, Atty. MacPaul B. reconveyance with damages, as follows:
Soriano, whose law office was burned. Upon realizing that the documents involved
"WHEREFORE, in view of all the foregoing,
here had been irretrievably lost because of the fire, Atty. Soriano suggested to
judgment is hereby rendered as follows:
petitioners that they should see their other lawyer, Atty. Laggui, who could provide
them with certified true copies thereof. This certification, however, does not imply that 1. Ordering the dismissal of the instant case;
the documents certified to were authentic writings although it proves the existence of
the documents purportedly evidencing the sale. Rule 132 provides the manner by 2. The defendants are hereby declared
which the due execution and authenticity of private writings like the deeds involved absolute owners of the land described in paragraph 2 of
here, should be established. Unfortunately for petitioners, the documents upon which the complaint, Lot No. 572 and Lot No. 579 Gattaran
they relied in establishing their claim of ownership, had not been duly presented in Cadastre, Gattaran, Cagayan;
evidence in accordance with the aforecited Rule. They failed to present any person
who could have witnessed the execution of the documents, like the instrumental 3. The affidavit of Self-Adjudication (Exhibit
witnesses thereof. Understandably, they could not even demonstrate the '6') and Transfer Certificate of Titles Nos. T-31699 and
genuineness of the signatures of the parties to the sale because the copies they T-31698 (Exhibit '7' & '8') are hereby declared valid; and
offered in evidence did not bear those signatures. Consequently, under the Rules of
Court, the documents' authenticity and due execution are suspect and may not be 4. Ordering the heirs of the late Bernardino
given that much weight. Ramos and other persons acting in their behalf, to refrain
2. CIVIL LAW; OBLIGATIONS AND CONTRACTS; RELATIVITY OF from molesting or disturbing the possession and
CONTRACTS; PRIVATE DOCUMENTS BIND ONLY IMMEDIATE PARTIES. — ownership of the defendants of the land described in
Assumingarguendo that the existence of the documents was properly established, paragraph 2 of the complaint, designated as Lot 572 and
still, the supposed agreement embodied in the two documents bound only the parties Lot 579 Gattaran Cadastre, Gattaran, Cagayan, covered
thereto, namely Pedro Tolentino and the petitioners, because the latter failed to prove by Original Certificate of Titles Nos. 17811 and 17812
that these were later registered as to operate against the whole world. They could not
which was (sic) cancelled by Transfer Certificate of Titles by final judgment of the Court (sic). Diaz vs. Rodriguez,
Nos. T-31699 and T-31698. L-20300-01 and Republic vs. Court of Appeals, L-20355-
56, April 30, 1965, 13 SCRA 704.
No pronouncement as to costs and
damages. In the same vein, it is a settled rule that a
party seeking the reconveyance to him of his land that he
SO ORDERED." claims had been wrongfully registered in the name of
another person, must recognize the validity of the
as well as the resolution of July 1, 1993, denying reconsideration thereof. certificate of title of the latter. It is also a settled rule that
The records disclose the following antecedent facts: a reconveyance may only take place if the land that is
claimed to be wrongfully registered is still registered in
On March 14, 1939, Pedro Tolentino, claiming absolute ownership over the name of the person who procured the wrongful
Lot Nos. 572 and 579 of the Gattaran cadastre in Lapogan, Gattaran, Cagayan, registration. No action for reconveyance can take place
separately sold said lots to petitioners, the spouses Bernardino Ramos and Rosalia as against a third party who acquired title over the
Oli, in consideration of the amount of eighty pesos (P80.00) for each sale. The registered property in good faith and for value. Defendant
aforesaid conveyances were allegedly evidenced by two documents both entitled Rodolfo Bautista fittingly steps into the shoes of an
"Escritura de Compra Venta" 3 and acknowledged before a notary public. innocent third person." [Underscoring supplied].

Subsequently, however, petitioners instituted on January 8, 1976 an Dissatisfied with the trial court's disposition of the case, petitioners
action for reconveyance with damages 4 alleging that while they were "in open, seasonably appealed the same to the Court of Appeals. The appellate court,
public, adverse, peaceful and continuous possession" of the subject lots "in good faith however, found the conclusions reached by the trial court in accord with law and the
and with just title, for not less than fifty (50) years, personally and through their evidence presented, hence, it affirmed the same in toto on October 23, 1992. Having
predecessors-in-interest," they were surprised to discover in November 1975, that been denied reconsideration, petitioners interposed the instant petition for review
decrees of registration 5 covering Lot Nos. 572 and 579 were already issued on on certiorari alleging the following as grounds therefor:
January 7, 1940. They complained further the subsequent issuance by the Register
of Deeds of Cagayan on March 11, 1941, Original Certificates of Title Nos. 17811 1. RESPONDENT COURT OF APPEALS ERRED AND ACTED WITH
and 17812 covering Lot Nos. 572 and 579, respectively, in favor of Lucia Bautista GRAVE ABUSE OF DISCRETION IN AFFIRMING IN TOTO THE DECISION OF
since the latter allegedly neither laid claim of ownership nor took possession of them, THE TRIAL COURT WHICH FOUND BY MERE PRESUMPTION THAT PRIVATE
either personally or through another. Petitioners claimed instead that they were the RESPONDENTS ARE IN POSSESSION OF THE LAND IN SUIT WHEN THE
ones who acquired prior ownership and possession over the lots to the exclusion of FACTS ADDUCED DURING THE TRIAL CLEARLY PROVED THAT PETITIONERS
the whole world. Thus, they concluded that the original certificates of title as well as HAVE BEEN IN POSSESSION THEREOF FOR MORE THAN 30 YEARS.
Transfer Certificates of Title Nos. T-31698 and T-31699 obtained by private
respondent Rodolfo Bautista who adjudicated unto himself said lots on September 2. RESPONDENT COURT OF APPEALS ERRED AND ACTED WITH
20, 1975, as sole heir of Lucia Bautista 6 were null and void. On the theory that they GRAVE ABUSE OF DISCRETION IN FINDING THAT THE INSTANT ACTION FOR
already acquired the subject lots by acquisitive prescription, petitioners demanded RECONVEYANCE INSTITUTED BY PETITIONERS HAD ALREADY PRESCRIBED.
their return but private respondents refused to do so, hence, compelling them to file
3. RESPONDENT COURT OF APPEALS ERRED IN CONFORMING
a complaint for reconveyance with damages.
WITH THE TRIAL COURT'S DECISION THAT RECONVEYANCE WILL NO
On the other hand, herein private respondents, the spouses Rodolfo LONGER PROSPER IF THE LANDS IN SUIT HAD ALREADY BEEN
Bautista and Felisa Lopez, likewise claimed absolute ownership of the lots covered TRANSFERRED TO A THIRD PERSON IN GOOD FAITH AND FOR VALUE WHEN
by TCT Nos. T-31698 and T-31699. They alleged that while the records of the Bureau THE FACTS SHOW THAT PRIVATE RESPONDENTS HAD ADMITTED THEY
of Lands showed that during the cadastral survey in Gattaran in 1932, Pedro ALLEGEDLY INHERITED THE LANDS IN SUIT AND THEREFORE THEY ARE NOT
Tolentino was a claimant over lands in the cadastre, the same was only with respect THIRD PARTIES.
to Lot No. 1399 which was eventually titled under his name as OCT No. 16110. It just
We sustain the appellate court's decision.
happened that Lot No. 1399 was adjacent to Lot No. 572, a portion of which was
occupied by petitioners upon the tolerance of the original registrant Lucia Bautista. Inasmuch as petitioners anchor their claim of ownership over the parcels
of land on the alleged deeds of sale executed by Pedro Tolentino in their favor, we
By way of affirmative defense, private respondents maintained that the
believe that the issue of the authenticity and binding effect of those documents should
action for reconveyance filed by petitioners was tantamount to a reopening of the be addressed at the outset. LLjur
cadastral proceedings or a collateral attack on the decrees of registration which
cannot be done without violating the rule on conclusiveness of the decree of The two documents denominated as Escritura de Compra Venta which
registration. Moreover, they argued that since the lots were already under the were executed in 1939 would have well qualified as ancient documents 7 since they
operation of the Torrens System, acquisitive prescription would no longer be possible. were already in existence for more than thirty years in 1976 when the case for
reconveyance was initially filed. The original documents, however, were not
After due proceedings, the trial court dismissed petitioners' complaint
presented in evidence as these had been apparently lost in the fire that gutted the
underscoring the fact that during the cadastral proceedings in 1940, Bernardino
office of petitioners' counsel. Under the circumstances, it should have been the duty
Ramos did not file an answer for the two lots although he was allegedly the claimant
of petitioners therefore to prove the existence of the documents in accordance with
and possessor thereof under the deeds of sale executed by Pedro Tolentino in his
Rule 130 of the Revised Rules of Court which states:
favor on March 14, 1939. Since it was only Lucia Bautista who filed an answer and
who appeared to be the lawful claimant in the proceedings, she was therefore issued "SECTION 5. When original document is
original certificates of title for the subject lots. The trial court presumed that everyone unavailable. — When the original document has been
was notified about the proceedings inasmuch as cadastral proceedings are in rem. lost or destroyed, or can not be produced in court, the
More notably, within one year from the issuance of the decree of registration on offeror, upon proof of its execution or existence and the
January 9, 1940, Bernardino Ramos likewise failed to avail of a petition to reopen the cause of its unavailability without bad faith on his part,
proceedings on the ground of fraud as he subsequently alleged in his belated action may prove its contents by a copy, or by a recital of its
for reconveyance. Consequently, when the action for reconveyance was finally filed, contents in some authentic document, or by the
more than thirty-six (36) years had already elapsed and laches had set in. The trial testimony of witnesses in the order stated."
court ruled in this wise:
It appears that the loss of the two documents of sale was shown by
"The settled rule on the indefeasibility and testimonial evidence of petitioners' counsel, Atty. MacPaul B. Soriano, whose law
incontrovertibility of the title after the expiration of one office was burned. Upon realizing that the documents involved here had been
year from the entry of the final decree of registration, now irretrievably lost because of the fire, Atty. Soriano suggested to petitioners that they
bars the plaintiffs from availing this action for should see their other lawyer, Atty. Laggui, who could provide them with certified true
reconveyance; the property in question not having been copies thereof. 8 Thus, the copies of the documents that petitioners presented in
satisfactorily shown that same was wrongfully titled to in court each contained the following certification:
the name of Lucia Bautista. Accordingly, her titles
thereto, Exhibit '4' and Exhibit '5', are therefore valid. By "CERTIFICATION
operation of law Transfer Certificate of Title Nos. 31699
and 31698 in the name of Rodolfo Bautista (Exhibit '7' & I, ANTONIO N. LAGGUI, Notary Public for
'8') are also valid. The defendant Rodolfo Bautista is a and in the Province of Cagayan, hereby certify that the
possessor with a Torrens title who is not aware of any foregoing is a true, correct and literal copy of the original
flaw of his title which invalidates it, is considered copy of Doc. No. 1, Page No. 44, Book No. 1, Series of
possessor in good faith and his possession does not lose 1939 of the Notarial Register Luis Rosacia, shown to me
this character except in the case and from the moment by, and in possession of Bernardino Ramos."
This certification, however, does not imply that the documents certified father never declared the lots for taxation purposes and neither did they ever pay real
to were authentic writings although it proves the existence of the documents property taxes thereon. In short, the alleged mortgage papers could very well refer to
purportedly evidencing the sale. Rule 132 provides the manner by which the due properties other than Lot Nos. 572 and 579 and that the trial court correctly ruled that
execution and authenticity of private writings like the deeds involved here, should be what petitioners proved can not ripen into ownership "in derogation to that of the
established. Thus: registered owner." 13

"SECTION 20. Proof of private document. Petitioners' supposed possession of the lots for more than forty (40)
— Before any private document offered as authentic is years, therefore, stands as a bare claim with nothing whatsoever to prop it up. Under
received in evidence, its due execution and authenticity the circumstances of the case, they would only succeed upon sufficient evidence to
must be proved either: support their allegation that fraud attended the registration of the property in Lucia
Bautista's name. As it is, however, petitioners failed to present evidence on the matter
1. By anyone who saw the document thereby leaving their claim barren. LexLib
executed or written; or;
In contrast, private respondent Rodolfo Bautista's claim to the properties
2. By evidence of the genuineness of the registered under the Torrens system which he traces to his aunt, Lucia Bautista,
signature or handwriting of the maker; appears incontrovertible. Under the Cadastral Act, the original certificates of title
issued to the original registrant, shall have the same effect as certificates of title
Any other private document need only be granted on application for registration of land under the Land Registration Act,
identified as that which it is claimed to be." because "no title to registered land in derogation to that of the registered owner shall
be acquired by prescription or adverse possession." 14 Pedro Tolentino and
Unfortunately for petitioners, the documents upon which they relied in
petitioners, as the former's alleged successors-in-interest, have therefore no valid
establishing their claim of ownership, had not been duly presented in evidence in
claim of ownership over the property, particularly since petitioners simply failed to
accordance with the aforecited Rule. They failed to present any person who could
substantiate the nature and extent of Tolentino's rights and interests over the lots.
have witnessed the execution of the documents, like the instrumental witnesses
Such being the case, the conveyances in their favor were void as the subject
thereof. Understandably, they could not even demonstrate the genuineness of the
properties were lawfully owned by another person. 15
signatures of the parties to the sale because the copies they offered in evidence did
not bear those signatures. Consequently, under the Rules of Court, the documents' Neither may petitioners' argument that private respondent Rodolfo
authenticity and due execution are suspect and may not be given that much weight. Bautista, being the son-in-law of Pedro Tolentino, was bound by the sale and
therefore he and his present wife hold the properties in trust for petitioners'
Furthermore, assuming arguendo that the existence of the documents
successors-in-interest hold. On that basis, they aver that their right to claim the
was properly established, still, the supposed agreement embodied in the two
property in trust is imprescriptible.
documents bound only the parties thereto, namely Pedro Tolentino and the
petitioners, because the latter failed to prove that these were later registered as to But petitioners' argument would only be tenable upon proof that the
operate against the whole world. They could not have bound third persons like Lucia property was acquired through mistake or fraud. As earlier observed, however,
Bautista because of the basic civil law principle of relativity of contracts which petitioners' claim of fraud was never substantiated and, hence, it has remained a
provides that contracts can only bind the parties who had entered into it, and it cannot groundless charge. Consequently, petitioner's claim of imprescriptibility of the action
favor or prejudice a third person. 9 This basic principle applies even if the sales were for reconveyance is baseless.
supposedly concluded at a time prior to the operation of the Torrens system of land
registration over the properties involved. When the properties were eventually titled Section 38 of the Land Registration Act provides that a decree of
in favor of Lucia Bautista, the sale between Pedro Tolentino and petitioners could not registration duly issued is subject "to the right of any person deprived of land or of
have affected Lucia Bautista and her successor-in-interest because the pertinent law any estate or interest therein by decree of registration obtained by fraud to file in the
in point, Act No. 496, as amended by P.D. No. 1529 unequivocably provides: competent Court of First Instance (now the Regional Trial Court) a petition for review
within one year after entry of the decree, provided no innocent purchaser for value
"SECTION 50. . . . But no deed, mortgage, has acquired an interest." The same law provides that upon the expiration of the term
lease, or other voluntary instrument except a will, of one year, "every decree or certificate of title . . . shall be imprescriptible."
purporting to convey or affect registered land, shall take
effect as a conveyance or bind the land, but shall operate Under the law, an action for reconveyance of real property resulting from
only as a contract between the parties and as evidence fraud prescribes in four (4) years from the discovery of the fraud. 16 Discovery of the
of authority to the clerk or register of deeds to make fraud must be deemed to have taken place when Lucia Bautista was issued OCT
registration. The act of registration shall be the operative Nos. 178111 and 17812 because registration of real property is considered a
act to convey and affect the land, and in all cases under "constructive notice to all persons" and it shall be counted "from the time of such
this Act the registration shall be made in the office of the registering, filing or entering." 17 An action based on implied or constructive trust
register of deeds for the province or provinces or city, prescribes in ten (10) years. This means that petitioners should have enforced the
where the land lies. [Underscoring supplied]. trust within ten (10) years from the time of its creation 18or upon the alleged
fraudulent registration of the property. But as it is, petitioners failed to avail of any of
SECTION 51. Every conveyance, the aforementioned remedies within the prescribed periods. With no remedy in view,
mortgage, lease, lien, attachment, order, decree, their claims should forever be foreclosed.
instrument, or entry affecting registered land which would
under existing laws, if recorded, filed, or entered in the The Court, however, subscribes to petitioners' argument that the courts a
office of the register of deeds, affect the real estate to quo incorrectly held that private respondents are third persons to whom ownership of
which it relates shall, if registered, filed, or entered in the the properties had been transmitted. But this error alone may not save the day for
office of the register of deeds in the province or city petitioners. They have, in a sense, slept on whatever rights they claimed to have over
where the real estate to which such instrument relates the properties and by the time they were roused, the law had stepped in to bar their
lies, be notice to all person from the time of such claims. On the other hand, private respondents' inattention to the property from the
registering, filing, or entering." time of Lucia Bautista's death until private respondent Rodolfo Bautista's retirement
from the military should not be construed as an abandonment thereof. Private
Hence, petitioners' failure to register the Escritura de Compra Venta resulted respondents have in their favor the law that protects holders of title under the Torrens
in the sale being binding only between them and the vendor, Pedro Tolentino. System of land registration. As this Court so eloquently pronounced in 1915:
Lucia Bautista and her successors-in-interest, being third parties to the sale,
could not have been bound thereby. "Once a title is registered, the owner may
To give a semblance of ownership over the properties, petitioners rest secure, without the necessity of waiting in the portals
introduced in evidence documents showing that their successors-in-interest of the court, or sitting in the 'mirador de su casa,' to avoid
mortgaged the properties. While only owners of properties have the right to mortgage the possibility of losing his land." 19
the same, the papers evidencing the alleged mortgages do not, however, conform to
WHEREFORE, the instant petition for review on certiorari is hereby
the formal and substantive requirements therefor. One such document 10 dated May
DENIED for lack of merit. The decision and the resolution appealed from in CA-G.R.
24, 1987 and handwritten in the English language described the property allegedly
CV No. 30033 dated October 23, 1992 and July 1, 1993, respectively, are
mortgaged to a certain Santos Tolentino as "a certain parcel of land estimated at one
AFFIRMED.
hectare." The other alleged mortgage instrument dated August 12, 1985, 11 likewise
handwritten but in the Ilocano dialect, did not sufficiently describe the subject property Costs against petitioners.
of the mortgage. There is indeed no way that we can ever determine if the lands
referred to in the mortgage were the lots now in controversy. At any rate, while SO ORDERED.
petitioners' daughter, Erlinda Ramos, testified that the properties in controversy were
the ones she and her sisters mortgaged, that claim is now self-serving since they are
presently the claimants of the lands. 12 Interestingly, Erlinda herself admitted that her
FIRST DIVISION Decree No. 694438 issued on February 27, 1934) in the name of
[G.R. No. L-46345. January 30, 1990.] "Vicente Dabon married to Marcela [or Marcelina] Ceniza." (pp. 7 and
19, Record on Appeal).
RESTITUTO CENIZA and JESUS
CENIZA, petitioners, vs. THE HON. COURT OF Petitioners are the descendants of Manuel Ceniza while
APPEALS, MAGNO DABON, VICENTA DABON, the private respondents are the descendants of his sister, Sofia
TERESITA DABON, EUGENIA DABON, and TOMAS Ceniza. Sofia Ceniza was childless but she had an adopted daughter
DABON, respondents. named Flaviana Ceniza, who begot a daughter named Marced Ceniza
and who in turn had a daughter named Marcelina (or Marcela) Ceniza
Vicente P. Valera and Pedro Rosito & Jesus F. who married Vicente Dabon. Private respondents are the children of
Balicanta for petitioners. this marriage and they are the great-great-grandchildren of Sofia
Victorino U. Montecillo for respondents. Ceniza. prcd
On the other hand, Manuel Ceniza had an only son, Pablo,
SYLLABUS who had two sons, Santiago and Jose Ceniza. Petitioners Restituto
1. CIVIL LAW; CO-OWNERSHIP; AS A GENERAL RULE; and Jesus Ceniza and a certain Nemesia Ceniza-Albina are their
PRESCRIPTION SHALL NOT RUN IN FAVOR OF A CO-OWNER; children and the great-grandchildren of Manuel Ceniza.
EXCEPTION. — Since a trust relation and co-ownership were proven The records disclose that when Hacienda de Mandaue was
to exist between the predecessors-in-interest of both petitioners and subdivided for resale to the occupants in 1929, Jose Ceniza and
private respondents, prescription did not run in favor of Dabon's heirs Vicente Dabon, who were residing in the hacienda, jointly purchased
except from the time that they repudiated the co-ownership and made Lot 627 on installment basis and they agreed, for convenience, to have
the repudiation known to the other co-owners, Restituto and Jesus the land registered in the name of Dabon. Since then, Jose Ceniza,
Ceniza (Cortes vs. Oliva, 33 Phil. 480). Paragraph 5 of Article 494 of Vicente Dabon, and their heirs have possessed their respective
the Civil Code provides — "No prescription shall run in favor of a co- portions of the land, declared the same for taxation, paid real estate
owner or co-heir against his co-owners or co-heirs so long as he taxes on their respective shares, and made their respective
expressly or impliedly recognizes the co-ownership." In Custodio v. installment payments to the Seminario de San Carlos de Cebu.
Casiano, 9 SCRA 841, we ruled that: "Where title to land was issued
in the name of a co-heir merely with the understanding that he would After Dabon died in 1954, his seven (7) children, named
act as a trustee of his sisters, and there is no evidence that this trust Magno, Jacinta, Tomas, Flaviana, Soledad, Teresita and Eugenia,
relation had ever been repudiated by said trustee, it is held that a succeeded to his possession of a portion of the land.
relation of co-ownership existed between such trustee and his sisters On November 4, 1961, a private land surveyor, Espiritu
and the right of the successors-in-interest of said sisters to bring the Bunagan, on the request of Jacinta Dabon and Restituto Ceniza who
present action for recovery of their shares therein against the jointly defrayed the cost, divided Lot 627 into three parts, namely:
successors-in-interest of said trustee cannot be barred by
prescription, despite the lapse of 25 years from the date of registration (1) Lot No. 627-A with 3,538 square meters for Marcela
of the land in the trustee's name." Ceniza;
2. ID.; IMPLIED TRUST; ESTABLISHED IN THE CASE AT (2) Lot No. 627-B with 884 square meters for Restituto
BAR. — The registration of Lot No. 627 in the name of Vicente Dabon Ceniza; and
created a trust in favor of his co-owner Jose Ceniza, and the latter's
(3) Lot No. 627-C with 834 square meters for Nemesia
heirs. Article 1452 of the Civil Code states: "If two or more persons
Ceniza-Albina, who later bequeathed her share to her brother, Jesus
agree to purchase property and by common consent the legal title is
Ceniza. (p. 19, Record on Appeal).
taken in the name of one of them for the benefit of all, a trust is created
by force of law in favor of the others in proportion to the interest of The present controversy arose because the private
each." respondents refused to convey Lots Nos. 627-B and 627-C to the
petitioners. They claimed that their predecessor-in-interest, Vicente
3. ID.; ID.; AS A GENERAL RULE, TRUSTEE'S
Dabon, was the sole and exclusive owner of Lot 627, by purchase from
POSSESSION IS NOT ADVERSE AND CANNOT RIPEN INTO TITLE
the Seminario de San Carlos de Cebu. In their answer to the
BY PRESCRIPTION; ELEMENTS FOR TRUSTEE'S ADVERSE
petitioners' complaint for reconveyance in June 1967, they alleged that
POSSESSION. — This Court has ruled in numerous cases involving
the petitioners' right of action had already prescribed.
fiduciary relations that, as a general rule, the trustee's possession is
not adverse and therefore cannot ripen into a title by prescription. Petitioners replied that Vicente Dabon held the land in trust
Adverse possession requires the concurrence of the following for them, as co-owners, hence, their action for reconveyance was
circumstances: a) that the trustee has performed unequivocal acts of imprescriptible.
repudiation amounting to the ouster of the cestui que trust; b) that such
positive acts of repudiation have been made known to the cestui que On August 31, 1970, the trial court rendered judgment for
trust; and c) that the evidence thereon should be clear and conclusive. the petitioners. Finding that there existed a co-ownership among the
parties, it ordered the private respondents to execute deeds of
DECISION conveyance of Lots Nos. 627-B and 627-C in favor of the plaintiffs,
Restituto and Jesus Ceniza, respectively (p. 35, Record on
GRIÑO-AQUINO, J p: Appeal). llcd
This is a petition for review of the order dated October 29, On appeal by the defendants (now private respondents)
1976, of the Court of Appeals in CA-G.R. No. 48546 entitled, the Court of Appeals on October 29, 1976, reversed that decision of
"Restituto Ceniza, et al. vs.Magno Dabon, et al.," dismissing the the trial court. It ruled that the petitioners' right of action had prescribed
petitioners' complaint for reconveyance of their shares in co- after the lapse of 20 years from the date of registration of the land on
ownership property and reversing the decision of the trial court in their February 8, 1939 in Vicente Dabon's name (p. 32, Rollo).
favor.
The petitioners have appealed to this Court by a petition for
On June 14, 1967, the petitioners filed against private review under Rule 45 of the Rules of Court.
respondents, an action in the Court of First Instance of Cebu for
recovery of their title to Lots Nos. 627-B and 627-C (being portions of The legal issue presented by the petition is whether the
Lot No. 627 with an area of approximately 5,306 square meters) registration of the title of the land in the name of one of the co-owners
situated in Casuntingan, Mandaue, Cebu (now Mandaue City), which constituted a repudiation of the co-ownership for purposes of
originally formed part of "Hacienda de Mandaue" of the Seminario de acquisitive prescription.
San Carlos de Cebu. The property is covered by reconstituted Original We find merit in the petition for review.
Certificate of Title No. RO-10996 issued on February 8, 1939 (formerly
The trial court correctly ruled that since a trust relation and WHEREFORE, the decision of the Court of Appeals is
co-ownership were proven to exist between the predecessors-in- hereby REVERSED AND SET ASIDE and the decision dated August
interest of both petitioners and private respondents, prescription did 31, 1970 of the then Court of First Instance of Cebu, Branch VI, in Civil
not run in favor of Dabon's heirs except from the time that they Case No. R-10030 is reinstated. Costs against the private
repudiated the co-ownership and made the repudiation known to the respondents. cdrep
other co-owners, Restituto and Jesus Ceniza (Cortes vs. Oliva, 33
Phil. 480). SO ORDERED.

Paragraph 5 of Article 494 of the Civil Code provides — ||| (Ceniza v. Court of Appeals, G.R. No. L-46345, [January 30, 1990], 260
PHIL 600-606)
"No prescription shall run in favor of a co-owner or
co-heir against his co-owners or co-heirs so long
as he expressly or impliedly recognizes the co-
ownership."
The registration of Lot No. 627 in the name of Vicente
Dabon created a trust in favor of his co-owner Jose Ceniza, and the
latter's heirs. Article 1452 of the Civil Code states:
"If two or more persons agree to purchase property
and by common consent the legal title is taken in
the name of one of them for the benefit of all, a
trust is created by force of law in favor of the others
in proportion to the interest of each."
This Court has ruled in numerous cases involving fiduciary
relations that, as a general rule, the trustee's possession is not
adverse and therefore cannot ripen into a title by prescription. Adverse
possession requires the concurrence of the following circumstances:
a) that the trustee has performed unequivocal acts of
repudiation amounting to the ouster of the cestui que trust;
b) that such positive acts of repudiation have been made
known to the cestui que trust; and
c) that the evidence thereon should be clear and
conclusive. LexLib
The above elements are not present here for the petitioners
co-owners have not been ousted from the land. They continue to
possess their respective shares of Lot 627 and they have been paying
the realty taxes thereon. Restituto's house stands on his portion of the
Land. Assuming that the private respondents' rejection of the
subdivision plan for the partition of the land was an act of repudiation
of the co-ownership, prescription had not yet set in when the
petitioners instituted the present action for reconveyance. These
circumstances were overlooked by the Court of Appeals.
In Custodio v. Casiano, 9 SCRA 841, we ruled that:
"Where title to land was issued in the name of a
co-heir merely with the understanding that he
would act as a trustee of his sisters, and there is no
evidence that this trust relation had ever been
repudiated by said trustee, it is held that a relation
of co-ownership existed between such trustee and
his sisters and the right of the successors-in-
interest of said sisters to bring the present action
for recovery of their shares therein against the
successors-in-interest of said trustee cannot be
barred by prescription, despite the lapse of 25
years from the date of registration of the land in the
trustee's name." (Emphasis supplied.)

In Escobar v. Locsin, 74 Phil. 86, we affirmed the duty of


the courts to shield fiduciary relations "against every manner of
chicanery or detestable design cloaked by legal technicalities" and to
guard against misuse of the Torrens system "to foment betrayal in the
performance of a trust."
In this case, since the statutory period of limitation within
which to file an action for reconveyance, after the defendants had
repudiated the co-ownership in 1961, had not yet run its course when
the petitioners filed said action in 1967, the action was not barred by
prescription.
THIRD DIVISION not, however, proffered any reason warranting the disturbance of the trial
court's findings of facts.
[G.R. No. 161720. November 22, 2005.]
6. CIVIL LAW; MODES OF ACQUIRING OWNERSHIP;
HEIRS OF FLORES RESTAR namely: ACQUISITIVE PRESCRIPTION; ADVERSE POSSESSION;
ESMENIA R. RESTAR, BERNARDITA R. ESTABLISHED IN CASE AT BAR. — Indeed, the following acts of Flores
RENTINO, LUCIA RESTAR, RODOLFO show possession adverse to his co-heirs: the cancellation of the tax
RESTAR, JANET R. RELOJERO, LORNA R. declaration certificate in the name of Restar and securing another in his
RAMOS, MANUEL RESTAR, NENITA R. name; the execution of a Joint Affidavit stating that he is the owner and
BELLEZA, MIRASOL R. DELA CRUZ, possessor thereof to the exclusion of respondents; payment of real estate
ROSELLE R. MATORRE, POLICARPIO tax and irrigation fees without respondents having ever contributed any
RESTAR and ADOLFO share therein; and continued enjoyment of the property and its produce to
RESTAR, petitioners, vs. HEIRS OF DOLORES the exclusion of respondents. And Flores' adverse possession was
R. CICHON, namely: RUDY R. CICHON, continued by his heirs.
NORMA C. LACHICA, NILDA C. JUMAYAO,
LYDIA C. SANTOS, and NELSON R. CICHON; DECISION
HEIRS OF PERPETUA R. STA. MARIA, namely
GEORGE STA. MARIA, LILIA M. MANIAGO, CARPIO MORALES, J p:
DERLY M. CONCEPCION, GERVY STA.
MARIA, DORY M. INDULO; HEIRS OF MARIA In 1935, Emilio Restar (Restar) died intestate, leaving eight (8)
R. ROSE, namely: TERESITA R. MALOCO, children-compulsory heirs, namely: Flores Restar, Dolores Restar-Cichon,
ROLANDO ROSE, EDELYN R. PALACIO and Perpetua Restar-Sta. Maria, Paciencia Restar-Manares, Dominica Restar-
MINERVA R. PASTRANA, DOMINICA RESTAR- Relojero, Policarpio Restar, Maria Restar-Rose and Adolfo Restar.
RELOJERO and PACIENCIA RESTAR In 1960, Restar's eldest child, Flores, on the basis of a July 12,
MANARES, respondents. 1959 Joint Affidavit 1 he executed with one Helen Restar, caused the
cancellation of Tax Declaration No. 6696 2 in Restar's name covering a
Singson Valdez & Associates for petitioners. 5,918 3 square meter parcel of land, Lot 3177 (the lot), located at Barangay
Diomedes T. Resurreccion for respondents. Carugdog, Lezo, Aklan which was among the properties left by Restar, and
the issuance of Tax Declaration No. 11134 in his name.
SYLLABUS
Flores died on June 10, 1989.
1. CIVIL LAW; PROPERTY; CO-OWNERSHIP; WHILE THE On November 5, 1998, the co-heirs of Flores discovered the
ACTION TO DEMAND PARTITION OF A CO-OWNED PROPERTY DOES cancellation of Restar's Tax Declaration No. 6696 and the issuance in lieu
NOT PRESCRIBE, A CO-OWNER MAY ACQUIRE OWNERSHIP thereof of Tax Declaration No. 11134 4 in his name.
THEREOF BY PRESCRIPTION. — While the action to demand partition of
a co-owned property does not prescribe, a co-owner may acquire ownership On January 21, 1999, the heirs of Flores' sisters Dolores R.
thereof by prescription where there exists a clear repudiation of the co- Cichon, Perpetua Sta. Maria, and Maria Rose who had in the meantime
ownership, and the co-owners are apprised of the claim of adverse and died, together with Flores' surviving sisters Dominica Restar-Relojero and
exclusive ownership. Paciencia Restar-Manares, filed a Complaint 5 against Flores' heirs for
2. ID.; MODES OF ACQUIRING OWNERSHIP; ACQUISITIVE "partition [of the lot], declaration of nullity of documents, ownership with
PRESCRIPTION; ORDINARY AND EXTRAORDINARY ACQUISITIVE damages and preliminary injunction" before the Regional Trial Court (RTC)
PRESCRIPTION, DISTINGUISHED. — Acquisitive prescription of dominion of Aklan.
and other real rights may be ordinary or extraordinary. Ordinary acquisitive
prescription requires possession of things in good faith and with just title for Flores' brothers Policarpio and Adolfo were impleaded also as
a period of ten years. Without good faith and just title, acquisitive defendants, they being unwilling co-plaintiffs.
prescription can only be extraordinary in character which requires The plaintiffs, herein respondents, alleged that, inter alia, during
uninterrupted adverse possession for thirty years. the lifetime of Flores, they were given their shares of palay from the lot and
3. REMEDIAL LAW; CIVIL PROCEDURE; APPEALS; even after Flores death up to 1991; after Flores' death in 1989, his widow
PETITION FOR REVIEW ON CERTIORARI; THE SUPREME COURT MAY Esmenia appealed to them to allow her to hold on to the lot to finance the
REVIEW THE EVIDENCE ON RECORDS IF THE INFERENCE DRAWN education of her children, to which they (the plaintiffs) agreed on the
BY THE APPELLATE COURT FROM THE FACTS IS MANIFESTLY condition that after the children had finished their education, it would be
MISTAKEN; CASE AT BAR. — While this Court is not a trier of facts, if the divided into eight (8) equal parts; and upon their demand for partition of the
inference drawn by the appellate court from the facts is manifestly mistaken, lot, the defendants Heirs of Flores refused, they claiming that they were the
it may, in the interest of justice, review the evidence in order to arrive at the lawful owners thereof as they had inherited it from Flores. SaITHC
correct factual conclusions based on the record. Contrary to the findings of By Answer 6 filed February 23, 1999, the defendants-herein
the appellate court, the records of the case amply support petitioners' claim petitioners Heirs of Flores claimed that they had been in possession of the
that the requirements for extraordinary prescription had been duly met. lot in the concept of owner for more than thirty (30) years and have been
4. CIVIL LAW; PROPERTY: OWNERSHIP; TAX paying realty taxes since time immemorial. And they denied having shared
DECLARATIONS AND RECEIPTS ARE NOT CONCLUSIVE EVIDENCE with the plaintiffs the produce of the lot or that upon Flores' death in 1989,
OF OWNERSHIP, BUT WHEN COUPLED WITH ACTUAL POSSESSION, Esmenia requested the plaintiffs to allow her to hold on to it to finance her
THEY CONSTITUTE EVIDENCE OF GREAT WEIGHT AND CAN BE THE children's education, they contending that by 1977, the children had already
BASIS OF CLAIM OF OWNERSHIP THROUGH PRESCRIPTION. — While finished their respective courses. 7
tax declarations and receipts are not conclusive evidence of ownership and The defendants Heirs of Flores further claimed that after World
do not prove title to the land, nevertheless, when coupled with actual War II and under the "new Tax Declaration in 1945," Flores caused the
possession, they constitute evidence of great weight and can be the basis transfer of parcels of ricelands situated in Carugdog, Lezo, Aklan to his
of a claim of ownership through prescription. siblings as their shares from the estate of their father Restar; 8 and an extra-
5. REMEDIAL LAW; EVIDENCE; CREDIBILITY; FACTUAL judicial partition was subsequently executed on September 28, 1973 by
FINDINGS OF THE TRIAL COURT, GENERALLY NOT DISTURBED ON Restar's heirs, which was notarized by one Atty. Jose Igtanloc, dividing and
APPEAL. — Unless there are strong and impelling reasons to disturb the apportioning among themselves four (4) parcels of land. 9
trial court's findings of facts which must, as a matter of judicial policy, be The defendant Adolfo Restar, by separate Answer, 10 alleged
accorded with the highest respect, they must remain. Respondents have that the complaint did not state a cause of action as against him for he
interposed no objection to the partition of the lot among the heirs of Restar.
As for the defendant Policarpio Restar, he in his Amended B. THE COURT OF APPEALS PATENTLY
Answer 11 acknowledged Flores as the owner of the lot but claimed that a ERRED IN NOT RULING THAT
portion of it, 1,315 square meters, was sold to him as shown by a Deed of THERE WAS ACQUISITIVE
Absolute Sale dated May 14, 1981. 12 He thus prayed that, among other PRESCRIPTION ON THE LAND IN
things, an order for the partition of the lot among Restar's heirs be issued QUESTION NOTWITHSTANDING
excluding, however, that portion sold to him by Flores. 13 THAT THE LAND IN QUESTION HAS
BEEN DECLARED IN THE NAME OF
After trial, Branch 3 of the RTC of Kalibo, Aklan held that Flores' FLORES RESTAR, FATHER OF
share in Restar's estate was not the lot but that covered by Cadastral Lot PETITIONERS, AS EARLY AS 1960
No. 3183. Nevertheless, the trial court, holding that Flores and his heirs had AND THAT PETITIONERS AND
performed acts sufficient to constitute repudiation of the co-ownership, THEIR PREDECESSOR-IN-
concluded that they had acquired the lot by prescription. 14 INTEREST HAVE BEEN IN OPEN,
CONTINUOUS, EXCLUSIVE AND
Respecting the defendant Policarpio's claim that a portion of the
NOTORIOUS POSSESSION OF THE
lot was sold to him, the trial court discredited the same upon noting that
LAND IN QUESTION IN THE
Flores' signature in the purported Deed of Sale differed from those
CONCEPT OF OWNER FOR MORE
appearing in other documents submitted by the parties; in 1981, when the
THAN THIRTY (30) YEARS. 20
said Deed of Sale was alleged to have been executed, Flores was
admittedly paralyzed and bedridden and could not have written his name in The petition is impressed with merit.
a "straight" manner, as in fact his signature appearing in at least two
documents dated 1980 was "crooked," and there existed discrepancies in Article 494 of the New Civil Code expressly provides:
the spelling of Flores' wife's signature which read "Esmeña" in the deed,
and not as "Esmenia." 15 ART. 494. No co-owner shall be
obliged to remain in the co-ownership. Each co-
The trial court thus dismissed the complaint by Decision of June owner may demand at any time the partition of the
30, 1999. 16 thing owned in common, insofar as his share is
concerned.
On appeal by the defendants Heirs of Flores and Policarpio
Restar, the appellate court, by Decision of October 29, 2002. 17 reversed xxx xxx xxx
the decision of the trial court, it finding that the defendants Heirs of Flores
failed to prove that their possession of the lot excluded their co-owners or No prescription shall run in favor of a
that they derived title to it from a separate conveyance to them by Restar. co-owner or co-heir against his co-owners or co-
heirs so long as he expressly or impliedly
The appellate court further found that there was no adequate recognizes the co-ownership.
notice by Flores to his other co-heirs/co-owners of the repudiation of the co-
ownership and neither was there a categorical assertion by the defendants While the action to demand partition of a co-owned property
of their exclusive right to the entire lot that barred the plaintiffs' claim of does not prescribe, a co-owner may acquire ownership thereof by
ownership. 18 prescription 21 where there exists a clear repudiation of the co-ownership,
and the co-owners are apprised of the claim of adverse and exclusive
And the appellate court found it credible for the plaintiffs to have ownership. 22
failed to immediately take legal action to protect their rights on account of
forbearance towards their eldest brother who had asked them to continue Acquisitive prescription of dominion and other real rights may be
cultivating the lot to support his children's education. 19 ordinary or extraordinary. Ordinary acquisitive prescription requires
possession of things in good faith and with just title for a period of ten years.
Respecting the defendant Policarpio's claim that part of the lot Without good faith and just title, acquisitive prescription can only be
had been sold to him by Flores, the appellate court sustained the trial court's extraordinary in character which requires uninterrupted adverse possession
rejection thereof. for thirty years.
Accordingly, the appellate court disposed: Thus, the New Civil Code provides:

WHEREFORE, in view of all the ART. 1117. Acquisitive prescription of


foregoing, the appeal is hereby GRANTED in so dominion and other real rights may be ordinary or
far as plaintiffs-appellants Heirs of Dolores Cichon, extraordinary.
et al., are concerned and DENIED in so far as
defendant-appellant Policarpio Restar. The Ordinary acquisitive prescription
decision of the Regional Trial Court of Kalibo, requires possession of things in good faith and
Aklan, Branch 3, dated June 30, 1999 with just title for the time fixed by law.
isMODIFIED. The ruling of the said court that the
ART. 1134. Ownership and other real
heirs of Flores Restar have acquired ownership by
rights over immovable property are acquired by
adverse possession of the land in question,
ordinary prescription through possession of ten
Cadastral Lot No. 6686, is hereby REVERSED.
years.
SO ORDERED. (Emphasis in the
ART. 1137. Ownership and other real
original)
rights over immovables also prescribe through
The appellate court having denied reconsideration of its uninterrupted adverse possession thereof for thirty
decision, only the defendants Heirs of Flores filed the present petition, years, without need of title or of good faith.
assigning the following errors:

A. THE COURT OF APPEALS PATENTLY


Resolving the main issue of whether petitioners acquired
ERRED IN REVERSING THE RULING
ownership over the lot by extraordinary prescription, the appellate court held
OF THE LOWER COURT THAT THE in the negative.
PETITIONERS AS HEIRS OF
FLORES RESTAR HAVE ACQUIRED While this Court is not a trier of facts, if the inference drawn by
OWNERSHIP BY ADVERSE the appellate court from the facts is manifestly mistaken, it may, in the
POSSESSION OF THE LAND IN interest of justice, review the evidence in order to arrive at the correct factual
QUESTION. TADaES conclusions based on the record. 23
Contrary to the findings of the appellate court, the records of the While tax declarations and receipts are not conclusive evidence
case amply support petitioners' claim that the requirements for extraordinary of ownership and do not prove title to the land, nevertheless, when coupled
prescription had been duly met. with actual possession, they constitute evidence of great weight 26 and can
be the basis of a claim of ownership through prescription. 27
When Restar died in 1935, his eight children became pro
indiviso co-owners of the lot by intestate succession. Respondents never As for respondents' claim that they have been receiving shares
possessed the lot, however, much less asserted their claim thereto until from the produce of the land, it was correctly discredited by the trial court.
January 21, 1999 when they filed the complaint for partition subject of the
present petition. TDcEaH [P]laintiffs' claim that Flores Restar
gave them five to eight gantas each as their shares
In contrast, Flores took possession of the lot after Restar's death in the produce cannot be sustained. A few gantas
and exercised acts of dominion thereon — tilling and cultivating the land, cannot be considered one-eight share of sixty (60)
introducing improvements, and enjoying the produce thereof. cavans of palay produced per cropping. One eight
of sixty cavans would be at least six cavans, not
The statutory period of prescription, however, commenced not in merely gantas after excluding expenses for
1935 but in 1960 when Flores, who had neither title nor good faith, secured cultivation and production. If plaintiffs were to be
a tax declaration in his name and may, therefore, be said to have adversely believed, their whole 7/8 share of the produce
claimed ownership of the lot. And respondents were also deemed to have would total two cavans, six gantas only at the usual
been on said date become aware of the adverse claim. 24 rate of 25 gantas per cavan. 28
Flores' possession thus ripened into ownership through Unless there are strong and impelling reasons to disturb the trial
acquisitive prescription after the lapse of thirty years in accordance with the court's findings of facts which must, as a matter of judicial policy, be
earlier quoted Article 1137 of the New Civil Code. accorded with the highest respect, they must remain. Respondents have
not, however, proffered any reason warranting the disturbance of the trial
The following observations of the trial court thus merit this
court's findings of facts. cCaEDA
Court's approval.
Indeed, the following acts of Flores show possession adverse to
The evidence proved that as far back
his co-heirs: the cancellation of the tax declaration certificate in the name of
as 1959, Flores Restar adjudicated unto himself
Restar and securing another in his name; the execution of a Joint Affidavit
the whole land in question as his share from his
stating that he is the owner and possessor thereof to the exclusion of
father by means of a joint affidavit which he
respondents; payment of real estate tax and irrigation fees without
executed with one Helen Restar, and he requested
respondents having ever contributed any share therein; and continued
the Provincial Treasurer/Assessor to have the land
enjoyment of the property and its produce to the exclusion of respondents.
declared in his name. It was admitted by the
And Flores' adverse possession was continued by his heirs.
parties during the pre-trial that this affidavit was the
basis of the transfer of Tax Declaration No. 6686 The appellate court's crediting of respondents' justification for
from Emilio Restar to Flores Restar. So that from failing to immediately take legal action to protect their rights — forbearance
1960 the land was declared in the name of Flores toward Flores and/or his wife who asked to be allowed to cultivate the land
Restar (Exhibit 10). This was the first concrete act to support their children's education — does not impress. For assuming
of repudiation made by Flores of the co-ownership such justification to be true, why did not any of respondents assail Flores'
over the land in question. . . . continuous possession after his children completed their college education
in 1977?
Plaintiffs did not deny that aside from
the verbal partition of one parcel of land in The trial court's finding and conclusion that Flores and his heirs
Carugdog, Lezo, Aklan way back in 1945, they had for more than 38 years possessed the land in open, adverse and
also had an amicable partition of the lands of continuous possession in the concept of owner — which length of
Emilio Restar in Cerrudo and Palale, Banga Aklan possession had never been questioned, rebutted or disputed by any of
on September 28, 1973 (exhibit "20"). If they were respondents, being thus duly supported by substantial evidence, he and his
able to demand the partition, why then did they not heirs have become owner of the lot by extraordinary prescription. It is
demand the inclusion of the land in question in unfortunate that respondents slept on their rights. Dura lex sed lex.
order to settle once and for all the inheritance from
their father Emilio Restar, considering that at that WHEREFORE, the petition is GRANTED. The decision of the
time all of the brothers and sisters, the eight heirs Court of Appeals is REVERSED and SET ASIDE and the June 30, 1999
of Emilio Restar, were still alive and participated in decision of the trial court is REINSTATED.
the signing of the extra-judicial partition?
No pronouncement as to costs.
Also it was admitted that Flores died
only in 1989. Plaintiffs had all the chances (sic) to SO ORDERED.
file a case against him from 1960, or a period of 29
Panganiban, Corona and Garcia, JJ., concur.
years when he was still alive, yet they failed to do
so. They filed the instant case only on January 22, Sandoval-Gutierrez, J., is on leave.
1999, almost ten (10) years after Flores' death.
||| (Heirs of Restar v. Heirs of Cichon, G.R. No. 161720, [November 22,
From the foregoing evidence, it can be 2005], 512 PHIL 377-389)
seen that the adverse possession of Flores started
in 1960, the time when the tax declaration was
transferred in his name. The period of acquisitive
prescription started to run from this date. Hence,
the adverse possession of Flores Restar from
1960 vested in him exclusive ownership of the land
considering the lapse of more than 38 years.
Acquisitive prescription of ownership, laches and
prescription of the action for partition should be
considered in favor of Flores Restar and his
heirs. 25
FIRST DIVISION DECISION
HERMOSISIMA, JR., J p:
[G.R. No. 68166. February 12, 1997.] Unique is the legal question visited upon the claim of an applicant in a
Land Registration case by oppositors thereto, the Government and a Government
HEIRS OF EMILIANO NAVARRO, petitioner, vs. lessee, involving as it does ownership of land formed by alluvium.
INTERMEDIATE APPELLATE COURT AND HEIRS
OF SINFOROSO PASCUAL, respondents. The applicant owns the property immediately adjoining the land sought
to be registered. His registered property is bounded on the east by the Talisay River,
Yolanda Quisumbing - Javellana & Associates for petitioner. on the west by the Bulacan River, and on the north by the Manila Bay. The Talisay
Joracio R. Viola, Sr. for private respondents. River and the Bulacan River flow down towards the Manila Bay and act as boundaries
of the applicant's registered land on the east and on the west.
SYLLABUS
1. CIVIL LAW; PROPERTY; ACCRETION AS A MODE OF ACQUIRING The land sought to be registered was formed at the northern tip of the
PROPERTY; REQUISITES; LEGAL CONSEQUENCES. — Accretion as a mode of applicant's land. Applicant's registered property is bounded on the north by the Manila
acquiring property under Article 457 of the Civil Code, requires the concurrence of Bay.
the following requisites: (1) that the accumulation of soil or sediment be gradual and
The issue: May the land sought to be registered be deemed an accretion
imperceptible; (2) that it be the result of the action of the waters of the river: and (3)
in the sense that it naturally accrues in favor of the riparian owner or should the land
that the land where the accretion takes place is adjacent to the bank of the river. be considered as foreshore land?
Accretion is the process whereby the soil is deposited, while alluvium is the soil
deposited on the estate fronting the river bank; the owner of such estate is called the Before us is a petition for review of: (1) the decision 1 and (2) two
riparian owner. Riparian owners are, strictly speaking, distinct from littoral owners, subsequent resolutions 2 of the Intermediate Appellate Court 3 (now the Court of
the latter being owners of lands bordering the shore of the sea or lake or other tidal Appeals) in Land Registration Case No. N-84, 4 the application over which was filed
waters. The alluvium, by mandate of Article 457 of the Civil Code, is automatically by private respondents' predecessor-in-interest, Sinforoso Pascual, now deceased,
owned by the riparian owner from the moment the soil deposit can be seen hut is not before the Court of First Instance 5 (now the Regional Trial Court) of Balanga,
automatically registered property, hence, subject to acquisition through prescription Bataan.
by third persons.
There is no dispute as to the following facts:
2. ID.; ID.; ID.; THIRD REQUISITE NOT PRESENT IN CASE AT BAR.
— There is no dispute as to the location of: (a) the disputed land; (b) petitioners' own On October 3, 1946, Sinforoso Pascual, now deceased, filed an
tract of land: (c) the Manila Bay; and, (d) the Talisay and Bulacan Rivers. Petitioners' application for foreshore lease covering a tract of foreshore land in Sibocon, Balanga,
own land lies between the Talisay and Bulacan Rivers; in front of their land on the Bataan, having an area of approximately seventeen (17) hectares. This application
northern side lies now the disputed land where before 1948, there lay the Manila Bay. was denied on January 15, 1953. So was his motion for reconsideration.
If the accretion were to be attributed to the action of either or both of the Talisay and
Bulacan Rivers, the alluvium should have been deposited on either or both of the Subsequently, petitioners' predecessor-in-interest, also now deceased,
eastern and western boundaries of petitioners' own tract of land, not on the northern Emiliano Navarro, filed a fishpond application with the Bureau of Fisheries covering
portion thereof which is adjacent to the Manila Bay. Clearly lacking, thus, is the third twenty five (25) hectares of foreshore land also in Sibocon, Balanga, Bataan. Initially,
requisite of accretion, which is, that the alluvium is deposited on the portion of such application was denied by the Director of Fisheries on the ground that the
claimant's land which is adjacent to the river bank. property formed part of the public domain. Upon motion for reconsideration, the
Director of Fisheries, on May 27, 1988, gave due course to his application but only to
3. ID.; ID.; ID.; ID.; THE DISPUTED LAND IS AN ACCRETION NOT ON the extent of seven (7) hectares of the property as may be certified by the Bureau of
A RIVER BANK BUT ON A SEA BANK; THE APPLICABLE LAW IS NOT ARTICLE Forestry as suitable for fishpond purposes.
457 OF THE CIVIL CODE BUT ARTICLE 4 OF THE SPANISH LAW OF WATERS
OF 1866. — There is no dispute as to the fact that petitioners' own tract of land adjoins The Municipal Council of Balanga, Bataan, had opposed Emiliano
the Manila Bay. Manila Bay is obviously not a river, and jurisprudence is already Navarro's application. Aggrieved by the decision of the Director of Fisheries, it
settled as to what kind of body of water the Manila Bay is. It is to be remembered that appealed to the Secretary of Natural Resources who, however, affirmed the grant.
we held in Ignacio vs. Director of Lands and Valeriano (108 Phil. 336, 338 [1960]) The then Executive Secretary, acting in behalf of the President of the Philippines,
that: "Appellant next contends that . . . Manila Bay cannot be considered as a sea. similarly affirmed the grant.
We find said contention untenable. A bay is part of the sea, being a mere indentation
of the same: 'Bay, — An opening into the land where the water is shut in on all sides On the other hand, sometime in the early part of 1960, Sinforoso Pascual
except at the entrance; an inlet of the sea; an arm of the sea, distinct from a river, a filed an application to register and confirm his title to a parcel of land, situated in
bending or curbing of the shore of the sea or of a lake,' 7 C.J. 1013-1014." The Sibocon, Balanga, Bataan, described in Plan Psu-175181 and said to have an area
disputed land, thus, is an accretion not on a river bank but on a sea bank, or on what of 146,611 square meters. Pascual claimed that this land is an accretion to his
used to be the foreshore of Manila Bay which adjoined petitioners' own tract of land property, situated in Barrio Puerto Rivas, Balanga, Bataan, and covered by Original
on the northern side. As such, the applicable law is not Article 457 of the Civil Code Certificate of Title No. 6830. It is bounded on the eastern side by the Talisay River,
but Article 4 of the Spanish Law of Waters of 1866. on the western side by the Bulacan River, and on the northern side by the Manila
Bay. The Talisay River as well as the Bulacan River flow downstream and meet at
4. ID.; ID.; ID.; THE DISPUTED PROPERTY IS AN ACCRETION ON A the Manila Bay thereby depositing sand and silt on Pascual's property resulting in an
SEA BANK, MANILA BAY BEING AN INLET OR AN ARM OF THE SEA; AS SUCH, accretion thereon. Sinforoso Pascual claimed the accretion as the riparian owner.
THE DISPUTED PROPERTY IS UNDER ARTICLE 4 OF THE SPANISH LAW OF
WATERS OF 1866, PART OF THE PUBLIC DOMAIN. — The instant controversy On March 25, 1960, the Director of Lands, represented by the Assistant
brings a situation calling for the application of Article 4 of the Spanish Law of Waters Solicitor General, filed an opposition thereto stating that neither Pascual nor his
of 1866, the disputed land being an accretion on the foreshore of Manila Bay which predecessors-in-interest possessed sufficient title to the subject property, the same
is, for all legal purposes, considered a sea. Article 4 of the Spanish Law of Waters of being a portion of the public domain and, therefore, it belongs to the Republic of the
August 3, 1866 provides as follows: "Lands added to the shores by accretions and Philippines. The Director of Forestry, through the Provincial Fiscal, similarly opposed
alluvial deposits caused by the action of the sea, form part of the public domain. When Pascual's application for the same reason as that advanced by the Director of Lands.
they are no longer washed by the waters of the sea and are not necessary for Later on, however, the Director of Lands withdrew his opposition. The Director of
purposes of public utility, or for the establishment of special industries, or for the Forestry become the sole oppositor.
coast-guard service, the Government shall declare them to be the property of the
On June 2, 1960, the court a quo issued an order of general default
owners of the estates adjacent thereto and as increment thereof." In the light of the
excepting the Director of Lands and the Director of Forestry. lexlib
aforecited vintage but still valid law, unequivocal is the public nature of the disputed
land in this controversy, the same being an accretion on a sea bank which, for all Upon motion of Emiliano Navarro, however, the order of general default
legal purposes, the foreshore of Manila Bay is. As part of the public domain, the was lifted and, on February 13, 1961, Navarro thereupon filed an opposition to
herein disputed land is intended for public uses, and "so long as the land in litigation Pascual's application. Navarro claimed that the land sought to be registered has
belongs to the national domain and is reserved for public uses, it is not capable of always been part of the public domain, it being a part of the foreshore of Manila Bay;
being appropriated by any private person, except through express authorization that he was a lessee and in possession of a part of the subject property by virtue of
granted in due form by a competent authority." Only the executive and possibly the a fishpond permit issued by the Bureau of Fisheries and confirmed by the Office of
legislative departments have the right and the power to make the declaration that the the President; and that he had already converted the area covered by the lease into
lands so gained by action of the sea is no longer necessary for purposes of public a fishpond.
utility or for the cause of establishment of special industries or for coast guard
services. Petitioners utterly fail to show that either the executive or legislative During the pendency of the land registration case, that is, on November
department has already declared the disputed land 1966, to be the property of 6, 1960, Sinforoso Pascual filed a complaint for ejectment against Emiliano Navarro,
petitioners as owners of the estates adjacent thereto. one Marcelo Lopez and their privies, alleged by Pascual to have unlawfully claimed
and possessed, through stealth, force and strategy, a portion of the subject property
covered by Plan Psu-175181. The defendants in the case were alleged to have built
a provisional dike thereon: thus they have thereby deprived Pascual of the premises This makes this case quite unique because
sought to be registered. This, notwithstanding repeated demands for defendants to while it is undisputed that the subject land is immediately
vacate the property. attached to appellants' [private respondents'] land and
forms the tip thereof, at the same time, said land
The case was decided adversely against Pascual. Thus, Pascual immediately faces the Manila Bay which is part of the
appealed to the Court of First Instance (now Regional Trial Court) of Balanga, Bataan, sea. We can understand therefore the confusion this
the appeal having been docketed as Civil Case No. 2873. Because of the similarity case might have caused the lower court, faced as it was
of the parties and the subject matter, the appealed case for ejectment was with the uneasy problem of deciding whether or not the
consolidated with the land registration case and was jointly tried by the court a quo. subject land was formed by the action of the two rivers or
by the action of the sea. Since the subject land is found
During the pendency of the trial of the consolidated cases, Emiliano at the shore of the Manila Bay facing appellants' [private
Navarro died on November 1, 1961 and was substituted by his heirs, the herein respondents'] land, it would be quite easy to conclude
petitioners. that it is foreshore and therefore part of the patrimonial
property of the State as the lower court did in fact rule . .
Subsequently, on August 26, 1962, Pascual died and was substituted by
..
his heirs, the herein private respondents.
xxx xxx xxx
On November 10, 1975, the court a quo rendered judgment finding the
subject property to be foreshore land and, being a part of the public domain, it cannot It is however undisputed that [private
be the subject of land registration proceedings. respondents'] land lies between these two rivers and it is
precisely appellants' [private respondents'] land which
The decision's dispositive portion reads:
acts as a barricade preventing these two rivers to meet.
"WHEREFORE, judgment is rendered:
Thus, since the flow of the two is downwards to the
(1) Dismissing plaintiff [private respondent] Sinforoso
Manila Bay the sediments of sand and silt are deposited
Pascual's complaint for ejectment in Civil
at their mouths.
Case No. 2873;
(2) Denying the application of Sinforoso Pascual for land It is, therefore, difficult to see how the Manila
registration over the land in question; and Bay could have been the cause of the deposit thereat for
(3) Directing said Sinforoso Pascual, through his heirs, in the natural course of things, the waves of the sea eat
as plaintiff in Civil Case No. 2873 and as the land on the shore, as they suge [sic] inland. It would
applicant in Land Registration Case No. N- not therefore add anything to the land but instead
84 to pay costs in both instances." 6 subtract from it due to the action of the waves and the
wind. It is then more logical to believe that the two rivers
The heirs of Pascual appealed and, before the respondent appellate flowing towards the bay emptied their cargo of sand, silt
court, assigned the following errors: and clay at their mouths, thus causing [private
respondents'] land to accumulate therein
"1. The lower court erred in not finding the land in
question as an accretion by the action of the However, our distinguished colleage [sic],
Talisay and Bulacan Rivers to the land Mr. Justice Serrano, do [sic] not seem to accept this
admittedly owned by applicants-appellants theory and stated that the subject land arose only when
[private respondents]. . . . Pascual planted 'palapat' and 'bakawan' trees thereat
2. The lower court erred in holding that the land in to serve as a boundary or strainer. But we do not see
question is foreshore land. how this act of planting trees by Pascual would explain
3.. The lower court erred in not ordering the registration how the land mass came into being. Much less will it
of the and is controversy in favor of prove that the same came from the sea. Following Mr.
applicants-appellants [private respondents]. Justice Serrano's argument that it were the few trees that
4. The lower court erred in not finding that the applicants- acted as strainers or blocks, then the land that grew
appellants [private respondents] are entitled would have stopped at the place where the said trees
to eject the oppositor-appellee were planted. But this is not so because the land mass
[petitioners]." 7 went far beyond the boundary, or where the trees were
planted.
On appeal, the respondent court reversed the findings of the court a
quo and granted the petition for registration of the subject property but excluding On the other hand, the picture-exhibits of
therefrom fifty (50) meters from corner 2 towards corner 1; and fifty meters (50) [private respondents'] clearly show that the land that
meters from corner 5 towards corner 6 of the Psu-175181. accumulated beyond the so-called boundary, as well as
the entire area being applied for is dry land, above sea
The respondent appellate court explained the reversal in this wise: level, and bearing innumerable trees . . . . The existence
of vegetation on the land could only confirm that the soil
"The paramount issue to be resolved in this thereat came from inland rather than from the sea, for
appeal as set forth by the parties in their respective briefs what could the sea bring to the shore but sand, pebbles,
is — whether or not the land sought to be registered is stones, rocks and corrals? On the other hand, the two
accretion or foreshore land, or, whether or not said land rivers would be bringing soil on their downward flow
was formed by the action of the two rivers of Talisay and which they brought along from the eroded mountains, the
Bulacan or by the action of the Manila Bay. If formed by lands along their path, and dumped them all on the
the action of the Talisay and Bulacan rivers, the subject northern portion of appellants' [private respondents']
land is accretion but if formed by the action of the Manila land.
Bay then it is foreshore land.
In view of the foregoing, we have to deviate
xxx xxx xxx from the lower court's finding. While it is true that the
subject land is found at the shore of the Manila Bay
It is undisputed that applicants-appellants
fronting appellants' [private respondents'] land, said land
[private respondents] owned the land immediately
is not foreshore but an accretion from the action of the
adjoining the land sought to be registered. Their property
Talisay and Bulacan rivers. In fact, this is exactly what
which is covered by OCT No. 6830 is bounded on the
the Bureau of Lands found out, as shown in the following
east by the Talisay River, on the west by the Bulacan
report of the Acting Provincial Officer, Jesus M. Orozco,
River, and on the north by the Manila Bay. The Talisay
to wit:
and Bulacan rivers come from inland flowing
downstream towards the Manila Bay. In other words, 'Upon ocular inspection of the
between the Talisay River and the Bulacan River is the land subject of this registration made on
property of applicants with both rivers acting as the June 11, 1960, it was found out that the said
boundary to said land and the flow of both rivers meeting land is . . . sandwitched [sic] by two big rivers
and emptying into the Manila Bay. The subject land was . . . These two rivers bring down
formed at the tip or apex of appellants' [private considerable amount of soil and sediments
respondents'] land adding thereto the land now sought to during floods every year thus raising the soil
be registered. of the land adjoining the private property of
the applicant [private respondents']. About motion for reconsideration shall be made ex-parte and filed within fifteen (15) days
four-fifth [sic] of the area applied for is now from the notice of the final order or judgment.
dry land whereon are planted palapat trees
thickly growing thereon. It is the natural Hence this petition where the respondent appellate court is imputed to
action of these two rivers that has caused have palpably erred in appreciating the facts of the case and to have gravely
the formation of said land . . . subject of this misapplied statutory and case law relating to accretion, specifically, Article 457 of the
registration case. It has been formed, Civil Code.
therefore, by accretion. And having been
formed by accretion, the said land may be We find no merit in the petition.
considered the private property of the
The disputed property was brought forth by both the withdrawal of the
riparian owner who is the applicant [private
waters of Manila Bay and the accretion formed on the exposed foreshore land by the
respondents'] . . . .
action of the sea which brought soil and sand sediments in turn trapped by the palapat
In view of the above, the and bakawan trees planted thereon by petitioner Sulpicio Pascual in 1948.
opposition hereto filed by the government
Anchoring their claim of ownership on Article 457 of the Civil Code,
should be withdrawn, except for the portion
petitioners vigorously argue that the disputed 14-hectare land is an accretion caused
recommended by the land investigator in
by the joint action of the Talisay and Bulacan Rivers which run their course on the
his report dated May 2, 1960, to be
eastern and western boundaries, respectively, of petitioners' own tract of land.
excluded and considered foreshore. . . .'
Accretion as a mode of acquiring property under said Article 457,
Because of this report, no less than the
requires the concurrence of the following requisites: (1) that the accumulation of soil
Solicitor General representing the Bureau of Lands
or sediment be gradual and imperceptible; (2) that it be the result of the action of the
withdrew his opposition dated March 25, 1960, and
waters of the river; and (3) that the land where the accretion takes place is adjacent
limited 'the same to the northern portion of the land
to the bank of the river. 11 Accretion is the process whereby the soil is deposited,
applied for, compromising a strip 50 meters wide along
while alluvium is the soil deposited on the estate fronting the river bank 12 ; the owner
the Manila Bay, which should be declared public land as
of such estate is called the riparian owner. Riparian owners are, strictly speaking,
part of the foreshore' . . . . 8
distinct from littoral owners, the latter being owners of lands bordering the shore of
Pursuant to the aforecited decision, the respondent appellate court ordered the sea or lake or other tidal waters. 13 The alluvium, by mandate of Article 457 of
the issuance of the corresponding decree of registration in the name of the Civil Code, is automatically owned by the riparian owner from the moment the soil
private respondents and the reversion to private respondents of the deposit can be seen 14 but is not automatically registered property, hence, subject
possession of the portion of the subject property included in Navarro's to acquisition through prescription by third persons. 15
fishpond permit.
Petitioners' claim of ownership over the disputed property under the
On December 20, 1978, petitioners filed a motion for reconsideration of principle of accretion, is misplaced.
the aforecited decision. The Director of Forestry also moved for the reconsideration
of the same decision. Both motions were opposed by private respondents on January First, the title of petitioners' own tract of land reveals its northeastern
27, 1979. boundary to be Manila Bay. Petitioners' land, therefore, used to adjoin, border or front
the Manila Bay and not any of the two rivers whose torrential action, petitioners insist,
On November 21, 1980, respondent appellate court promulgated a is to account for the accretion on their land. In fact, one of the petitioners, Sulpicio
resolution denying the motion for reconsideration filed by the Director of Forestry. It, Pascual, testified in open court that the waves of Manila Bay used to hit the disputed
however, modified its decision, to read, viz: land being part of the bay's foreshore but, after he had planted palapat and bakawan
trees thereon in 1948, the land began to rise. 16
"(3). Ordering private oppositors Heirs of
Emiliano Navarro to vacate that portion included in their Moreover, there is no dispute as to the location of: (a) the disputed land;
fishpond permit covered by Plan Psu-175181 and hand (b) petitioners' own tract of land; (c) the Manila Bay; and, (d) the Talisay and Bulacan
over possession of said portion to applicants-appellants, Rivers. Petitioners' own land lies between the Talisay and Bulacan Rivers; in front of
if the said portion is not within the strip of land fifty (50) their land on the northern side lies now the disputed land where before 1948, there
meters wide along Manila Bay on the northern portion of lay the Manila Bay. If the accretion were to be attributed to the action of either or both
the land subject of the registration proceedings and of the Talisay and Bulacan Rivers, the alluvium should have been deposited on either
which area is more particularly referred to as fifty (50) or both of the eastern and western boundaries of petitioners' own tract of land, not on
meters from corner 2 towards corner 1; and fifty (50) the northern portion thereof which is adjacent to the Manila Bay. Clearly lacking, thus,
meters from corner 5 towards corner 6 of Plan Psu- is the third requisite of accretion, which is, that the alluvium is deposited on the portion
175181 . . ." 9 of claimant's land which is adjacent to the river bank.

On December 15, 1980, we granted the Solicitor General, acting as Second, there is no dispute as to the fact that petitioners' own tract of
counsel for the Director of Forestry, an extension of time within which to file in this land adjoins the Manila Bay. Manila Bay is obviously not a river, and jurisprudence is
court, a petition for review of the decision dated November 29, 1978 of the respondent already settled as to what kind of body of water the Manila Bay is. It is to be
appellate court and of the aforecited resolution dated November 21, 1980. remembered that we held that:

Thereafter, the Solicitor General, in behalf of the Director of Forestry, "Appellant next contends that . . . Manila Bay
filed a petition for review entitled, "The Director of Forestry vs. the Court of cannot be considered as a sea. We find said contention
Appeals." 10 We, however, denied the same in a minute resolution dated July 20, untenable. A bay is part of the sea, being a mere
1981, such petition having been prematurely filed at a time when the Court of Appeals indentation of the same:
was yet to resolve petitioners' pending motion to set aside the resolution dated
November 21, 1980. 'Bay. — An opening into the
land where the water is shut in on all sides
On October 9, 1981, respondent appellate court denied petitioners' except at the entrance; an inlet of the sea;
motion for reconsideration of the decision dated November 29, 1978. an arm of the sea, distinct from a river, a
bending or curbing of the shore of the sea or
On October 17, 1981, respondent appellate court made an entry of of a lake.' 7 C.J. 1013-1014." 17
judgment stating that the decision dated November 29, 1978 had become final and
executory as against herein petitioners as oppositors in L.R.C. Case No. N-84 and The disputed land, thus, is an accretion not on a river bank but on a sea
Civil Case No. 2873 of the Court of First Instance (now the Regional Trial Court) of bank, or on what used to be the foreshore of Manila Bay which adjoined petitioners'
Balanga, Bataan. own tract of land on the northern side. As such, the applicable law is not Article 457
of the Civil Code but Article 4 of the Spanish Law of Waters of 1866.
On October 26, 1981, a second motion for reconsideration of the
decision dated November 29, 1978 was filed by petitioners' new counsel. The process by which the disputed land was formed, is not difficult to
discern from the facts of the case. As the trial court correctly observed:
On March 26, 1982, respondent appellate court issued a resolution
granting petitioners' request for leave to file a second motion for reconsideration. "A perusal of the survey plan . . . of the land
subject matter of these cases shows that on the eastern
On July 13, 1984, after hearing, respondent appellate court denied side, the property is bounded by Talisay River, on the
petitioners' second motion for reconsideration on the ground that the same was filed western side by Bulacan River, on the southern side by
out of time, citing Rule 52, Section 1 of the Rules of Court which provides that a Lot 1436 and on the northern side by Manila Bay. It is not
correct to state that the Talisay and Bulacan Rivers meet
a certain portion because the two rivers both flow up eventually raising the former shore leading to the
towards Manila Bay. The Talisay River is straight while formation of the land in question." 19
the Bulacan River is a little bit meandering and there is
no portion where the two rivers meet before they end up In other words, the combined and interactive effect of the planting of palapat
at Manila Bay. The land which is adjacent to the property and bakawan trees, the withdrawal of the waters of Manila Bay eventually
belonging to Pascual cannot be considered an accretion resulting in the drying up of its former foreshore, and the regular torrential
caused by the action of the two rivers]. action of the waters of Manila Bay, is the formation of the disputed land on
the northern boundary of petitioners' own tract of land.
Applicant Pascual . . . has not presented
proofs to convince the Court that the land he has applied
for registration is the result of the settling down on his The disputed property is an accretion on a sea bank, Manila Bay being an
registered land of soil, earth or other deposits so as to be inlet or an arm of the sea; as such, the disputed property is, under Article 4 of
rightfully be considered as an accretion [caused by the the Spanish Law of Waters of 1866, part of the public domain
action of the two rivers]. Said Art. 457 finds no
applicability where the accretion must have been caused
by action of the bay." 18 At the outset, there is a need to distinguish between Manila Bay and
Laguna de Bay.
The conclusion formed by the trial court on the basis of the foregoing
observation is that the disputed land is part of the foreshore of Manila Bay and While we held in the case of Ignacio v. Director of Lands and
therefore, part of the public domain. The respondent appellate court, however, Valeriano 20 that Manila Bay is considered a sea for purposes of determining which
perceived the fact that petitioners' own land lies between the Talisay and Bulacan law on accretion is to be applied in multifarious situations, we have ruled differently
Rivers, to be basis to conclude that the disputed land must be an accretion formed insofar as accretions on lands adjoining the Laguna de Bay are concerned.
by the action of the two rivers because petitioners' own land acted as a barricade
preventing the two rivers to meet and that the current of the two rivers carried In the cases of Government of the P.I. v. Colegio de San
sediments of sand and silt downwards to the Manila Bay which accumulated Jose 21 , Republic v. Court of Appeals 22 , Republic v. Alagad 23 , and Meneses
somehow to a 14-hectare land. These conclusions, however, are fatally incongruous v. Court of Appeals24 , we categorically ruled that Laguna de Bay is a lake the
in the light of the one undisputed critical fact: the accretion was deposited, not on accretion on which, by the mandate of Article 84 of the Spanish Law of Waters of
either the eastern or western portion of petitioners' land where a river each runs, but 1866, belongs to the owner of the land contiguous thereto.
on the northern portion of petitioners' land which adjoins the Manila Bay. Worse, such
conclusions are further eroded of their practical logic and consonance with natural The instant controversy, however, brings a situation calling for the
experience in the light of Sulpicio Pascual's admission as to having planted palapat application of Article 4 of the Spanish Law of Waters of 1866, the disputed land being
and bakawan trees on the northern boundary of their own land. In amplification of an accretion on the foreshore of Manila Bay which is, for all legal purposes,
this, plainly more reasonable and valid are Justice Mariano Serrano's observations in considered a sea.
his dissenting opinion when he stated that:
Article 4 of the Spanish Law of Waters of August 3, 1866 provides as
"As appellants' (titled) land . . . acts as a follows:
barricade that prevents the two rivers to meet, and
considering the wide expanse of the boundary between "Lands added to the shores by accretions
said land and the Manila Bay, measuring some 593.00 and alluvial deposits caused by the action of the sea,
meters . . . it is believed rather farfetched for the land in form part of the public domain. When they are no longer
question to have been formed through 'sediments of washed by the waters of the sea and are not necessary
sand and salt [sic] . . . deposited at their [rivers'] mouths.' for purposes of public utility, or for the establishment of
Moreover, if 'since the flow of the two rivers is downwards special industries, or for the coast-guard service, the
to the Manila Bay the sediments of sand and silt are Government shall declare them to be the property of the
deposited at their mouths,' why then would the alleged owners of the estates adjacent thereto and as increment
cargo of sand, silt and clay accumulate at the northern thereof."
portion of appellants' titled land facing Manila Bay
In the light of the aforecited vintage but still valid law, unequivocal is the
instead of merely at the mouths and banks of these two
public nature of the disputed land in this controversy, the same being an accretion on
rivers? That being the case, the accretion formed at said
a sea bank which, for all legal purposes, the foreshore of Manila Bay is. As part of
portion of appellants' titled [land] was not caused by the
the public domain, the herein disputed land is intended for public uses, and "so long
current of the two rivers but by the action of the sea
as the land in litigation belongs to the national domain and is reserved for public uses,
(Manila Bay) into which the rivers empty.
it is not capable of being appropriated by any private person, except through express
The conclusion . . . is not supported by any authorization granted in due form by a competent authority." 25 Only the executive
reference to the evidence which, on the contrary, shows and possibly the legislative departments have the right and the power to make the
that the disputed land was formed by the action of the declaration that the lands so gained by action of the sea is no longer necessary for
sea. Thus, no less than Sulpicio Pascual, one of the heirs purposes of public utility or for the cause of establishment of special industries or for
of the original applicant, testified on cross-examination coast guard services. 26 Petitioners utterly fail to show that either the executive or
that the land in dispute was part of the shore and it was legislative department has already declared the disputed land as qualified, under
only in 1948 that he noticed that the land was beginning Article 4 of the Spanish Law of Waters of 1866, to be the property of petitioners as
to get higher after he had planted trees thereon in 1948. owners of the estates adjacent thereto.
. . . cdasia
WHEREFORE, the instant Petition for Review is hereby DENIED and
. . . it is established that before 1948 sea DISMISSED.
water from the Manila Bay at high tide could reach as far Costs against petitioners.
as the dike of appellants' fishpond within their titled
property, which dike now separates this titled property SO ORDERED.
from the land in question. Even in 1948 when appellants
had already planted palapat and bakawan trees in the ||| (Heirs of Navarro v. Intermediate Appellate Court, G.R. No. 68166, [February 12,
land involved, inasmuch as these trees were yet small, 1997], 335 PHIL 537-556)
the waves of the sea could still reach the dike. This must
be so because in . . . the survey plan of the titled property
approved in 1918, said titled land was bounded on the
north by Manila Bay. So Manila Bay was adjacent to it on
the north. It was only after the planting of the aforesaid
trees in 1948 that the land in question began to rise or to
get higher in elevation.

The trees planted by appellants in 1948


became a sort of strainer of the sea water and at the
same time a kind of block to the strained sediments from
being carried back to the sea by the very waves that
brought them to the former shore at the end of the dike,
which must have caused the shoreline to recede and dry
EN BANC On January 19, 1988, then President Corazon C. Aquino issued
[G.R. No. 133250. July 9, 2002.] Special Patent No. 3517, granting and transferring to PEA "the parcels of
land so reclaimed under the Manila-Cavite Coastal Road and Reclamation
FRANCISCO I. CHAVEZ, petitioner,vs.PUBLIC Project (MCCRRP) containing a total area of one million nine hundred
ESTATES AUTHORITY and AMARI COASTAL fifteen thousand eight hundred ninety four (1,915,894) square meters."
BAY DEVELOPMENT Subsequently, on April 9, 1988, the Register of Deeds of the Municipality of
CORPORATION, respondents. Parañaque issued Transfer Certificates of Title Nos. 7309, 7311, and 7312,
in the name of PEA, covering the three reclaimed islands known as the
DECISION "Freedom Islands" located at the southern portion of the Manila-Cavite
Coastal Road, Parañaque City. The Freedom Islands have a total land area
CARPIO, J p: of One Million Five Hundred Seventy Eight Thousand Four Hundred and
This is an original Petition for Mandamus with prayer for a writ of Forty One (1,578,441) square meters or 157.841 hectares.
preliminary injunction and a temporary restraining order. The petition seeks
to compel the Public Estates Authority ("PEA" for brevity) to disclose all facts On April 25, 1995, PEA entered into a Joint Venture Agreement
on PEA's then on-going renegotiations with Amari Coastal Bay and ("JVA" for brevity) with AMARI, a private corporation, to develop the
Development Corporation ("AMARI" for brevity) to reclaim portions of Manila Freedom Islands. The JVA also required the reclamation of an additional
Bay. The petition further seeks to enjoin PEA from signing a new agreement 250 hectares of submerged areas surrounding these islands to complete
with AMARI involving such reclamation. the configuration in the Master Development Plan of the Southern
Reclamation Project-MCCRRP. PEA and AMARI entered into the JVA
The Facts through negotiation without public bidding. 4 On April 28, 1995, the Board
of Directors of PEA, in its Resolution No. 1245, confirmed the JVA. 5 On
On November 20, 1973, the government, through the
June 8, 1995, then President Fidel V. Ramos, through then Executive
Commissioner of Public Highways, signed a contract with the Construction
Secretary Ruben Torres, approved the JVA. 6
and Development Corporation of the Philippines ("CDCP' for brevity) to
reclaim certain foreshore and offshore areas of Manila Bay. The contract On November 29, 1996, then Senate President Ernesto Maceda
also included the construction of Phases I and II of the Manila-Cavite delivered a privilege speech in the Senate and denounced the JVA as the
Coastal Road. CDCP obligated itself to carry out all the works in "grandmother of all scams." As a result, the Senate Committee on
consideration of fifty percent of the total reclaimed land. Government Corporations and Public Enterprises, and the Committee on
Accountability of Public Officers and Investigations, conducted a joint
On February 4, 1977, then President Ferdinand E. Marcos
investigation. The Senate Committees reported the results of their
issued Presidential Decree No. 1084 creating PEA. PD No. 1084 tasked
investigation in Senate Committee Report No. 560 dated September 16,
PEA "to reclaim land, including foreshore and submerged areas," and "to
1997. 7 Among the conclusions of their report are: (1) the reclaimed lands
develop, improve, acquire, . . . lease and sell any and all kinds of
PEA seeks to transfer to AMARI under the JVA are lands of the public
lands." 1 On the same date, then President Marcos issued Presidential
domain which the government has not classified as alienable lands and
Decree No. 1085 transferring to PEA the "lands reclaimed in the foreshore
therefore PEA cannot alienate these lands; (2) the certificates of title
and offshore of the Manila Bay " 2 under the Manila-Cavite Coastal Road
covering the Freedom Islands are thus void, and (3) the JVA itself is illegal.
and Reclamation Project (MCCRRP).
On December 5, 1997, then President Fidel V. Ramos issued
On December 29, 1981, then President Marcos issued a
Presidential Administrative Order No. 365 creating a Legal Task Force to
memorandum directing PEA to amend its contract with CDCP, so that "[A]ll
conduct a study on the legality of the JVA in view of Senate Committee
future works in MCCRRP ...shall be funded and owned by PEA."
Report No. 560. The members of the Legal Task Force were the Secretary
Accordingly, PEA and CDCP executed a Memorandum of Agreement dated
of Justice, 8 the Chief Presidential Legal Counsel, 9 and the Government
December 29, 1981, which stated:
Corporate Counsel. 10 The Legal Task Force upheld the legality of the JVA,
"(i) CDCP shall undertake all contrary to the conclusions reached by the Senate Committees. 11
reclamation, construction, and such other works in
On April 4 and 5, 1998, the Philippine Daily
the MCCRRP as may be agreed upon by the
Inquirer and Today published reports that there were on-going
parties, to be paid according to progress of works
renegotiations between PEA and AMARI under an order issued by then
on a unit price/lump sum basis for items of work to
President Fidel V. Ramos. According to these reports, PEA Director Nestor
be agreed upon, subject to price escalation,
Kalaw, PEA Chairman Arsenio Yulo and retired Navy Officer Sergio Cruz
retention and other terms and conditions provided
composed the negotiating panel of PEA.
for in Presidential Decree No. 1594. All the
financing required for such works shall be provided On April 13, 1998, Antonio M. Zulueta filed before the Court
by PEA. a Petition for Prohibition with Application for the Issuance of a Temporary
Restraining Order and Preliminary Injunction docketed as G.R. No. 132994
xxx xxx xxx
seeking to nullify the JVA. The Court dismissed the petition "for unwarranted
(iii) ...CDCP shall give up all its disregard of judicial hierarchy, without prejudice to the refiling of the case
development rights and hereby agrees to cede before the proper court." 12
and transfer in favor of PEA, all of the rights, title,
On April 27, 1998, petitioner Frank I. Chavez ("Petitioner" for
interest and participation of CDCP in and to all the
brevity) as a taxpayer, filed the instant Petition for Mandamus with Prayer
areas of land reclaimed by CDCP in the MCCRRP
for the Issuance of a Writ of Preliminary Injunction and Temporary
as of December 30, 1981 which have not yet been
Restraining Order. Petitioner contends the government stands to lose
sold, transferred or otherwise disposed of by
billions of pesos in the sale by PEA of the reclaimed lands to AMARI.
CDCP as of said date, which areas consist of
Petitioner prays that PEA publicly disclose the terms of any renegotiation of
approximately Ninety-Nine Thousand Four
the JVA, invoking Section 28, Article II, and Section 7, Article III, of the 1987
Hundred Seventy Three (99,473) square meters in
Constitution on the right of the people to information on matters of public
the Financial Center Area covered by land pledge
concern. Petitioner assails the sale to AMARI of lands of the public domain
No. 5 and approximately Three Million Three
as a blatant violation of Section 3, Article XII of the 1987
Hundred Eighty Two Thousand Eight Hundred
Constitution prohibiting the sale of alienable lands of the public domain to
Eighty Eight (3,382,888) square meters of
private corporations. Finally, petitioner asserts that he seeks to enjoin the
reclaimed areas at varying elevations above Mean
loss of billions of pesos in properties of the State that are of public dominion.
Low Water Level located outside the Financial
Center Area and the First Neighborhood Unit." 3 After several motions for extension of time, 13 PEA and AMARI
filed their Comments on October 19, 1998 and June 25, 1998, respectively.
Meanwhile, on December 28, 1998, petitioner filed an Omnibus Motion: (a)
to require PEA to submit the terms of the renegotiated PEA-AMARI contract; signing of the Amended JVA is now moot because PEA and AMARI have
(b) for issuance of a temporary restraining order; and (c) to set the case for already signed the Amended JVA on March 30, 1999. Moreover, the Office
hearing on oral argument. Petitioner filed a Reiterative Motion for Issuance of the President has approved the Amended JVA on May 28, 1999.
of a TRO dated May 26, 1999, which the Court denied in a Resolution dated
June 22, 1999. Petitioner counters that PEA and AMARI cannot avoid the
constitutional issue by simply fast-tracking the signing and approval of the
In a Resolution dated March 23, 1999, the Court gave due Amended JVA before the Court could act on the issue. Presidential approval
course to the petition and required the parties to file their respective does not resolve the constitutional issue or remove it from the ambit of
memoranda. judicial review.
On March 30, 1999, PEA and AMARI signed the Amended Joint We rule that the signing and of the Amended JVA by PEA and
Venture Agreement ("Amended JVA," for brevity).On May 28, 1999, the AMARI and its approval by the President cannot operate to moot the petition
Office of the President under the administration of then President Joseph E. and divest the Court of its jurisdiction. PEA and AMARI have still to
Estrada approved the Amended JVA. implement the Amended JVA. The prayer to enjoin the signing of the
Amended JVA on constitutional grounds necessarily includes preventing its
Due to the approval of the Amended JVA by the Office of the implementation if in the meantime PEA and AMARI have signed one in
President, petitioner now prays that on "constitutional and statutory grounds violation of the Constitution. Petitioner's principal basis in assailing the
the renegotiated contract be declared null and void." 14 renegotiation of the JVA is its violation of the Section 3, Article XII of
the Constitution, which prohibits the government from alienating lands of the
The Issues
public domain to private corporations. If the Amended JVA indeed violates
The issues raised by petitioner, PEA 15 and the Constitution, it is the duty of the Court to enjoin its implementation, and
AMARI 16 are as follows: if already implemented, to annul the effects of such unconstitutional
I. WHETHER THE PRINCIPAL RELIEFS contract.
PRAYED FOR IN THE PETITION ARE The Amended JVA is not an ordinary commercial contract but
MOOT AND ACADEMIC BECAUSE one which seeks to transfer title and ownership to 367.5 hectares of
OF SUBSEQUENT EVENTS; reclaimed lands and submerged areas of Manila Bay to a single private
II. WHETHER THE PETITION MERITS corporation. It now becomes more compelling for the Court to resolve the
DISMISSAL FOR FAILING TO issue too insure the government itself does not violate a provision of
OBSERVE THE PRINCIPLE the Constitution intended to safeguard the national patrimony. Supervening
GOVERNING THE HIERARCHY OF events whether intended or accidental, cannot prevent the Court from
COURTS; rendering a decision if there is a grave violation of the Constitution. In the
instant case, if the Amended JVA runs counter to theConstitution, the Court
III. WHETHER THE PETITION MERITS can still prevent the transfer of title and ownership of alienable lands of the
DISMISSAL FOR NON-EXHAUSTION public domain in the name of AMARI. Even in cases where supervening
OF ADMINISTRATIVE REMEDIES; events had made the cases moot, the Court did not hesitate to resolve the
legal or constitutional issues raised to formulate controlling principles to
IV. WHETHER PETITIONER HAS LOCUS guide the bench, bar, and the public. 17
STANDI TO BRING THIS SUIT;
Also, the instant petition is a case of first impression. All previous
V. WHETHER THE CONSTITUTIONAL RIGHT decisions of the Court involving Section 3, Article XII of the 1987
TO INFORMATION INCLUDES Constitution, or its counterpart provision in the 1973
OFFICIAL INFORMATION ON ON- Constitution, 18 covered agricultural lands sold to private corporations
GOING NEGOTIATIONS BEFORE A which acquired the lands from private parties. The transferors of the private
FINAL AGREEMENT; corporations claimed or could claim the right to judicial confirmation of their
imperfect titles 19 under Title II of Commonwealth Act. 141 ("CA No. 141"
VI. WHETHER THE STIPULATIONS IN THE for brevity). In the instant case, AMARI seeks to acquire from PEA, a public
AMENDED JOINT VENTURE corporation, reclaimed lands and submerged areas for non-
AGREEMENT FOR THE TRANSFER agricultural purposes by purchase under PD No. 1084 (charter of PEA)
TO AMARI OF CERTAIN LANDS, and Title III of CA No. 141. Certain undertakings by AMARI under the
RECLAIMED AND STILL TO BE Amended JVA constitute the consideration for the purchase. Neither AMARI
RECLAIMED, VIOLATE THE 1987 nor PEA can claim judicial confirmation of their titles because the lands
CONSTITUTION; AND covered by the Amended JVA are newly reclaimed or still to be reclaimed.
VII. WHETHER THE COURT IS THE PROPER Judicial confirmation of imperfect title requires open, continuous, exclusive
FORUM FOR RAISING THE ISSUE and notorious occupation of agricultural lands of the public domain for at
OF WHETHER THE AMENDED JOINT least thirty years since June 12, 1945 or earlier. Besides, the deadline for
VENTURE AGREEMENT IS filing applications for judicial confirmation of imperfect title expired on
GROSSLY DISADVANTAGEOUS TO December 31, 1987. 20
THE GOVERNMENT. Lastly, there is a need to resolve immediately the constitutional
The Court's Ruling issue raised in this petition because of the possible transfer at any time by
PEA to AMARI of title and ownership to portions of the reclaimed lands.
First issue: whether the principal reliefs prayed for in the petition are Under the Amended JVA, PEA is obligated to transfer to AMARI the latter's
moot and academic because of subsequent events. seventy percent proportionate share in the reclaimed areas as the
reclamation progresses. The Amended JVA even allows AMARI to
mortgage at any time the entire reclaimed area to raise financing for the
The petition prays that PEA publicly disclose the "terms and reclamation project. 21
conditions of the on-going negotiations for a new agreement." The petition
also prays that the Court enjoin PEA from "privately entering into, perfecting Second issue: whether the petition merits dismissal for failing to
and/or executing any new agreement with AMARI. observe the principle governing the hierarchy of courts.

"PEA and AMARI claim the petition is now moot and academic PEA and AMARI claim petitioner ignored the judicial hierarchy
because AMARI furnished petitioner on June 21, 1999 a copy of the signed by seeking relief directly from the Court. The principle of hierarchy of courts
Amended JVA containing the terms and conditions agreed upon in the applies generally to cases involving factual questions. As it is not a trier of
renegotiations. Thus, PEA has satisfied petitioner's prayer for a public facts, the Court cannot entertain cases involving factual issues. The instant
disclosure of the renegotiations. Likewise, petitioner's prayer to enjoin the case, however, raises constitutional issues of transcendental importance to
the public. 22 The Court can resolve this case without determining any "Besides, petitioner emphasizes, the
factual issue related to the case. Also, the instant case is a petition matter of recovering the ill-gotten wealth of the
for mandamus which falls under the original jurisdiction of the Court Marcoses is an issue of 'transcendental
under Section 5, Article VIII of the Constitution. We resolve to exercise importance to the public.' He asserts that ordinary
primary jurisdiction over the instant case. taxpayers have a right to initiate and prosecute
actions questioning the validity of acts or orders of
Third issue: whether the petition merits dismissal for non-exhaustion government agencies or instrumentalities, if the
of administrative remedies. issues raised are of 'paramount public interest,'
PEA faults petitioner for seeking judicial intervention in and if they 'immediately affect the social, economic
compelling PEA to disclose publicly certain information without first asking and moral well-being of the people.'
PEA the needed information. PEA claims petitioner's direct resort to the
Moreover, the mere fact that he is a
Court violates the principle of exhaustion of administrative remedies. It also
citizen satisfies the requirement of personal
violates the rule thatmandamus may issue only if there is no other plain,
interest, when the proceeding involves the
speedy and adequate remedy in the ordinary course of law.
assertion of a public right, such as in this case. He
PEA distinguishes the instant case from Tañada v. invokes several decisions of this Court which have
Tuvera 23 where the Court granted the petition for mandamus even if the set aside the procedural matter of locus
petitioners there did not initially demand from the Office of the President the standi,when the subject of the case involved public
publication of the presidential decrees. PEA points out that in Tañada, the interest.
Executive Department had anaffirmative statutory duty under Article 2 of the
xxx xxx xxx
Civil Code 24 and Section 1 of Commonwealth Act No. 638 25 to publish
the presidential decrees. There was, therefore, no need for the petitioners In Tañada v. Tuvera,the Court asserted
in Tañada to make an initial demand from the Office of the President. In the that when the issue concerns a public right and the
instant case, PEA claims it has no affirmative statutory duty to disclose object of mandamus is to obtain the enforcement
publicly information about its renegotiation of the JVA. Thus, PEA asserts of a public duty, the people are regarded as the
that the Court must apply the principle of exhaustion of administrative real parties in interest; and because it is sufficient
remedies to the instant case in view of the failure of petitioner here to that petitioner is a citizen and as such is interested
demand initially from PEA the needed information. in the execution of the laws, he need not show that
he has any legal or special interest in the result of
The original JVA sought to dispose to AMARI public lands held
the action. In the aforesaid case, the petitioners
by PEA, a government corporation. Under Section 79 of the Government
sought to enforce their right to be informed on
Auditing Code, 26the disposition of government lands to private parties
matters of public concern, a right then recognized
requires public bidding. PEA was under a positive legal duty to disclose to
in Section 6, Article IV of the 1973 Constitution, in
the public the terms andconditions for the sale of its lands.The law obligated
connection with the rule that laws in order to be
PEA make this public disclosure even without demand from petitioner or
valid and enforceable must be published in the
from anyone. PEA failed to make this public disclosure because the original
Official Gazette or otherwise effectively
JVA, like the Amended JVA, was the result of a negotiated contract, not of
promulgated. In ruling for the petitioners' legal
a public bidding. Considering that PEA had an affirmative statutory duty to
standing, the Court declared that the right they
make the public disclosure, and was even in breach of this legal duty,
sought to be enforced 'is a public right recognized
petitioner had the right to seek direct judicial intervention.
by no less than the fundamental law of the land.'
Moreover, and this alone, is determinative of this issue, the
Legaspi v. Civil Service
principle of exhaustion of administrative remedies does not apply when the
Commission,while reiterating Tañada,further
issue involved is a purely legal or constitutional question. 27 The principal
declared that 'when a mandamus proceeding
issue in the instant case is the capacity of AMARI to acquire lands held by
involves the assertion of a public right, the
PEA in view of the constitutional ban prohibiting the alienation of lands of
requirement of personal interest is satisfied by the
the public domain to private corporations. We rule that the principle of
mere fact that petitioner is a citizen and, therefore,
exhaustion of administrative remedies does not apply in the instant case.
part of the general 'public' which possesses the
Fourth issue: whether petitioner has locus standi to bring this suit. right.'
PEA argues that petitioner has no standing to Further, in Albano v. Reyes, we said
institute mandamus proceedings to enforce his constitutional right to that while expenditure of public funds may not
information without a showing that PEA refused to perform an affirmative have been involved under the questioned contract
duty imposed on PEA by the Constitution. PEA also claims that petitioner for the development, management and operation
has not shown that he will suffer any concrete injury because of the signing of the Manila International Container Terminal,
or implementation of the Amended JVA. Thus, there is no actual controversy 'public interest [was] definitely involved
requiring the exercise of the power of judicial review. considering the important role [of the subject
contract] . . . in the economic development of the
The petitioner has standing to bring this taxpayer's suit because country and the magnitude of the financial
the petition seeks to compel PEA to comply with its constitutional duties. consideration involved.' We concluded that, as a
There are two constitutional issues involved here. First is the right of citizens consequence, the disclosure provision in
to information on matters of public concern. Second is the application of a the Constitution would constitute sufficient
constitutional provision intended to insure the equitable distribution of authority for upholding the petitioner's standing.
alienable lands of the public domain among Filipino citizens. The thrust of
the first issue is to compel PEA to disclose publicly information on the sale Similarly, the instant petition is
of government lands worth billions of pesos, information which anchored on the right of the people to information
the Constitution and statutory law mandate PEA to disclose. The thrust of and access to official records, documents and
the second issue is to prevent PEA from alienating hundreds of hectares of papers — a right guaranteed under Section 7,
alienable lands of the public domain in violation of the Constitution, Article III of the 1987 Constitution. Petitioner, a
compelling PEA to comply with a constitutional duty to the nation. former solicitor general, is a Filipino citizen.
Because of the satisfaction of the two basic
Moreover, the petition raises matters of transcendental requisites laid down by decisional law to sustain
importance to the public. In Chavez v. PCGG, 28 the Court upheld the right petitioner's legal standing, i.e. (1) the enforcement
of a citizen to bring a taxpayer's suit on matters of transcendental of a public right (2) espoused by a Filipino citizen,
importance to the public, thus — we rule that the petition at bar should be allowed."
We rule that since the instant petition, brought by a citizen, "Mr. Suarez.And when we say
involves the enforcement of constitutional rights — to information and to the 'transactions' which should be distinguished from
equitable diffusion of natural resources — matters of transcendental public contracts, agreements, or treaties or whatever,
importance, the petitioner has the requisite locus standi. does the Gentleman refer to the steps leading to
the consummation of the contract, or does he refer
Fifth issue: whether the constitutional right to information to the contract itself?
includes official information on on-going negotiations before a final
agreement. Mr. Ople: The 'transactions' used here,
I suppose is generic and therefore, it can cover
Section 7, Article III of the Constitution explains the people's
both steps leading to a contract and already
right to information on matters of public concern in this manner:
consummated contract, Mr. Presiding Officer.
"Sec. 7. The right of the people to
Mr. Suarez: This contemplates
information on matters of public concern shall be
inclusion of negotiations leading to the
recognized. Access to official records, and to
consummation of the transaction.
documents, and papers pertaining to official acts,
transactions, or, decisions, as well as to Mr. Ople: Yes, subject only to
government research data used as basis for policy reasonable safeguards on the national interest.
development, shall be afforded the citizen, subject
to such limitations as may be provided by law." Mr. Suarez:Thank you." 32 (Italics
(Emphasis supplied) supplied)

The State policy of full transparency in all transactions involving AMARI argues there must first be a consummated contract before
public interest reinforces the people's right to information on matters petitioner can invoke the right. Requiring government officials to
of public concern. This State policy is expressed in Section 28, reveal their deliberations at the pre-decisional stage will degrade the
Article II of the Constitution, thus: quality of decision-making in government agencies. Government
officials will hesitate to express their real sentiments during
"Sec. 28. Subject to reasonable
deliberations if there is immediate public dissemination of their
conditions prescribed by law, the State adopts and
discussions, putting them under all kinds of pressure before they
implements a policy of full public disclosure of all
decide.
its transactions involving public interest." (Italics
supplied) We must first distinguish between information the law on public
bidding requires PEA to disclose publicly, and information the constitutional
These twin provisions of the Constitution seek to promote right to information requires PEA to release to the public. Before the
transparency in policy-making and in the operations of the consummation of the contract, PEA must, on its own and without demand
government, as well as provide the people sufficient information to from anyone, disclose to the public matters relating to the disposition of its
exercise effectively other constitutional rights. These twin provisions property. These include the size, location, technical description and nature
are essential to the exercise of freedom of expression. If the of the property being disposed of, the terms and conditions of the
government does not disclose its official acts, transactions and disposition, the parties qualified to bid, the minimum price and similar
decisions to citizens, whatever citizens say, even if expressed information. PEA must prepare all these data and disclose them to the
without any restraint, will be speculative and amount to public at the start of the disposition process, long before the consummation
nothing. These twin provisions are also essential to hold public of the contract, because the Government Auditing Code requires public
officials "at all times . . . accountable to the people," 29 for unless bidding.If PEA fails to make this disclosure, any citizen can demand from
citizens have the proper information, they cannot hold public officials PEA this information at any time during the bidding process.
accountable for anything. Armed with the right information, citizens
can participate in public discussions leading to the formulation of Information, however, on on-going evaluation or review of bids
government policies and their effective implementation. An informed or proposals being undertaken by the bidding or review committee is not
citizenry is essential to the existence and proper functioning of any immediately accessible under the right to information. While the evaluation
democracy. As explained by the Court in Valmonte v. Belmonte, or review is still on-going, there are no "official acts, transactions, or
Jr. 30 — decisions" on the bids or proposals. However, once the committee makes
its official recommendation,there arises a "definite proposition" on the part
"An essential element of these
of the government. From this moment, the public's right to information
freedoms is to keep open a continuing dialogue or
attaches, and any citizen can access all the non-proprietary information
process of communication between the
leading to such definite proposition. InChavez v. PCGG, 33 the Court ruled
government and the people. It is in the interest of
as follows:
the State that the channels for free political
discussion be maintained to the end that the "Considering the intent of the framers
government may perceive and be responsive to of the Constitution, we believe that it is incumbent
the people's will. Yet, this open dialogue can be upon the PCGG and its officers, as well as other
effective only to the extent that the citizenry is government representatives, to disclose sufficient
informed and thus able to formulate its will public informations on any proposed settlement
intelligently. Only when the participants in the they have decided to take up with the ostensible
discussion are aware of the issues and have owners and holders of ill-gotten wealth. Such
access to information relating thereto can such information though, must pertain to definite
bear fruit." propositions of the government,not necessarily to
intra-agency or inter-agency recommendations or
PEA asserts, citing Chavez v. PCGG, 31 that in cases of on-
communications during the stage when common
going negotiations the right to information is limited to "definite propositions
assertions are still in the process of being
of the government." PEA maintains the right does not include access to
formulated or are in the "exploratory" stage. There
"intra-agency or inter-agency recommendations or communications during
is need, of course, to observe the same restrictions
the stage when common assertions are still in the process of being
on disclosure of information in general, as
formulated or are in the 'exploratory stage.'"
discussed earlier — such as on matters involving
Also AMARI contends that petitioner cannot invoke the right at national security, diplomatic or foreign relations,
the pre-decisional stage or before the closing of the transaction. To support intelligence and other classified information."
its contention, AMARI cites the following discussion in the 1986 (Italics supplied)
Constitutional Commission:
Contrary to AMARI's contention, the commissioners of the 1986 Sixth issue: whether stipulations in the Amended JVA for the transfer
Constitutional Commission understood that the right to to AMARI of lands, reclaimed or to be reclaimed, violate
information "contemplates inclusion of negotiations leading to the the Constitution.
consummation of the transaction." Certainly, a consummated contract is not
The Regalian Doctrine
a requirement for the exercise of the right to information. Otherwise, the
people can never exercise the right if no contract is consummated, and if The ownership of lands reclaimed from foreshore and
one is consummated, it may be too late for the public to expose its defects. submerged areas is rooted in the Regalian doctrine which holds that the
State owns all lands and waters of the public domain. Upon the Spanish
Requiring a consummated contract will keep the public in the conquest of the Philippines, ownership of all "lands, territories and
dark until the contract, which may be grossly disadvantageous to the possessions" in the Philippines passed to the Spanish Crown. 42 The King,
government or even illegal, becomes a fait accompli. This negates the State as the sovereign ruler and representative of the people, acquired and owned
policy of full transparency on matters of public concern, a situation which all lands and territories in the Philippines except those he disposed of by
the framers of theConstitution could not have intended. Such a requirement grant or sale to private individuals.
will prevent the citizenry from participating in the public discussion of
any proposed contract, effectively truncating a basic right enshrined in the The 1935, 1973 and 1987 Constitutions adopted the
Bill of Rights. We can allow neither an emasculation of a constitutional right, Regalian doctrine substituting, however, the State, in lieu of the King,
nor a retreat by the State of its avowed "policy of full disclosure of all its as the owner of all lands and waters of the public domain. The
transactions involving public interest." Regalian doctrine is the foundation of the time-honored principle of
land ownership that "all lands that were not acquired from the
The right covers three categories of information which are Government, either by purchase or by grant, belong to the public
"matters of public concern," namely: (1) official records; (2) documents and domain." 43 Article 339 of the Civil Code of 1889, which is now
papers pertaining to official acts, transactions and decisions; and (3) Article 420 of the Civil Code of 1950, incorporated the Regalian
government research data used in formulating policies. The first category doctrine.
refers to any document that is part of the public records in the custody of
government agencies or officials. The second category refers to documents Ownership and Disposition of Reclaimed Lands
and papers recording, evidencing, establishing, confirming, supporting, The Spanish Law of Waters of 1866 was the first statutory law
justifying or explaining official acts, transactions or decisions of government governing the ownership and disposition of reclaimed lands in the
agencies or officials. The third category refers to research data, whether Philippines. On May 18, 1907, the Philippine Commission enacted Act No.
raw, collated or processed, owned by the government and used in 1654 which provided for the lease, but not the sale, of reclaimed lands of
formulating government policies. the government to corporations and individuals. Later, on November 29,
1919, the Philippine Legislature approved Act No. 2874, the Public Land
The information that petitioner may access on the renegotiation
Act, which authorized the lease, but not the sale, of reclaimed lands of the
of the JVA includes evaluation reports, recommendations, legal and expert
government to corporations and individuals. On November 7, 1936, the
opinions, minutes of meetings, terms of reference and other documents
National Assembly passed Commonwealth Act No. 141, also known as the
attached to such reports or minutes, all relating to the JVA. However, the
Public Land Act, which authorized the lease, but not the sale, of reclaimed
right to information does not compel PEA to prepare lists, abstracts,
lands of the government to corporations and individuals. CA No.
summaries and the like relating to the renegotiation of the JVA. 34 The right
141 continues to this day as the general law governing the classification and
only affords access to records, documents and papers, which means the
disposition of lands of the public domain.
opportunity to inspect and copy them. One who exercises the right must
copy the records, documents and papers at his expense. The exercise of The Spanish Law of Waters of 1866 and the Civil Code of 1889
the right is also subject to reasonable regulations to protect the integrity of
the public records and to minimize disruption to government operations, like Under the Spanish Law of Waters of 1866, the shores, bays,
rules specifying when and how to conduct the inspection and copying. 35 coves, inlets and all waters within the maritime zone of the Spanish territory
belonged to the public domain for public use. 44 The Spanish Law of Waters
The right to information, however, does not extend to matters of 1866 allowed the reclamation of the sea under Article 5, which provided
recognized as privileged information under the separation of as follows:
powers. 36 The right does not also apply to information on military and
diplomatic secrets, information affecting national security, and information "Article 5. Lands reclaimed from the
on investigations of crimes by law enforcement agencies before the sea in consequence of works constructed by the
prosecution of the accused, which courts have long recognized as State, or by the provinces, pueblos or private
confidential. 37 The right may also be subject to other limitations that persons, with proper permission, shall become the
Congress may impose by law. property of the party constructing such works,
unless otherwise provided by the terms of the
There is no claim by PEA that the information demanded by grant of authority."
petitioner is privileged information rooted in the separation of powers. The
information does not cover Presidential conversations, correspondence, or Under the Spanish Law of Waters, land reclaimed from the sea
discussions during closed-door Cabinet meetings which, like internal belonged to the party undertaking the reclamation, provided the
deliberations of the Supreme Court and other collegiate courts, or executive government issued the necessary permit and did not reserve
sessions of either house of Congress, 38 are recognized as confidential. ownership of the reclaimed land to the State.
This kind of information cannot be pried open by a co-equal branch of Article 339 of the Civil Code of 1889 defined property of public
government. A frank exchange of exploratory ideas and assessments, free dominion as follows:
from the glare of publicity and pressure by interested parties, is essential to
protect the independence of decision-making of those tasked to exercise "Art. 339. Property of public dominion is
Presidential, Legislative and Judicial Power. 39 This is not the situation in —
the instant case.
1. That devoted to public use, such as roads,
We rule, therefore, that the constitutional right to information canals, rivers, torrents, ports and
includes official information on on-going negotiations before a final contract. bridges constructed by the State,
The information, however, must constitute definite propositions by the riverbanks, shores, roadsteads, and
government and should not cover recognized exceptions like privileged that of a similar character;
information, military and diplomatic secrets and similar matters affecting
national security and public order. 40 Congress has also prescribed other 2. That belonging exclusively to the State which,
limitations on the right to information in several legislations. 41 without being of general public use, is
employed in some public service, or in
the development of the national wealth,
such as walls, fortresses, and other
works for the defense of the territory, Act No. 1654, however did not repeal Section 5 of the Spanish
and mines, until granted to private Law of Waters of 1866. Act No. 1654 did not prohibit private parties from
individuals. reclaiming parts of the sea under Section 5 of the Spanish Law of Waters.
Lands reclaimed from the sea by private parties with government
Property devoted to public use referred to property open for use by permission remained private lands.
the public. In contrast, property devoted to public service referred to
property used for some specific public service and open only to Act No. 2874 of the Philippine Legislature
those authorized to use the property.
On November 29, 1919, the Philippine Legislature enacted Act
Property of public dominion referred not only to property devoted No. 2874, the Public Land Act. 46 The salient provisions of Act No. 2874,
to public use, but also to property not so used but employed to develop the on reclaimed lands, were as follows:
national wealth.This class of property constituted property of public
dominion although employed for some economic or commercial activity to "Sec. 6. The Governor-General, upon
increase the national wealth. the recommendation of the Secretary of
Agriculture and Natural Resources, shall from time
Article 341 of the Civil Code of 1889 governed the re- to time classify the lands of the public domain
classification of property of public dominion into private property, to wit: into —

"Art. 341. Property of public dominion, (a) Alienable or disposable,


when no longer devoted to public use or to the
defense of the territory, shall become a part of the (b) Timber, and
private property of the State."
(c) Mineral lands, ...
This provision, however, was not self-executing. The legislature, or
Sec. 7. For the purposes of the
the executive department pursuant to law, must declare the property
government and disposition of alienable or
no longer needed for public use or territorial defense before the
disposable public lands, the Governor-General,
government could lease or alienate the property to private parties. 45
upon recommendation by the Secretary of
Act No. 1654 of the Philippine Commission Agriculture and Natural Resources, shall from time
to time declare what lands are open to disposition
On May 8, 1907, the Philippine Commission enacted Act
or concession under this Act."
No. 1654 which regulated the lease of reclaimed and foreshore
lands. The salient provisions of this law were as follows: Sec. 8. Only those lands shall be
"Section 1. The control and disposition declared open to disposition or concession which
of the foreshore as defined in existing law, and have been officially delimited or classified....
the title to all Government or public lands made or
xxx xxx xxx
reclaimed by the Government by dredging or filling
or otherwise throughout the Philippine Sec. 55. Any tract of land of the public
Islands, shall be retained by the domain which, being neither timber nor mineral
Government without prejudice to vested rights and land, shall be classified as suitable for residential
without prejudice to rights conceded to the City of purposes or for commercial, industrial, or other
Manila in the Luneta Extension. productive purposes other than agricultural
purposes,and shall be open to disposition or
Section 2. (a) The Secretary of the
concession, shall be disposed of under the
Interior shall cause all Government or public lands
provisions of this chapter, and not otherwise.
made or reclaimed by the Government by
dredging or filling or otherwise to be divided into Sec. 56. The lands disposable under
lots or blocks, with the necessary streets and this title shall be classified as follows:
alleyways located thereon, and shall cause plats
and plans of such surveys to be prepared and filed (a) Lands reclaimed by the
with the Bureau of Lands. Government by dredging, filling, or other means;

(b) Upon completion of such plats and (b) Foreshore;


plans the Governor-General shall give notice to
the public that such parts of the lands so made or (c) Marshy lands or lands covered with
reclaimed as are not needed for public purposes water bordering upon the shores or banks of
will be leased for commercial and business navigable lakes or rivers;
purposes,.... (d) Lands not included in any of the
xxx xxx xxx foregoing classes.

(e) The leases above provided for shall xxx xxx xxx.
be disposed of to the highest and best Sec. 58. The lands comprised in
bidder therefore, subject to such regulations and classes (a),(b),and (c) of section fifty-six shall be
safeguards as the Governor-General may by disposed of to private parties by lease only and not
executive order prescribe." (Italics supplied) otherwise,as soon as the Governor-General, upon
Act No. 1654 mandated that the government should retain title recommendation by the Secretary of Agriculture
to all lands reclaimed by the government. The Act also vested in the and Natural Resources, shall declare that the
government control and disposition of foreshore lands. Private parties could same are not necessary for the public service and
lease lands reclaimed by the government only if these lands were no longer are open to disposition under this chapter.The
needed for public purpose. Act No. 1654 mandate public bidding in the lands included in class (d) may be disposed of by
lease of government reclaimed lands. Act No. 1654 made government sale or lease under the provisions of this
reclaimed lands sui generis in that unlike other public lands which the Act." (Italics supplied)
government could sell to private parties, these reclaimed lands were Section 6 of Act No. 2874 authorized the Governor-General to
available only for lease to private parties. "classify lands of the public domain into ...alienable or disposable" 47 lands.
Section 7 of the Act empowered the Governor-General to "declare what
lands are open to disposition or concession." Section 8 of the Act limited the measure and limit of the grant." (Italics
alienable or disposable lands only to those lands which have been "officially supplied)
delimited and classified."
The 1935 Constitution barred the alienation of all natural
Section 56 of Act No. 2874 stated that lands "disposable under resources except public agricultural lands, which were the only natural
this title 48 shall be classified" as government reclaimed, foreshore and resources the State could alienate. Thus, foreshore lands, considered part
marshy lands, as well as other lands. All these lands, however, must be of the State's natural resources, became inalienable by constitutional fiat,
suitable for residential, commercial, industrial or other productive non- available only for lease for 25 years, renewable for another 25 years. The
agricultural purposes. These provisions vested upon the Governor-General government could alienate foreshore lands only after these lands were
the power to classify inalienable lands of the public domain into disposable reclaimed and classified as alienable agricultural lands of the public domain.
lands of the public domain. These provisions also empowered the Government reclaimed and marshy lands of the public domain, being
Governor-General to classify further such disposable lands of the public neither timber nor mineral lands, fell under the classification of public
domain into government reclaimed, foreshore or marshy lands of the public agricultural lands. 50 However, government reclaimed and marshy lands,
domain, as well as other non-agricultural lands. although subject to classification as disposable public agricultural lands,
could only be leased and not sold to private parties because of Act No.
Section 58 of Act No. 2874 categorically mandated that 2874.
disposable lands of the public domain classified as government reclaimed,
foreshore and marshy lands "shall be disposed of to private parties by lease The prohibition on private parties from acquiring ownership of
only and not otherwise." The Governor-General, before allowing the lease government reclaimed and marshy lands of the public domain was only a
of these lands to private parties, must formally declare that the lands were statutory prohibition and the legislature could therefore remove such
"not necessary for the public service." Act No. 2874 reiterated the State prohibition. The 1935 Constitution did not prohibit individuals and
policy to lease and not to sell government reclaimed, foreshore and marshy corporations from acquiring government reclaimed and marshy lands of the
lands of the public domain, a policy first enunciated in 1907 in Act No. 1654. public domain that were classified as agricultural lands under existing public
Government reclaimed, foreshore and marshy lands remained sui land laws. Section 2, Article XIII of the 1935 Constitution provided as
generis,as the only alienable or disposable lands of the public domain that follows:
the government could not sell to private parties.
"Section 2. No private corporation or
The rationale behind this State policy is obvious. Government association may acquire, lease, or hold public
reclaimed, foreshore and marshy public lands for non-agricultural purposes agricultural lands in excess of one thousand and
retain their inherent potential as areas for public service. This is the reason twenty four hectares, nor may any individual
the government prohibited the sale, and only allowed the lease, of these acquire such lands by purchase in excess of one
lands to private parties. The State always reserved these lands for some hundred and forty hectares,or by lease in excess
future public service. of one thousand and twenty-four hectares, or by
homestead in excess of twenty-four hectares.
Act No. 2874 did not authorize the reclassification of government Lands adapted to grazing, not exceeding two
reclaimed, foreshore and marshy lands into other non-agricultural lands thousand hectares, may be leased to an individual,
under Section 56 (d).Lands falling under Section 56 (d) were the only lands private corporation, or association." (Italics
for non-agricultural purposes the government could sell to private supplied)
parties. Thus, under Act No. 2874, the government could not sell
government reclaimed, foreshore and marshy lands to private Still, after the effectivity of the 1935 Constitution, the legislature did
parties, unless the legislature passed a law allowing their sale. 49 not repeal Section 58 of Act No. 2874 to open for sale to private
parties government reclaimed and marshy lands of the public
Act No. 2874 did not prohibit private parties from reclaiming parts domain. On the contrary, the legislature continued the long
of the sea pursuant to Section 5 of the Spanish Law of Waters of 1866. established State policy of retaining for the government title and
Lands reclaimed from the sea by private parties with government ownership of government reclaimed and marshy lands of the public
permission remained private lands. domain.
Dispositions under the 1935 Constitution Commonwealth Act No. 141 of the Philippine National Assembly
On May 14, 1935, the 1935 Constitution took effect upon its On November 7, 1936, the National Assembly
ratification by the Filipino people. The 1935 Constitution, in adopting the approved Commonwealth Act No. 141, also known as the Public Land Act,
Regalian doctrine, declared in Section 1, Article XIII, that — which compiled the then existing laws on lands of the public domain. CA
No. 141, as amended, remains to this day the existing general
"Section 1. All agricultural, timber, and law governing the classification and disposition of lands of the public
mineral lands of the public domain, waters, domain other than timber and mineral lands. 51
minerals, coal, petroleum, and other mineral oils,
all forces of potential energy and other natural Section 6 of CA No. 141 empowers the President to classify
resources of the Philippines belong to the State, lands of the public domain into "alienable or disposable" 52 lands of the
and their disposition, exploitation, development, or public domain, which prior to such classification are inalienable and outside
utilization shall be limited to citizens of the the commerce of man. Section 7 of CA No. 141 authorizes the President to
Philippines or to corporations or associations at "declare what lands are open to disposition or concession." Section 8 of CA
least sixty per centum of the capital of which is No. 141 states that the government can declare open for disposition or
owned by such citizens, subject to any existing concession only lands that are "officially delimited and classified." Sections
right, grant, lease, or concession at the time of the 6, 7 and 8 of CA No. 141 read as follows:
inauguration of the Government established under
this Constitution. Natural resources, with the "Sec. 6. The President, upon the
exception of public agricultural land, shall not be recommendation of the Secretary of Agriculture
alienated,and no license, concession, or lease for and Commerce, shall from time to time classify the
the exploitation, development, or utilization of any lands of the public domain into —
of the natural resources shall be granted for a
(a) Alienable or disposable,
period exceeding twenty-five years, renewable for
another twenty-five years, except as to water (b) Timber and
rights for irrigation, water supply, fisheries, or
industrial uses other than the development of (c) Mineral lands,
water power, in which cases beneficial use may be
and may at any time and in like manner
transfer such lands from one class to
another, 53 for the purpose of their administration the lease of such lands to private parties. The government could sell to
and disposition. private parties only lands falling under Section 59 (d) of CA No. 141, or
those lands for non-agricultural purposes not classified as government
"Sec. 7. For the purposes of the reclaimed, foreshore and marshy disposable lands of the public domain.
administration and disposition of alienable or Foreshore lands, however, became inalienable under the 1935
disposable public lands, the President, upon Constitution which only allowed the lease of these lands to qualified private
recommendation by the Secretary of Agriculture parties.
and Commerce, shall from time to time declare
what lands are open to disposition or Section 58 of CA No. 141 expressly states that disposable lands
concession under this Act. of the public domain intended for residential, commercial, industrial or other
productive purposes other than agricultural "shall be disposed of under the
Sec. 8. Only those lands shall be provisions of this chapter and not otherwise." Under Section 10 of CA No.
declared open to disposition or concession which 141, the term "disposition" includes lease of the land. Any disposition of
have been officially delimited and classified and, government reclaimed, foreshore and marshy disposable lands for non-
when practicable, surveyed, and which have not agricultural purposes must comply with Chapter IX, Title III of CA No.
been reserved for public or quasi-public uses,nor 141, 54 unless a subsequent law amended or repealed these provisions.
appropriated by the Government, nor in any
manner become private property, nor those on In his concurring opinion in the landmark case of Republic Real
which a private right authorized and recognized by Estate Corporation v. Court of Appeals, 55 Justice Reynato S. Puno
this Act or any other valid law may be claimed, or summarized succinctly the law on this matter, as follows:
which, having been reserved or appropriated,
have ceased to be so. ...." "Foreshore lands are lands of public
dominion intended for public use. So too are lands
Thus, before the government could alienate or dispose of lands of reclaimed by the government by dredging, filling,
the public domain, the President must first officially classify these or other means. Act 1654 mandated that the
lands as alienable or disposable, and then declare them open to control and disposition of the foreshore and lands
disposition or concession. There must be no law reserving these under water remained in the national government.
lands for public or quasi-public uses. Said law allowed only the 'leasing' of reclaimed
land. The Public Land Acts of 1919 and 1936 also
The salient provisions of CA No. 141, on government reclaimed,
declared that the foreshore and lands reclaimed by
foreshore and marshy lands of the public domain, are as follows:"
the government were to be "disposed of to private
"Sec. 58. Any tract of land of the public parties by lease only and not otherwise." Before
domain which, being neither timber nor mineral leasing, however, the Governor-General, upon
land, is intended to be used for residential recommendation of the Secretary of Agriculture
purposes or for commercial, industrial, or other and Natural Resources, had first to determine that
productive purposes other than agricultural, and is the land reclaimed was not necessary for the
open to disposition or concession, shall be public service. This requisite must have been met
disposed of under the provisions of this chapter before the land could be disposed of. But even
and not otherwise. then, the foreshore and lands under water were
not to be alienated and sold to private parties. The
Sec. 59. The lands disposable under disposition of the reclaimed land was only by
this title shall be classified as follows: lease. The land remained property of the
State."(Italics supplied)
(a) Lands reclaimed by the
Government by dredging, filling, or other means; As observed by Justice Puno in his concurring opinion,
"Commonwealth Act No. 141 has remained in effect at present."
(b) Foreshore;
The State policy prohibiting the sale to private parties of
(c) Marshy lands or lands covered with government reclaimed, foreshore and marshy alienable lands of the public
water bordering upon the shores or banks of domain, first implemented in 1907 was thus reaffirmed in CA No. 141 after
navigable lakes rivers; the 1935 Constitution took effect. The prohibition on the sale of foreshore
lands, however, became a constitutional edict under the 1935 Constitution,
(d) Lands not included in any of the Foreshore lands became inalienable as natural resources of the State,
foregoing classes. unless reclaimed by the government and classified as agricultural lands of
Sec. 60. Any tract of land comprised the public domain, in which case they would fall under the classification of
under this title may be leased or sold, as the case government reclaimed lands.
may be, to any person, corporation, or association After the effectivity of the 1935 Constitution, government
authorized to purchase or lease public lands for reclaimed and marshy disposable lands of the public domain continued to
agricultural purposes. .... be only leased and not sold to private parties. 56 These lands remained sui
Sec. 61. The lands comprised in generis,as the only alienable or disposable lands of the public domain the
classes (a),(b),and (c) of section fifty-nine shall be government could not sell to private parties.
disposed of to private parties by lease only and not Since then and until now, the only way the government can sell
otherwise,as soon as the President,upon to private parties government reclaimed and marshy disposable lands of the
recommendation by the Secretary of public domain is for the legislature to pass a law authorizing such sale. CA
Agriculture, shall declare that the same are not No. 141 does not authorize the President to reclassify government
necessary for the public service and are open to reclaimed and marshy lands into other non-agricultural lands under Section
disposition under this chapter. The lands included 59 (d).Lands classified under Section 59 (d) are the only alienable or
in class (d) may be disposed of by sale or lease disposable lands for non-agricultural purposes that the government could
under the provisions of this Act." (Italics supplied) sell to private parties.
Section 61 of CA No. 141 readopted, after the effectivity of Moreover, Section 60 of CA No. 141 expressly requires
the 1935 Constitution, Section 58 of Act No. 2874 prohibiting the sale of congressional authority before lands under Section 59 that the government
government reclaimed, foreshore and marshy disposable lands of the public previously transferred to government units or entities could be sold to
domain. All these lands are intended for residential, commercial, industrial private parties. Section 60 of CA No. 141 declares that —
or other non-agricultural purposes. As before, Section 61 allowed only
"Sec. 60. ...The area so leased or sold banks, shores, roadsteads, and others
shall be such as shall, in the judgment of the of similar character;
Secretary of Agriculture and Natural Resources,
be reasonably necessary for the purposes for (2) Those which belong to the State, without being
which such sale or lease is requested, and shall for public use, and are intended for
not exceed one hundred and forty-four hectares: some public service or for the
Provided, however, That this limitation shall not development of the national wealth.
apply to grants, donations, or transfers made to a
xxx xxx xxx.
province, municipality or branch or subdivision of
the Government for the purposes deemed by said Art. 422. Property of public dominion,
entities conducive to the public interest; but the when no longer intended for public use or for public
land so granted, donated, or transferred to a service, shall form part of the patrimonial property
province, municipality or branch or subdivision of of the State."
the Government shall not be alienated,
encumbered, or otherwise disposed of in a manner Again, the government must formally declare that the property of
affecting its title, except when authorized by public dominion is no longer needed for public use or public service, before
Congress:...." (Italics supplied) the same could be classified as patrimonial property of the State. 59 In the
case of government reclaimed and marshy lands of the public domain, the
The congressional authority required in Section 60 of CA No. declaration of their being disposable, as well as the manner of their
141 mirrors the legislative authority required in Section 56 of Act No. disposition, is governed by the applicable provisions of CA No. 141.
2874.
Like the Civil Code of 1889, the Civil Code of 1950 included as
One reason for the congressional authority is that Section 60 property of public dominion those properties of the State which, without
of CA No. 141 exempted government units and entities from the maximum being for public use, are intended for public service or the "development of
area of public lands that could be acquired from the State. These the national wealth." Thus, government reclaimed and marshy lands of the
government units and entities should not just turn around and sell these State, even if not employed for public use or public service, if developed to
lands to private parties in violation of constitutional or statutory limitations. enhance the national wealth, are classified as property of public dominion.
Otherwise, the transfer of lands for non-agricultural purposes to government
units and entities could be used to circumvent constitutional limitations on Dispositions under the 1973 Constitution
ownership of alienable or disposable lands of the public domain. In the same The 1973 Constitution, which took effect on January 17, 1973,
manner, such transfers could also be used to evade the statutory prohibition likewise adopted the Regalian doctrine. Section 8, Article XIV of the 1973
in CA No. 141 on the sale of government reclaimed and marshy lands of the Constitutionstated that —
public domain to private parties. Section 60 of CA No. 141 constitutes by
operation of law a lien on these lands. 57 "Sec. 8. All lands of the public domain,
waters, minerals, coal, petroleum and other
In case of sale or lease of disposable lands of the public domain mineral oils, all forces of potential energy,
falling under Section 59 of CA No. 141, Sections 63 and 67 require a public fisheries, wildlife, and other natural resources of
bidding. Sections 63 and 67 of CA No. 141 provide as follows: the Philippines belong to the State. With the
"Sec. 63. Whenever it is decided that exception of agricultural, industrial or commercial,
lands covered by this chapter are not needed for residential, and resettlement lands of the public
public purposes, the Director of Lands shall ask domain, natural resources shall not be
the Secretary of Agriculture and Commerce (now alienated, and no license, concession, or lease for
the Secretary of Natural Resources) for authority the exploration, development, exploitation, or
to dispose of the same. Upon receipt of such utilization of any of the natural resources shall be
authority, the Director of Lands shall give notice by granted for a period exceeding twenty-five years,
public advertisement in the same manner as in the renewable for not more than twenty-five years,
case of leases or sales of agricultural public land, except as to water rights for irrigation, water
... supply, fisheries, or industrial uses other than the
development of water power, in which cases,
Sec. 67. The lease or sale shall be beneficial use may be the measure and the limit of
made by oral bidding; and adjudication shall be the grant." (Italics supplied)
made to the highest bidder....." (Italics supplied)
The 1973 Constitution prohibited the alienation of all natural
Thus, CA No. 141 mandates the Government to put to public auction resources with the exception of "agricultural, industrial or commercial,
all leases or sales of alienable or disposable lands of the public residential, and resettlement lands of the public domain." In contrast,
domain. 58 the 1935 Constitution barred the alienation of all natural resources except
"public agricultural lands." However, the term "public agricultural lands" in
Like Act No. 1654 and Act No. 2874 before it, CA No. 141 did
the 1935 Constitution encompassed industrial, commercial, residential and
not repeal Section 5 of the Spanish Law of Waters of 1866. Private parties
resettlement lands of the public domain. 60 If the land of public domain were
could still reclaim portions of the sea with government permission. However,
neither timber nor mineral land, it would fall under the classification of
the reclaimed land could become private land only if classified as
agricultural land of the public domain.Both the 1935 and 1973
alienable agricultural land of the public domain open to disposition
Constitutions, therefore, prohibited the alienation of all natural resources
under CA No. 141. The 1935 Constitution prohibited the alienation of all
except agricultural lands of the public domain.
natural resources except public agricultural lands.
The 1973 Constitution, however, limited the alienation of lands
The Civil Code of 1950
of the public domain to individuals who were citizens of the Philippines.
The Civil Code of 1950 readopted substantially the definition of Private corporations, even if wholly owned by Philippine citizens, were no
property of public dominion found in the Civil Code of 1889. Articles 420 and longer allowed to acquire alienable lands of the public domain unlike in
422 of the Civil Code of 1950 state that — the 1935 Constitution. Section 11, Article XIV of the 1973
Constitution declared that —
"Art. 420. The following things are
property of public dominion: "Sec. 11. The Batasang Pambansa,
taking into account conservation, ecological, and
(1) Those intended for public use, such as roads, development requirements of the natural
canals, rivers, torrents, ports and resources, shall determine by law the size of land
bridges constructed by the State, of the public domain which may be developed,
held or acquired by, or leased to, any qualified PD No. 1084 authorizes PEA to reclaim both foreshore and
individual, corporation, or association, and the submerged areas of the public domain. Foreshore areas are those covered
conditions therefor. No private corporation or and uncovered by the ebb and flow of the tide. 61 Submerged areas are
association may hold alienable lands of the public those permanently under water regardless of the ebb and flow of the
domain except by lease not to exceed one tide. 62 Foreshore and submerged areas indisputably belong to the public
thousand hectares in area nor may any citizen hold domain 63 and are inalienable unless reclaimed, classified as alienable
such lands by lease in excess of five hundred lands open to disposition, and further declared no longer needed for public
hectares or acquire by purchase, homestead or service.
grant, in excess of twenty-four hectares. No
private corporation or association may hold by The ban in the 1973 Constitution on private corporations from
lease, concession, license or permit, timber or acquiring alienable lands of the public domain did not apply to PEA since it
forest lands and other timber or forest resources in was then, and until today, a fully owned government corporation. The
excess of one hundred thousand hectares. constitutional ban applied then, as it still applies now, only to "private
However, such area may be increased by the corporations and associations."PD No. 1084 expressly empowers PEA "to
Batasang Pambansa upon recommendation of the hold lands of the public domain" even "in excess of the area permitted to
National Economic and Development Authority." private corporations by statute." Thus, PEA can hold title to private lands,
(Italics supplied) as well as title to lands of the public domain.

Thus, under the 1973 Constitution, private corporations could In order for PEA to sell its reclaimed foreshore and submerged
hold alienable lands of the public domain only through lease. Only alienable lands of the public domain, there must be legislative authority
individuals could now acquire alienable lands of the public domain, empowering PEA to sell these lands. This legislative authority is necessary
and private corporations became absolutely barred from acquiring any kind in view of Section 60 of CA No. 141, which states —
of alienable land of the public domain. The constitutional ban extended to
"Sec. 60. ...;but the land so granted,
all kinds of alienable lands of the public domain, while the statutory ban
donated or transferred to a province, municipality,
under CA No. 141 applied only to government reclaimed, foreshore and
or branch or subdivision of the Government shall
marshy alienable lands of the public domain.
not be alienated, encumbered or otherwise
PD No. 1084 Creating the Public Estates Authority disposed of in a manner affecting its title, except
when authorized by Congress;...." (Italics
On February 4, 1977, then President Ferdinand Marcos supplied)
issued Presidential Decree No. 1084 creating PEA, a wholly government
owned and controlled corporation with a special charter. Sections 4 and 8
of PD No. 1084, vests PEA with the following purposes and powers:
Without such legislative authority, PEA could not sell but only lease
"Sec. 4. Purpose.The Authority is its reclaimed foreshore and submerged alienable lands of the public
hereby created for the following purposes: domain. Nevertheless, any legislative authority granted to PEA to sell
its reclaimed alienable lands of the public domain would be subject
(a) To reclaim land, including foreshore to the constitutional ban on private corporations from acquiring
and submerged areas, by dredging, filling or other alienable lands of the public domain. Hence, such legislative
means, or to acquire reclaimed land; authority could only benefit private individuals.
(b) To develop, improve, acquire, Dispositions under the 1987 Constitution
administer, deal in, subdivide, dispose, lease and
The 1987 Constitution, like the 1935 and 1973
sell any and all kinds of lands,buildings, estates
Constitutions before it, has adopted the Regalian doctrine. The 1987
and other forms of real property, owned, managed,
Constitution declares that all natural resources are "owned by the
controlled and/or operated by the government;
State," and except for alienable agricultural lands of the public domain,
(c) To provide for, operate or natural resources cannot be alienated. Sections 2 and 3, Article XII of the
administer such service as may be necessary for 1987 Constitution state that —
the efficient, economical and beneficial utilization
"Section 2. All lands of the public
of the above properties.
domain, waters, minerals, coal, petroleum and
Sec. 5. Powers and functions of the other mineral oils, all forces of potential energy,
Authority. The Authority shall, in carrying out the fisheries, forests or timber, wildlife, flora and
purposes for which it is created, have the following fauna, and other natural resources are owned by
powers and functions: the State. With the exception of agricultural lands,
all other natural resources shall not be
(a) To prescribe its by-laws. alienated.The exploration, development, and
utilization of natural resources shall be under the
xxx xxx xxx full control and supervision of the State. ....
(i) To hold lands of the Section 3. Lands of the public domain
public domain in excess of the area are classified into agricultural, forest or timber,
permitted to private corporations by mineral lands, and national parks. Agricultural
statute. lands of the public domain may be further
(j) To reclaim lands and to classified by law according to the uses which they
construct work across, or otherwise, may be devoted. Alienable lands of the public
any stream, watercourse, canal, ditch, domain shall be limited to agricultural lands.
flume .... Private corporations or associations may not hold
such alienable lands of the public domain except
xxx xxx xxx by lease, for a period not exceeding twenty-five
years, renewable for not more than twenty-five
(o) To perform such acts years, and not to exceed one thousand hectares
and exercise such functions as may be in area.Citizens of the Philippines may lease not
necessary for the attainment of the more than five hundred hectares, or acquire not
purposes and objectives herein more than twelve hectares thereof by purchase,
specified." (Italics supplied) homestead, or grant.
Taking into account the requirements in preventing the break-up of farmlands. If the farmland is registered in the
of conservation, ecology, and development, and name of a corporation, upon the death of the owner, his heirs would inherit
subject to the requirements of agrarian reform, the shares in the corporation instead of subdivided parcels of the farmland. This
Congress shall determine, by law, the size of lands would prevent the continuing break-up of farmlands into smaller and smaller
of the public domain which may be acquired, plots from one generation to the next.
developed, held, or leased and the conditions
therefor." (Italics supplied) In actual practice, the constitutional ban strengthens the
constitutional limitation on individuals from acquiring more than the allowed
The 1987 Constitution continues the State policy in the 1973 area of alienable lands of the public domain. Without the constitutional ban,
Constitution banning private corporations from acquiring any kind of individuals who already acquired the maximum area of alienable lands of
alienable land of the public domain. Like the 1973 Constitution, the 1987 the public domain could easily set up corporations to acquire more alienable
Constitution allows private corporations to hold alienable lands of the public public lands. An individual could own as many corporations as his means
domain only through lease. As in the 1935 and 1973 Constitutions, the would allow him. An individual could even hide his ownership of a
general law governing the lease to private corporations of reclaimed, corporation by putting his nominees as stockholders of the corporation. The
foreshore and marshy alienable lands of the public domain is still CA No. corporation is a convenient vehicle to circumvent the constitutional limitation
141. on acquisition by individuals of alienable lands of the public domain.
The Rationale behind the Constitutional Ban The constitutional intent, under the 1973 and 1987
Constitutions, is to transfer ownership of only a limited area of alienable land
The rationale behind the constitutional ban on corporations from
of the public domain to a qualified individual. This constitutional intent is
acquiring, except through lease, alienable lands of the public domain is not
safeguarded by the provision prohibiting corporations from acquiring
well understood. During the deliberations of the 1986 Constitutional
alienable lands of the public domain, since the vehicle to circumvent the
Commission, the commissioners probed the rationale behind this ban, thus:
constitutional intent is removed. The available alienable public lands are
"FR. BERNAS: Mr. Vice-President, my gradually decreasing in the face of an ever-growing population. The most
questions have reference to page 3, line 5 which effective way to insure faithful adherence to this constitutional intent is to
says: grant or sell alienable lands of the public domain only to individuals. This, it
would seem, is the practical benefit arising from the constitutional ban.
'No private corporation or
association may hold alienable lands of The Amended Joint Venture Agreement
the public domain except by lease, not The subject matter of the Amended JVA, as stated in its second
to exceed one thousand hectares in Whereas clause, consists of three properties, namely:
area.'
1. "[T]hree partially reclaimed and substantially
If we recall, this provision did not exist eroded islands along Emilio Aguinaldo
under the 1935 Constitution, but this was Boulevard in Parañaque and Las
introduced in the 1973 Constitution. In effect, it Piñas, Metro Manila, with a combined
prohibits private corporations from acquiring titled area of 1,578,441 square meters;"
alienable public lands. But it has not been very
clear in jurisprudence what the reason for this is.In 2. "[A]nother area of 2,421,559 square meters
some of the cases decided in 1982 and 1983, it contiguous to the three islands;" and
was indicated that the purpose of this is to prevent
large landholdings.Is that the intent of this 3. "[A]t AMARI's option as approved by PEA, an
provision? additional 350 hectares more or less to
regularize the configuration of the
MR. VILLEGAS: I think that is the spirit reclaimed area." 65
of the provision.
PEA confirms that the Amended JVA involves "the development of
FR. BERNAS: In existing decisions the Freedom Islands and further reclamation of about 250 hectares .
involving the Iglesia ni Cristo, there were instances . . ," plus an option "granted to AMARI to subsequently reclaim
where the Iglesia ni Cristo was not allowed to another 350 hectares . . . ." 66
acquire a mere 313-square meter land where a
In short, the Amended JVA covers a reclamation area of 750
chapel stood because the Supreme Court said it
hectares. Only 157.84 hectares of the 750-hectare reclamation project have
would be in violation of this." (Italics supplied)
been reclaimed, and the rest of the 592.15 hectares are still submerged
In Ayog v. Cusi, 64 the Court explained the rationale behind this areas forming part of Manila Bay.
constitutional ban in this way:
Under the Amended JVA, AMARI will reimburse PEA the sum of
"Indeed, one purpose of the P1,894,129,200.00 for PEA's "actual cost" in partially reclaiming the
constitutional prohibition against purchases of Freedom Islands. AMARI will also complete, at its own expense, the
public agricultural lands by private corporations is reclamation of the Freedom Islands. AMARI will further shoulder all the
to equitably diffuse land ownership or to reclamation costs of all the other areas, totaling 592.15 hectares, still to be
encourage 'owner-cultivatorship and the economic reclaimed. AMARI and PEA will share, in the proportion of 70 percent and
family-size farm' and to prevent a recurrence of 30 percent, respectively, the total net usable area which is defined in the
cases like the instant case. Huge landholdings by Amended JVA as the total reclaimed area less 30 percent earmarked for
corporations or private persons had spawned common areas. Title to AMARI's share in the net usable area, totaling 367.5
social unrest." hectares, will be issued in the name of AMARI. Section 5.2 (c) of the
Amended JVA provides that —
However, if the constitutional intent is to prevent huge landholdings,
the Constitution could have simply limited the size of alienable lands "...,PEA shall have the duty to execute
of the public domain that corporations could acquire. without delay the necessary deed of transfer or
The Constitution could have followed the limitations on individuals, conveyance of the title pertaining to AMARI's Land
who could acquire not more than 24 hectares of alienable lands of share based on the Land Allocation Plan. PEA,
the public domain under the 1973 Constitution, and not more than 12 when requested in writing by AMARI, shall then
hectares under the 1987 Constitution. cause the issuance and delivery of the proper
certificates of title covering AMARI's Land Share in
If the constitutional intent is to encourage economic family-size the name of AMARI,...;provided, that if more than
farms, placing the land in the name of a corporation would be more effective seventy percent (70%) of the titled area at any
given time pertains to AMARI, PEA shall deliver to "D. Conclusion
AMARI only seventy percent (70%) of the titles
pertaining to AMARI, until such time when a Reclaimed lands are lands of the public
corresponding proportionate area of additional domain. However, by statutory authority, the rights
land pertaining to PEA has been titled." (Italics of ownership and disposition over reclaimed lands
supplied) have been transferred to PEA, by virtue of which
PEA, as owner, may validly convey the same to
Indisputably, under the Amended JVA AMARI will acquire and own a any qualified person without violating
maximum of 367.5 hectares of reclaimed land which will be titled in the Constitution or any statute.
its name.
The constitutional provision prohibiting
To implement the Amended JVA, PEA delegated to the private corporations from holding public land,
unincorporated PEA-AMARI joint venture PEA's statutory authority, rights except by lease (Sec. 3, Art. XVII, 70 1987
and privileges to reclaim foreshore and submerged areas in Manila Bay. Constitution), does not apply to reclaimed lands
Section 3.2.a of the Amended JVA states that — whose ownership has passed on to PEA by
statutory grant."
"PEA hereby contributes to the joint
venture its rights and privileges to perform Under Section 2, Article XII of the 1987 Constitution, the
Rawland Reclamation and Horizontal foreshore and submerged areas of Manila Bay are part of the "lands of the
Development as well as own the Reclamation public domain, waters . . . and other natural resources" and consequently
Area, thereby granting the Joint Venture the full "owned by the State." As such, foreshore and submerged areas "shall not
and exclusive right, authority and privilege to be alienated," unless they are classified as "agricultural lands" of the public
undertake the Project in accordance with the domain. The mere reclamation of these areas by PEA does not convert
Master Development Plan." these inalienable natural resources of the State into alienable or disposable
lands of the public domain. There must be a law or presidential proclamation
The Amended JVA is the product of a renegotiation of the original
officially classifying these reclaimed lands as alienable or disposable and
JVA dated April 25, 1995 and its supplemental agreement dated
open to disposition or concession. Moreover, these reclaimed lands cannot
August 9, 1995.
be classified as alienable or disposable if the law has reserved them for
The Threshold Issue some public or quasi-public use. 71
The threshold issue is whether AMARI, a private corporation, Section 8 of CA No. 141 provides that "only those lands shall be
can acquire and own under the Amended JVA 367.5 hectares of reclaimed declared open to disposition or concession which have been officially
foreshore and submerged areas in Manila Bay in view of Sections 2 and 3, delimited and classified." 72 The President has the authority to classify
Article XII of the 1987 Constitution which state that: inalienable lands of the public domain into alienable or disposable lands of
the public domain, pursuant to Section 6 of CA No. 141. In Laurel vs.
"Section 2. All lands of the public
Garcia, 73 the Executive Department attempted to sell the Roppongi
domain, waters, minerals, coal, petroleum, and
property in Tokyo, Japan, which was acquired by the Philippine Government
other mineral oils, all forces of potential energy,
for use as the Chancery of the Philippine Embassy. Although the Chancery
fisheries, forests or timber, wildlife, flora and
had transferred to another location thirteen years earlier, the Court still ruled
fauna, and other natural resources are owned by
that, under Article 422 74 of the Civil Code, a property of public dominion
the State. With the exception of agricultural lands,
retains such character until formally declaredotherwise. The Court ruled that
all other natural resources shall not be alienated.

....
"The fact that the Roppongi site has not been
xxx xxx xxx
used for a long time for actual Embassy service
Section 3. ...Alienable lands of the does not automatically convert it to patrimonial
public domain shall be limited to agricultural property. Any such conversion happens only if the
lands. Private corporations or associations may property is withdrawn from public use (Cebu
not hold such alienable lands of the public domain Oxygen and Acetylene Co. v. Bercilles,66 SCRA
except by lease,...." (Italics supplied) 481 [1975].A property continues to be part of the
public domain, not available for private
Classification of Reclaimed Foreshore and Submerged Areas appropriation or ownership 'until there is a formal
PEA readily concedes that lands reclaimed from foreshore or declaration on the part of the government to
submerged areas of Manila Bay are alienable or disposable lands of the withdraw it from being such' (Ignacio v. Director of
public domain, In its Memorandum, 67 PEA admits that — Lands,108 Phil. 335 [1960]." (Italics supplied)

"Under the Public Land Act (CA 141, as PD No. 1085, issued on February 4, 1977, authorized the
amended), reclaimed lands are classified as issuance of special land patents for lands reclaimed by PEA from the
alienable and disposable lands of the public foreshore or submerged areas of Manila Bay. On January 19, 1988 then
domain: President Corazon C. Aquino issued Special Patent No. 3517 in the name
of PEA for the 157.84 hectares comprising the partially reclaimed Freedom
'Sec. 59. The lands Islands. Subsequently, on April 9, 1999 the Register of Deeds of the
disposable under this title shall be Municipality of Parañaque issued TCT Nos. 7309, 7311 and 7312 in the
classified as follows: name of PEA pursuant to Section 103 of PD No. 1529 authorizing the
issuance of certificates of title corresponding to land patents. To this day,
(a) Lands reclaimed by the government these certificates of title are still in the name of PEA.
by dredging, filling, or other
means; PD No. 1085, coupled with President Aquino's actual
issuance of a special patent covering the Freedom Islands, is equivalent to
xxx xxx xxx.'" (Italics supplied) an official proclamation classifying the Freedom Islands as alienable or
disposable lands of the public domain. PD No. 1085 and President Aquino's
Likewise, the Legal Task Force 68 constituted under issuance of a land patent also constitute a declaration that the Freedom
Presidential Administrative Order No. 365 admitted in its Report and Islands are no longer needed for public service. The Freedom Islands are
Recommendation to then President Fidel V. Ramos, "[R]eclaimed lands are thus alienable or disposable lands of the public domain, open to disposition
classified as alienable and disposable lands of the public domain." 69 The or concession to qualified parties.
Legal Task Force concluded that —
At the time then President Aquino issued Special Patent No. undertaken only by the National Government or by a person
3517, PEA had already reclaimed the Freedom Islands although contracted by the National Government. Private parties may reclaim
subsequently there were partial erosion on some areas. The government from the sea only under a contract with the National Government,
had also completed the necessary surveys on these islands. Thus, the and no longer by grant or permission as provided in Section 5 of the
Freedom Islands were no longer part of Manila Bay but part of the land Spanish Law of Waters of 1866.
mass. Section 3, Article XII of the 1987 Constitution classifies lands of the
Executive Order No. 525, issued on February 14, 1979,
public domain into "agricultural, forest or timber, mineral lands, and national
designated PEA as the National Government's implementing arm to
parks." Being neither timber, mineral, nor national park lands, the reclaimed
undertake "all reclamation projects of the government," which "shall be
Freedom Islands necessarily fall under the classification of agricultural
undertaken by the PEA or through a proper contract executed by it with any
lands of the public domain. Under the 1987 Constitution, agricultural lands
person or entity." Under such contract, a private party receives
of the public domain are the only natural resources that the State may
compensation for reclamation services rendered to PEA. Payment to the
alienate to qualified private parties. All other natural resources, such as the
contractor may be in cash, or in kind consisting of portions of the reclaimed
seas or bays, are "waters . . . owned by the State" forming part of the public
land, subject to the constitutional ban on private corporations from acquiring
domain, and are inalienable pursuant to Section 2, Article XII of the 1987
alienable lands of the public domain. The reclaimed land can be used as
Constitution.
payment in kind only if the reclaimed land is first classified as alienable or
AMARI claims that the Freedom Islands are private lands disposable land open to disposition, and then declared no longer needed
because CDCP, then a private corporation, reclaimed the islands under a for public service.
contract dated November 20, 1973 with the Commissioner of Public
The Amended JVA covers not only the Freedom Islands, but also
Highways. AMARI, citing Article 5 of the Spanish Law of Waters of 1866,
an additional 592.15 hectares which are still submerged and forming part of
argues that "if the ownership of reclaimed lands may be given to the party
Manila Bay.There is no legislative or Presidential act classifying these
constructing the works, then it cannot be said that reclaimed lands are lands
submerged areas as alienable or disposable lands of the public domain
of the public domain which the State may not alienate." 75 Article 5 of the
open to disposition.These submerged areas are not covered by any patent
Spanish Law of Waters reads as follows:
or certificate of title. There can be no dispute that these submerged areas
"Article 5. Lands reclaimed from the form part of the public domain, and in their present state are inalienable and
sea in consequence of works constructed by the outside the commerce of man. Until reclaimed from the sea, these
State, or by the provinces, pueblos or private submerged areas are, under the Constitution, "waters . . . owned by the
persons, with proper permission shall become the State," forming part of the public domain and consequently inalienable. Only
property of the party constructing such when actually reclaimed from the sea can these submerged areas be
works, unless otherwise provided by the terms of classified as public agricultural lands, which under the Constitution are the
the grant of authority." (Italics supplied) only natural resources that the State may alienate. Once reclaimed and
transformed into public agricultural lands, the government may then
Under Article 5 of the Spanish Law of Waters of 1866, private officially classify these lands as alienable or disposable lands open to
parties could reclaim from the sea only with "proper permission" from the disposition. Thereafter, the government may declare these lands no longer
State. Private parties could own the reclaimed land only if not "otherwise needed for public service. Only then can these reclaimed lands be
provided by the terms of the grant of authority." This clearly meant that no considered alienable or disposable lands of the public domain and within
one could reclaim from the sea without permission from the State because the commerce of man.
the sea is property of public dominion. It also meant that the State could
grant or withhold ownership of the reclaimed land because any reclaimed The classification of PEA's reclaimed foreshore and submerged
land, like the sea from which it emerged, belonged to the State. Thus, a lands into alienable or disposable lands open to disposition is necessary
private person reclaiming from the sea without permission from the State because PEA is tasked under its charter to undertake public services that
could not acquire ownership of the reclaimed land which would remain require the use of lands of the public domain. Under Section 5 of PD No.
property of public dominion like the sea it replaced.76 Article 5 of the 1084, the functions of PEA include the following: "[T]o own or operate
Spanish Law of Waters of 1866 adopted the time-honored principle of land railroads, tramways and other kinds of land transportation, . . . ; [T]o
ownership that "all lands that were not acquired from the government, either construct, maintain and operate such systems of sanitary sewers as may
by purchase or by grant, belong to the public domain." 77 be necessary; [T]o construct, maintain and operate such storm drains as
may be necessary." PEA is empowered to issue "rules and regulations as
Article 5 of the Spanish Law of Waters must be read together may be necessary for the proper use by private parties of any or all of the
with laws subsequently enacted on the disposition of public lands. In highways, roads, utilities, buildings and/or any of its properties and to
particular, CA No. 141requires that lands of the public domain must first be impose or collect fees or tolls for their use." Thus, part of the reclaimed
classified as alienable or disposable before the government can alienate foreshore and submerged lands held by the PEA would actually be needed
them. These land must not be reserved for public or quasi-public for public use or service since many of the functions imposed on PEA by its
purposes. 78 Moreover, the contract between CDCP and the government charter constitute essential public services.
was executed after the effectivity of the 1973 Constitution which barred
private corporations from acquiring any kind of alienable land of the public Moreover, Section 1 of Executive Order No. 525 provides that
domain. This contract could not have converted the Freedom Islands into PEA "shall be primarily responsible for integrating, directing, and
private lands of a private corporation. coordinating all reclamation projects and on behalf of the National
Government." The same section also states that "[A]ll reclamation projects
Presidential Decree No. 3-A, issued on January 11, 1973, shall be approved by the President upon the recommendation of the PEA,
revoked all laws authorizing the reclamation of areas under water and and shall be undertaken by the PEA or through a proper contract executed
revested solely in the National Government the power to reclaim lands. by it with any person or entity; . . . ." Thus, under EO No. 525, in relation
Section 1 of PD No. 3-A declared that — to PD No. 3-A and PD No. 1084, PEA became the primary implementing
agency of the National Government to reclaim foreshore and submerged
"The provisions of any law to the lands of the public domain. EO No. 525 recognized PEA as the entity "to
contrary notwithstanding,the reclamation of areas undertake the reclamation of lands and ensure their maximum utilization in
under water, whether foreshore or inland, shall promoting public welfare and interests." 79 Since large portions of these
be limited to the National Government or any reclaimed lands would obviously be needed for public service, there must
person authorized by it under a proper be a formal declaration segregating reclaimed lands no longer needed for
contract.(Italics supplied) public service from those still needed for public service.

Section 3 of EO No. 525, by declaring that all lands reclaimed by


xxx xxx xxx." PEA "shall belong to or be owned by the PEA," could not automatically
operate to classify inalienable lands into alienable or disposable lands of the
PD No. 3-A repealed Section 5 of the Spanish Law of Waters of public domain. Otherwise, reclaimed foreshore and submerged lands of the
1866 because reclamation of areas under water could now be
public domains would automatically become alienable once reclaimed by Clearly, the mere physical act of reclamation by PEA of
PEA, whether or not classified as alienable or disposable. foreshore or submerged areas does not make the reclaimed lands alienable
or disposable lands of the public domain, much less patrimonial lands of
The Revised Administrative Code of 1987, a later law than PEA. Likewise, the mere transfer by the National Government of lands of
either PD No. 1084 or EO No. 525, vests in the Department of Environment the public domain to PEA does not make the lands alienable or disposable
and Natural Resources ("DENR" for brevity) the following powers and lands of the public domain, much less patrimonial lands of PEA.
functions:
Absent two official acts — a classification that these lands are
"Sec. 4. Powers and Functions. The alienable or disposable and open to disposition and a declaration that these
Department shall: lands are not needed for public service, lands reclaimed by PEA remain
inalienable lands of the public domain. Only such an official classification
(1) ...
and formal declaration can convert reclaimed lands into alienable or
xxx xxx xxx disposable lands of the public domain, open to disposition under
the Constitution, Title I and Title III 83 of CA No. 141and other applicable
(4) Exercise supervision and laws. 84
control over forest lands, alienable and disposable
public lands,mineral resources and, in the process PEA's Authority to Sell Reclaimed Lands
of exercising such control, impose appropriate PEA, like the Legal Task Force, argues that as alienable or
taxes, fees, charges, rentals and any such form of disposable lands of the public domain, the reclaimed lands shall be
levy and collect such revenues for the exploration, disposed of in accordance with CA No. 141, the Public Land Act. PEA, citing
development, utilization or gathering of such Section 60 of CA No. 141, admits that reclaimed lands transferred to a
resources; branch or subdivision of the government "shall not be alienated,
xxx xxx xxx encumbered, or otherwise disposed of in a manner affecting its title, except
when authorized by Congress: . . . ." 85(Emphasis by PEA)
(14) Promulgate rules, regulations and
guidelines on the issuance of licenses, permits, In Laurel vs. Garcia, 86 the Court cited Section 48 of the
concessions, lease agreements and such other Revised Administrative Code of 1987, which states that —
privileges concerning the development, "Sec. 48. Official Authorized to Convey
exploration and utilization of the country's marine, Real Property. Whenever real property of the
freshwater, and brackish water and over all Government is authorized by law to be
aquatic resources of the country and shall conveyed,the deed of conveyance shall be
continue to oversee, supervise and police our executed in behalf of the government by the
natural resources;cancel or cause to cancel such following: ...."
privileges upon failure, non-compliance or
violations of any regulation, order, and for all other Thus, the Court concluded that a law is needed to convey any real
causes which are in furtherance of the property belonging to the Government. The Court declared that —
conservation of natural resources and supportive
"It is not for the President to convey
of the national interest;
real property of the government on his or her own
(15) Exercise exclusive jurisdiction on sole will. Any such conveyance must be
the management and disposition of all lands of the authorized and approved by a law enacted by the
public domain and serve as the sole agency Congress.It requires executive and legislative
responsible for classification,sub-classification, concurrence." (Italics supplied)
surveying and titling of lands in consultation with
PEA contends that PD No. 1085 and EO No. 525 constitute the
appropriate agencies." 80 (Italics supplied)
legislative authority allowing PEA to sell its reclaimed lands. PD No. 1085,
As manager, conservator and overseer of the natural resources issued on February 4, 1977, provides that —
of the State, DENR exercises "supervision and control over alienable and
"The land reclaimed in the foreshore
disposable public lands." DENR also exercises "exclusive jurisdiction on the
and offshore area of Manila Bay pursuant to the
management and disposition of all lands of the public domain." Thus, DENR
contract for the reclamation and construction of the
decides whether areas under water, like foreshore or submerged areas of
Manila-Cavite Coastal Project between the
Manila Bay, should be reclaimed or not. This means that PEA needs
Republic of the Philippines and the Construction
authorization from DENR before PEA can undertake reclamation projects in
and Development Corporation of the Philippines
Manila Bay, or in any part of the country.
dated November 20, 1973 and/or any other
DENR also exercises exclusive jurisdiction over the disposition contract or reclamation covering the same area is
of all lands of the public domain. Hence, DENR decides whether reclaimed hereby transferred, conveyed and assigned to the
lands of PEA should be classified as alienable under Sections 6 81 and ownership and administration of the Public Estates
7 82 of CA No. 141. Once DENR decides that the reclaimed lands should Authority established pursuant to PD No. 1084;
be so classified, it then recommends to the President the issuance of a Provided, however, That the rights and interests of
proclamation classifying the lands as alienable or disposable lands of the the Construction and Development Corporation of
public domain open to disposition. We note that then DENR Secretary the Philippines pursuant to the aforesaid contract
Fulgencio S. Factoran, Jr. countersigned Special Patent No. 3517 in shall be recognized and respected.
compliance with the Revised Administrative Code and Sections 6 and 7
Henceforth, the Public Estates
of CA No. 141.
Authority shall exercise the rights and assume the
In short, DENR is vested with the power to authorize the obligations of the Republic of the Philippines
reclamation of areas under water, while PEA is vested with the power to (Department of Public Highways) arising from, or
undertake the physical reclamation of areas under water, whether directly incident to, the aforesaid contract between the
or through private contractors. DENR is also empowered to classify lands Republic of the Philippines and the Construction
of the public domain into alienable or disposable lands subject to the and Development Corporation of the Philippines.
approval of the President. On the other hand, PEA is tasked to develop, sell
In consideration of the foregoing
or lease the reclaimed alienable lands of the public domain.
transfer and assignment, the Public Estates
Authority shall issue in favor of the Republic of the
Philippines the corresponding shares of stock in
said entity with an issued value of said shares of 63 and 67 of CA No. 141 requiring public auction, in the absence of a law
stock (which) shall be deemed fully paid and non- exempting PEA from holding a public auction. 88 Special Patent No. 3517
assessable. expressly states that the patent is issued by authority of
the Constitution and PD No. 1084, "supplemented by Commonwealth Act
The Secretary of Public Highways and No. 141, as amended." This is an acknowledgment that the provisions of CA
the General Manager of the Public Estates No. 141 apply to the disposition of reclaimed alienable lands of the public
Authority shall execute such contacts or domain unless otherwise provided by law. Executive Order No.
agreements with the Construction and 654,89 which authorizes PEA "to determine the kind and manner of
Development Corporation of the Philippines, as payment for the transfer" of its assets and properties, does not exempt PEA
may be necessary to implement the above. from the requirement of public auction. EO No. 654 merely authorizes PEA
to decide the mode of payment, whether in kind and in installment, but does
Special land patent/patents shall be
not authorize PEA to dispense with public auction.
issued by the Secretary of Natural Resources in
favor of the Public Estates Authority without Moreover, under Section 79 of PD No. 1445, otherwise known
prejudice to the subsequent transfer to the as the Government Auditing Code, the government is required to sell
contractor or his assignees of such portion or valuable government property through public bidding. Section 79 of PD No.
portions of the land reclaimed or to be reclaimed 1445 mandates that —
as provided for in the above-mentioned contract.
On the basis of such patents, the Land "Section 79. When government
Registration Commission shall issue the property has become unserviceable for any cause,
corresponding certificate of title." (Italics supplied) or is no longer needed,it shall, upon application of
the officer accountable therefor, be inspected by
On the other hand, Section 3 of EO No. 525, issued on February the head of the agency or his duly authorized
14, 1979, provides that — representative in the presence of the auditor
concerned and, if found to be valueless or
"Sec. 3. All lands reclaimed by PEA
unsaleable, it may be destroyed in their
shall belong to or be owned by the PEA which shall
presence. If found to be valuable, it may be sold at
be responsible for its administration, development,
public auction to the highest bidder under the
utilization or disposition in accordance with the
supervision of the proper committee on award or
provisions of Presidential Decree No. 1084. Any
similar body in the presence of the auditor
and all income that the PEA may derive from the
concerned or other authorized representative of
sale, lease or use of reclaimed lands shall be used
the Commission, after advertising by printed
in accordance with the provisions of Presidential
notice in the Official Gazette, or for not less than
Decree No. 1084."
three consecutive days in any newspaper of
There is no express authority under either PD No. 1085 or EO general circulation, or where the value of the
No. 525 for PEA to sell its reclaimed lands. PD No. 1085 merely transferred property does not warrant the expense of
"ownership and administration" of lands reclaimed from Manila Bay to PEA, publication, by notices posted for a like period in at
while EO No. 525 declared that lands reclaimed by PEA "shall belong to or least three public places in the locality where the
be owned by PEA."EO No. 525 expressly states that PEA should dispose property is to be sold. In the event that the public
of its reclaimed lands "in accordance with the provisions of Presidential auction fails, the property may be sold at a private
Decree No. 1084," the charter of PEA. sale at such price as may be fixed by the same
committee or body concerned and approved by
PEA's charter, however, expressly tasks PEA "to develop, the Commission."
improve, acquire, administer, deal in, subdivide, dispose, lease and sell any
and all kinds of lands . . . owned, managed, controlled and/or operated by It is only when the public auction fails that a negotiated sale is
the government." 87 (Italics supplied) There is, therefore, legislative allowed, in which case the Commission on Audit must approve the
authority granted to PEA to sell its lands, whether patrimonial or alienable selling price. 90 The Commission on Audit implements Section 79 of
lands of the public domain.PEA may sell to private parties its patrimonial the Government Auditing Code through Circular No. 89-296 91 dated
properties in accordance with the PEA charter free from constitutional January 27, 1989. This circular emphasizes that government assets
limitations. The constitutional ban on private corporations from acquiring must be disposed of only through public auction, and a negotiated
alienable lands of the public domain does not apply to the sale of PEA's sale can be resorted to only in case of "failure of public auction."
patrimonial lands. At the public auction sale, only Philippine citizens are
PEA may also sell its alienable or disposable lands of the public qualified to bid for PEA's reclaimed foreshore and submerged
domain to private individuals since, with the legislative authority, there is no alienable lands of the public domain. Private corporations are barred
longer any statutory prohibition against such sales and the constitutional from bidding at the auction sale of any kind of alienable land of the
ban does not apply to individuals. PEA, however, cannot sell any of its public domain.
alienable or disposable lands of the public domain to private corporations PEA originally scheduled a public bidding for the Freedom
since Section 3, Article XII of the 1987 Constitution expressly prohibits such Islands on December 10, 1991. PEA imposed a condition that the
sales. The legislative authority benefits only individuals. Private winning bidder should reclaim another 250 hectares of submerged
corporations remain barred from acquiring any kind of alienable land of the areas to regularize the shape of the Freedom Islands, under a 60-40
public domain, including government reclaimed lands. sharing of the additional reclaimed areas in favor of the winning
bidder. 92 No one, however, submitted a bid. On December 23,
The provision in PD No. 1085 stating that portions of the 1994, the Government Corporate Counsel advised PEA it could sell
reclaimed lands could be transferred by PEA to the "contractor the Freedom Islands through negotiation, without need of another
or his assignees" (Italics supplied) would not apply to private corporations public bidding, because of the failure of the public bidding on
but only to individuals because of the constitutional ban. Otherwise, the December 10, 1991. 93
provisions of PD No. 1085 would violate both the 1973 and 1987
Constitutions. However, the original JVA dated April 25, 1995 covered
not only the Freedom Islands and the additional 250 hectares still to
The requirement of public auction in the sale of reclaimed lands be reclaimed, it also granted an option to AMARI to reclaim another
Assuming the reclaimed lands of PEA are classified as alienable 350 hectares. The original JVA, a negotiated contract, enlarged the
or disposable lands open to disposition, and further declared no longer reclamation area to 750 hectares. 94 The failure of public bidding on
needed for public service, PEA would have to conduct a public bidding in December 10, 1991, involving only 407.84 hectares, 95 is not a valid
selling or leasing these lands. PEA must observe the provisions of Sections justification for a negotiated sale of 750 hectares, almost double the
area publicly auctioned. Besides, the failure of public bidding In support of their theory, PEA and AMARI cite the following rulings
happened on December 10, 1991, more than three years before the of the Court:
signing of the original JVA on April 25, 1995. The economic situation
1. Sumail v. Judge of CFI of Cotabato, 97 where
in the country had greatly improved during the intervening period.
the Court held —
"Once the patent was
Reclamation under the BOT Law and the Local Government Code granted and the corresponding
The constitutional prohibition in Section 3, Article XII of the certificate of title was issued, the land
1987 Constitution is absolute and clear: "Private corporations or ceased to be part of the public domain
associations may not hold such alienable lands of the public domain and became private property over
except by lease, . . . ." Even Republic Act No. 6957 ("BOT Law," for which the Director of Lands has neither
brevity), cited by PEA and AMARI as legislative authority to sell control nor jurisdiction."
reclaimed lands to private parties, recognizes the constitutional ban.
2. Lee Hong Hok v. David, 98 where the Court
Section 6 of RA No. 6957 states —
declared —
"Sec. 6. Repayment Scheme.— For
the financing, construction, operation and "After the registration and
maintenance of any infrastructure projects issuance of the certificate and duplicate
undertaken through the build-operate-and-transfer certificate of title based on a public land
arrangement or any of its variations pursuant to the patent, the land covered thereby
provisions of this Act, the project proponent ...may automatically comes under the
likewise be repaid in the form of a share in the operation of Republic Act 496 subject
revenue of the project or other non-monetary to all the safeguards provided therein."
payments, such as, but not limited to, the grant of
3. Heirs of Gregorio Tengco v. Heirs of Jose
a portion or percentage of the reclaimed
Aliwalas, 99 where the Court ruled —
land, subject to the constitutional requirements
with respect to the ownership of the land:...." "While the Director of Lands
(Italics supplied) has the power to review homestead
patents, he may do so only so long as
A private corporation, even one that undertakes the physical
the land remains part of the public
reclamation of a government BOT project, cannot acquire reclaimed
domain and continues to be under his
alienable lands of the public domain in view of the constitutional ban.
exclusive control; but once the patent is
Section 302 of the Local Government Code, also registered and a certificate of title is
mentioned by PEA and AMARI, authorizes local governments in land issued, the land ceases to be part of the
reclamation projects to pay the contractor or developer in kind public domain and becomes private
consisting of a percentage of the reclaimed land, to wit: property over which the Director of
Lands has neither control nor
"Section 302. Financing, Construction,
jurisdiction."
Maintenance, Operation, and Management of
Infrastructure Projects by the Private Sector. ... 4. Manalo v. Intermediate Appellate
Court, 100 where the Court held —
xxx xxx xxx
"When the lots in dispute
In case of land reclamation or
were certified as disposable on May 19,
construction of industrial estates, the repayment
1971, and free patents were issued
plan may consist of the grant of a portion or
covering the same in favor of the
percentage of the reclaimed land or the industrial
private respondents, the said lots
estate constructed."
ceased to be part of the public domain
Although Section 302 of the Local Government Code does not and, therefore, the Director of Lands
contain a proviso similar to that of the BOT Law, the constitutional lost jurisdiction over the same."
restrictions on land ownership automatically apply even though not
5. Republic v. Court of Appeals, 101 where the
expressly mentioned in the Local Government Code.
Court stated —
Thus, under either the BOT Law or the Local Government
Code, the contractor or developer, if a corporate entity, can only be "Proclamation No. 350,
paid with leaseholds on portions of the reclaimed land. If the dated October 9, 1956, of President
contractor or developer is an individual, portions of the reclaimed Magsaysay legally effected a land
land, not exceeding 12 hectares 96 of non-agricultural lands, may be grant to the Mindanao Medical Center,
conveyed to him in ownership in view of the legislative authority Bureau of Medical Services,
allowing such conveyance. This is the only way these provisions of Department of Health, of the whole lot,
the BOT Law and the Local Government Code can avoid a direct validly sufficient for initial registration
collision with Section 3, Article XII of the 1987 Constitution. under the Land Registration Act. Such
land grant is constitutive of a 'fee
Registration of lands of the public domain simple' title or absolute title in favor of
Finally, PEA theorizes that the "act of conveying the petitioner Mindanao Medical Center.
ownership of the reclaimed lands to public respondent PEA Thus, Section 122 of the Act, which
transformed such lands of the public domain to private lands." This governs the registration of grants or
theory is echoed by AMARI which maintains that the "issuance of the patents involving public lands, provides
special patent leading to the eventual issuance of title takes the that 'Whenever public lands in the
subject land away from the land of public domain and converts the Philippine Islands belonging to the
property into patrimonial or private property." In short, PEA and Government of the United States or to
AMARI contend that with the issuance of Special Patent No. 3517 the Government of the Philippines are
and the corresponding certificates of titles, the 157.84 hectares alienated, granted or conveyed to
comprising the Freedom Islands have become private lands of PEA. persons or to public or private
corporations, the same shall be
brought forthwith under the operation of alienable lands of the public domain must be transferred to qualified private
this Act (Land Registration Act, Act parties, or to government entities not tasked to dispose of public lands,
496) and shall become registered before these lands can become private or patrimonial lands. Otherwise, the
lands." constitutional ban will become illusory if Congress can declare lands of the
public domain as private or patrimonial lands in the hands of a government
The first four cases cited involve petitions to cancel the land agency tasked to dispose of public lands. This will allow private corporations
patents and the corresponding certificates of titles issued to private parties. to acquire directly from government agencies limitless areas of lands which,
These four cases uniformly hold that the Director of Lands has no prior to such law, are concededly public lands.
jurisdiction over private lands or that upon issuance of the certificate of title
the land automatically comes under the Torrens System. The fifth case cited Under EO No. 525, PEA became the central implementing
involves the registration under the Torrens System of a 12.8-hectare public agency of the National Government to reclaim foreshore and submerged
land granted by the National Government to Mindanao Medical Center, a areas of the public domain. Thus, EO No. 525 declares that —
government unit under the Department of Health. The National Government
transferred the 12.8-hectare public land to serve as the site for the hospital "EXECUTIVE ORDER NO. 525
buildings and other facilities of Mindanao Medical Center, which performed
Designating the Public Estates Authority as the
a public service. The Court affirmed the registration of the 12.8-hectare
Agency Primarily Responsible for all Reclamation
public land in the name of Mindanao Medical Center under Section 122
Projects
of Act No. 496. This fifth case is an example of a public land being registered
under Act No. 496 without the land losing its character as a property of Whereas, there are several
public dominion. reclamation projects which are ongoing or being
proposed to be undertaken in various parts of the
In the instant case, the only patent and certificates of title issued
country which need to be evaluated for
are those in the name of PEA, a wholly government owned corporation
consistency with national programs;
performing public as well as proprietary functions. No patent or certificate of
title has been issued to any private party. No one is asking the Director of Whereas, there is a need to give further
Lands to cancel PEA's patent or certificates of title. In fact, the thrust of the institutional support to the Government's declared
instant petition is that PEA's certificates of title should remain with PEA, and policy to provide for a coordinated, economical
the land covered by these certificates, being alienable lands of the public and efficient reclamation of lands;
domain, should not be sold to a private corporation.
Whereas, Presidential Decree No. 3-
Registration of land under Act No. 496 or PD No. 1529 does not A requires that all reclamation of areas shall be
vest in the registrant private or public ownership of the land. Registration is limited to the National Government or any person
not a mode of acquiring ownership but is merely evidence of ownership authorized by it under proper contract;
previously conferred by any of the recognized modes of acquiring
ownership. Registration does not give the registrant a better right than what Whereas, a central authority is needed
the registrant had prior to the registration. 102 The registration of lands of to act on behalf of the National Government which
the public domain under the Torrens system, by itself, cannot convert public shall ensure a coordinated and integrated
lands into private lands. 103 approach in the reclamation of lands;

Jurisprudence holding that upon the grant of the patent or Whereas, Presidential Decree No.
issuance of the certificate of title the alienable land of the public domain 1084 creates the Public Estates Authority as a
automatically becomes private land cannot apply to government units and government corporation to undertake reclamation
entities like PEA. The transfer of the Freedom Islands to PEA was made of lands and ensure their maximum utilization in
subject to the provisions of CA No. 141 as expressly stated in Special promoting public welfare and interests; and
Patent No. 3517 issued by then President Aquino, to wit:
Whereas, Presidential Decree No.
"NOW, THEREFORE, KNOW YE, that 1416 provides the President with continuing
by authority of the Constitution of the authority to reorganize the national government
Philippines and in conformity with the provisions including the transfer, abolition, or merger of
of Presidential Decree No. 1084, supplemented functions and offices.
by Commonwealth Act No. 141, as amended,there
are hereby granted and conveyed unto the Public NOW, THEREFORE, I, FERDINAND
Estates Authority the aforesaid tracts of land E. MARCOS, President of the Philippines, by
containing a total area of one million nine hundred virtue of the powers vested in me by
fifteen thousand eight hundred ninety four the Constitution and pursuant toPresidential
(1,915,894) square meters; the technical Decree No. 1416, do hereby order and direct the
description of which are hereto attached and made following:
an integral part hereof." (Italics supplied) Section 1. The Public Estates Authority
Thus, the provisions of CA No. 141 apply to the Freedom Islands (PEA) shall be primarily responsible for
on matters not covered by PD No. 1084. Section 60 of CA No. 141 prohibits, integrating, directing, and coordinating all
"except when authorized by Congress," the sale of alienable lands of the reclamation projects for and on behalf of the
public domain that are transferred to government units or entities. Section National Government.All reclamation projects
60 of CA No. 141constitutes, under Section 44 of PD No. 1529, a "statutory shall be approved by the President upon
lien affecting title" of the registered land even if not annotated on the recommendation of the PEA, and shall be
certificate of title. 104 Alienable lands of the public domain held by undertaken by the PEA or through a proper
government entitles under Section 60 of CA No. 141 remain public lands contract executed by it with any person or entity;
because they cannot be alienated or encumbered unless Congress passes Provided, that, reclamation projects of any national
a law authorizing their disposition. Congress, however, cannot authorize the government agency or entity authorized under its
sale to private corporations of reclaimed alienable lands of the public charter shall be undertaken in consultation with the
domain because of the constitutional ban. Only individuals can benefit from PEA upon approval of the President.
such law. xxx xxx xxx."
The grant of legislative authority to sell public lands in As the central implementing agency tasked to undertake
accordance with Section 60 of CA No. 141 does not automatically convert reclamation projects nationwide, with authority to sell reclaimed lands, PEA
alienable lands of the public domain into private or patrimonial lands. The took the place of DENR as the government agency charged with leasing or
selling reclaimed lands of the public domain. The reclaimed lands being otherwise disposed of in a manner affecting its title, except when authorized
leased or sold by PEA are not private lands, in the same manner that DENR, by Congress." This provision refers to government reclaimed, foreshore and
when it disposes of other alienable lands does not dispose of private lands marshy lands of the public domain that have been titled but still cannot be
but alienable lands of the public domain. Only when qualified private parties alienated or encumbered unless expressly authorized by Congress. The
acquire these lands will the lands become private lands. In the hands of the need for legislative authority prevents the registered land of the public
government agency tasked and authorized to dispose of alienable of domain from becoming private land that can be disposed of to qualified
disposable lands of the public domain, these lands are still public, not private parties.
private lands.
The Revised Administrative Code of 1987 also recognizes that
Furthermore, PEA's charter expressly states that PEA "shall hold lands of the public domain may be registered under the Torrens System.
lands of the public domain" as well as "any and all kinds of lands." PEA can Section 48, Chapter 12, Book I of the Code states —
hold both lands of the public domain and private lands. Thus, the mere fact
that alienable lands of the public domain like the Freedom Islands are "Sec. 48 Official Authorized to Convey
transferred to PEA and issued land patents or certificates of title in PEA's Real Property. Whenever real property of the
name does not automatically make such lands private. government is authorized by law to be conveyed,
the deed of conveyance shall be executed in
To allow vast areas of reclaimed lands of the public domain to behalf of the government by the following:
be transferred to PEA as private lands will sanction a gross violation of the
constitutional ban on private corporations from acquiring any kind of (1) ...
alienable land of the public domain. PEA will simply turn around, as PEA
(2) For property belonging
has now done under the Amended JVA, and transfer several hundreds of
to the Republic of the Philippines, but
hectares of these reclaimed and still to be reclaimed lands to a single private
titled in the name of any political
corporation in only one transaction. This scheme will effectively nullify the
subdivision or of any corporate agency
constitutional ban in Section 3, Article XII of the 1987 Constitution which
or instrumentality,by the executive
was intended to diffuse equitably the ownership of alienable lands of the
head of the agency or instrumentality."
public domain among Filipinos, now numbering over 80 million strong.
(Italics supplied)
This scheme, if allowed, can even be applied to alienable
Thus, private property purchased by the National Government for
agricultural lands of the public domain since PEA can "acquire . . . any and
expansion of a public wharf may be titled in the name of a
all kinds of lands." This will open the floodgates to corporations and even
government corporation regulating port operations in the country.
individuals acquiring hundreds of hectares of alienable lands of the public
Private property purchased by the National Government for
domain under the guise that in the hands of PEA these lands are private
expansion of an airport may also be titled in the name of the
lands. This will result in corporations amassing huge landholdings never
government agency tasked to administer the airport. Private property
before seen in this country — creating the very evil that the constitutional
donated to a municipality for use as a town plaza or public school
ban was designed to prevent. This will completely reverse the clear direction
site may likewise be titled in the name of the municipality. 106 All
of constitutional development in this country. The1935 Constitution allowed
these properties become properties of the public domain, and if
private corporations to acquire not more than 1,024 hectares of public
already registered under Act No. 496 or PD No. 1529, remain
lands. 105 The 1973 Constitution prohibited private corporations from
registered land. There is no requirement or provision in any existing
acquiring any kind of public land, and the 1987 Constitution has
law for the de-registration of land from the Torrens System.
unequivocally reiterated this prohibition.
Private lands taken by the Government for public use under its
The contention of PEA and AMARI that public lands, once own power of eminent domain become unquestionably part of the public
registered under Act No. 496 or PD No. 1529, automatically become private domain. Nevertheless, Section 85 of PD No. 1529 authorizes the Register
lands is contrary to existing laws. Several laws authorize lands of the public of Deeds to issue in the name of the National government new certificates
domain to be registered under the Torrens System or Act No. 496, now PD of title covering such expropriated lands. Section 85 of PD No. 1529 states
No. 1529, without losing their character as public lands. Section 122 of Act —
No. 496, and Section 103 of PD No. 1529, respectively, provide as follows:
"Sec. 85 Land taken by eminent
Act No. 496 domain. Whenever any registered land, or interest
therein, is expropriated or taken by eminent
"Sec. 122. Whenever public lands in
domain, the National Government, province, city
the Philippine Islands belonging to the
or municipality, or any other agency or
...Government of the Philippine Islands are
instrumentality exercising such right shall file for
alienated, granted, or conveyed to persons or
registration in the proper Registry a certified copy
the public or private corporations,the same shall
of the judgment which shall state definitely by an
be brought forthwith under the operation of this Act
adequate description, the particular property or
and shall become registered lands."
interest expropriated, the number of certificate of
PD No. 1529 title, and the nature of the public use. A
memorandum of the right or interest taken shall be
"Sec. 103. Certificate of Title to made on each certificate of title by the Register of
Patents. Whenever public land is by the Deeds, and where the fee simple is taken, a new
Government alienated, granted or conveyed certificate shall be issued in favor of the National
to any person,the same shall be brought forthwith Government, province, city, municipality,or any
under the operation of this Decree." (Italics other agency or instrumentality exercising such
supplied) right for the land so taken. The legal expenses
incident to the memorandum of registration or
Based on its legislative history, the phrase "conveyed to any person" issuance of a new certificate of title shall be for the
in Section 103 of PD No. 1529 includes conveyances of public lands account of the authority taking the land or interest
to public corporations. therein." (Italics supplied)
Alienable lands of the public domain "granted, donated, or
Consequently, lands registered under Act No. 496 or PD No.
transferred to a province, municipality, or branch or subdivision of the
1529 are not exclusively private or patrimonial lands. Lands of the
Government," as provided in Section 60 of CA No. 141, may be registered
public domain may also be registered pursuant to existing laws.
under the Torrens System pursuant to Section 103 of PD No. 1529. Such
registration, however, is expressly subject to the condition in Section 60
of CA No. 141 that the land "shall not be alienated, encumbered or
AMARI makes a parting shot that the Amended JVA is not a sale prohibits private corporations from
to AMARI of the Freedom Islands or of the lands to be reclaimed from acquiring any kind of alienable land of
submerged areas of Manila Bay. In the words of AMARI, the Amended JVA the public domain.
"is not a sale but a joint venture with a stipulation for reimbursement of the
original cost incurred by PEA for the earlier reclamation and construction 4. Since the Amended JVA also seeks to transfer
works performed by the CDCP under its 1973 contract with the Republic." to AMARI ownership of 290.156
Whether the Amended JVA is a sale or a joint venture, the fact remains that hectares 111 of still submerged areas
the Amended JVA requires PEA to "cause the issuance and delivery of the of Manila Bay, such transfer is void for
certificates of title conveying AMARI's Land Share on the name of being contrary to Section 2, Article XII
AMARI." 107 of the 1987 Constitution which
prohibits the alienation of natural
This stipulation still contravenes Section 3, Article XII of the 1987 resources other than agricultural lands
Constitution which provides that private corporations "shall not hold such of the public domain. PEA may reclaim
alienable lands of the public domain except by lease." the transfer of title these submerged areas. Thereafter,
and ownership to AMARI clearly means that AMARI will "hold' the reclaimed the government can classify the
lands other than by lease. The transfer of title and ownership is a reclaimed lands as alienable or
"disposition" of the reclaimed lands, a transaction considered a sale or disposable, and further declare them
alienation under CA No. 141, 108 theGovernment Auditing Code, 109 and no longer needed for public service.
Section 3, Article XII of the 1987 Constitution. Still, the transfer of such reclaimed
alienable lands of the public domain to
The Regalian doctrine is deeply implanted in our legal system. AMARI will be void in view of Section 3,
Foreshore and submerged areas form part of the public domain and are Article XII of the 1987
inalienable. Lands reclaimed from foreshore and submerged areas also Constitution which prohibits private
form part of the public domain and are also inalienable, unless converted corporations from acquiring any kind of
pursuant to law into alienable or disposable lands of the public domain. alienable land of the public domain.
Historically, lands reclaimed by the government are sui generis, not
available for sale to private parties unlike other alienable public lands. Clearly the Amended JVA violates glaringly Sections 2 and 3, Article
Reclaimed lands retain their inherent potential as areas for public use or XII of the 1987 Constitution. under Article 1409 112 of the Civil Code,
public service. Alienable lands of the public domain, increasingly becoming contracts whose "object or purpose is contrary to law," or whose
scarce natural resources, are to be distributed equitably among our ever- "object is outside the commerce of men," are "inexistent and void
growing population. To insure such equitable distribution, the 1973 from the beginning." The Court must perform its duty to defend and
and 1987 Constitutions have barred private corporations from acquiring any uphold the Constitution, and therefore declares the Amended
kind of alienable land of the public domain. Those who attempt to dispose JVA null and void ab initio. EcICDT
of inalienable natural resources of the State, or seek to circumvent the
Seventh issue: whether the Court is the proper forum to raise the
conditional ban on alienation of lands of the public domain to private
issue of whether the Amended JVA is grossly disadvantageous to
corporations, do so at their own risks.
the government.
We can now summarize our conclusions as follows; Considering that the Amended JVA is null and void ab
initio,there is no necessity to rule on this last issue. Besides, the Court is
1. The 157.84 hectares of reclaimed lands
not the trier of facts, and this last issue involves a determination of factual
comprising the Freedom Islands, now
matters.
covered by certificates of title in the
name of PEA, arealienable lands of the WHEREFORE, the petition is GRANTED. The Public Estates
public domain. PEA may lease these Authority and Amari Coastal Bay Development Corporation are
lands to private corporations but may PERMANENTLY ENJOINED from implementing the Amended Joint
not sell or transfer ownership of these Venture Agreement which is hereby declared NULL and VOID ab
lands to private corporations. PEA may initio. HSIaAT
only sell these lands to Philippine
citizens, subject to the ownership SO ORDERED.
limitations in the 1987 Constitution and
existing laws. ||| (Chavez v. Public Estates Authority, G.R. No. 133250, [July 9, 2002],
433 PHIL 506-592)
2. The 592.15 hectares of submerged areas of
Manila Bay remain inalienable natural
resources of the public domain until
classified as alienable or disposable
lands open to disposition and declared
no longer needed for public service.
The government can make such
classification and declaration only after
PEA has reclaimed these submerged
areas. Only then can these lands
qualify as agricultural lands of the
public domain, which are the only
natural resources the government can
alienate. In their present state, the
592.15 hectares of submerged areas
are inalienable and outside the
commerce of man.
3. Since the Amended JVA seeks to transfer to
AMARI, a private corporation,
ownership of 77.34 hectares 110 of the
Freedom Islands, such transfer is void
for being contrary to Section 3, Article
XII of the 1987 Constitution which
SECOND DIVISION contained a total area of 100,034 sq. m. This plan was approved on July 12,
1961 by the Acting Director of Lands.

[G.R. No. 117734. February 22, 2001.] On November 23, 1968, Tax Declaration No. 12927 which
cancelled Tax Declaration No. 9032 was secured by Gamos and declared
therein that the area of the consolidated property was 10.0034 hectares with
VICENTE G. DIVINA, petitioner, vs. HON. 2500 sq. m. planted to coconut, 3.8187 irrigated for rice planting and 5.9347
COURT OF APPEALS and VILMA GAJO- were thickets.
SY, respondents.
On January 19, 1967, Teotimo Berosa conveyed to Vicente G.
Divina, herein petitioner, a portion of Lot 1893 referred to as Lot 1893-B. It
is described as follows:
DECISION
"A parcel of dry and thicket land
situated in San Ignacio, Gubat, Sorsogon,
Philippines, having an area of 54,818 square
meters and bounded on the N., by Lot 1888
QUISUMBING, J p:
(Inocencio Eroe); Lot 1887 (Jaime Enaje); and Lot
1462 (Heirs of Zacarias Espadilla); on the
Before us is a petition for review of the decision 1 dated October (Illegible), by Lot 1466; on the SE, by Lot 1893-A;
27, 1994 of the Court of Appeals in CA-GR CV No. 03068 reversing and on the S., by Lot 1898 (Heirs of Silverio Garcia);
setting aside the judgment dated July 7, 1979 of the Court of First Instance on the E., by Lot 1892 (Antonio Escobedo) and Lot
of Sorsogon, Branch II, in LRC Case No. N-147. STcaDI 1890 (Eugenia Espedido); and on the NE, by Lot
The facts of this case are as follows: 1889 (Pedro Fajardo); all of Gubat Cadastre.
NOTE: This lot is designed as Lot 1893-B, a
Lot No. 1893 located at Gubat, Sorsogon, was originally owned portion of Lot 1893, Cad 308-D". 4
by Antonio Berosa. On July 22, 1960, he sold it to Teotimo Berosa. The
portion is particularly described as: On November 28, 1968, two years from the date of said sale and
five (5) days after November 23, 1968, when Gamos secured Tax
"A parcel of land unirrigated situated in Declaration No. 12927 declaring the consolidated property as containing
San Ignacio, Gubat, Sorsogon, Philippines, with 100,034 sq. m., the deed of sale was registered. An undated "Subdivision
an area of TWENTY THOUSAND (20,000) square PLAN" of Lot 1893, was prepared for petitioner. The plan, without Bureau
meters and bounded on the North by Lot #1464 — of Lands approval, showed that Lot 1893 was divided into two, Lot 1893-A
Fausto Ayson and Lot #1888 — Gloria Fajardo: on and Lot 1893-B.
the East, by Lot # 1446 — Silverio Garcia: on the
South, by Lot #1891 — Antonio Escobedo and on On July 24, 1970 Gamos sold the consolidated property to
the West, by Lot #1880 — Federico Faronas and private respondent Vilma Gajo-Sy, for P20,000.00. The land was
Lot #1890 — Eugenia Espedido. Cadastral particularly described as follows:
concrete posts are the visible signs of boundary. It "A parcel of land located at San
has no permanent improvement Ignacio, Gubat, Sorsogon, under Tax Declaration
thereon. Designated as Lot 1893 of Antonio No. 12927 in the name of Jose P. Gamos, covered
Berosa. Declared under Tax No. 13038, valued at by Lots No. 1466 and 1893 of the Gubat Cadastre,
P760.00 for the current year in the name of with an area of 100,034 sq. m., more or less . . .
ANTONIO BEROSA" 2 ." 5
On March 23, 1961, the Berosa spouses sold the same Lot 1893 On July 29, 1970, Tax Declaration No. 13768 secured by private
to Jose P. Gamos. In the deed of sale to Gamos, the lot was more respondent, was cancelled by Tax Declaration No. 12509.
particularly described as:
On August 28, 1972, she filed an application for registration of
"A parcel of RICE land situated in San title to the property at the then Court of First Instance of Sorsogon, docketed
Ignacio, Gubat, Sorsogon, Philippines, with an as LRC Case No. N-147, GLRO Record No. 42920. The application was
area of TWENTY THOUSAND (20,000) square amended on March 8, 1973, on order of Branch II of the said court "to
meters and bounded on the North, by Lot #1462 include therein the postal address of Inocencio Erpe, adjoining owner of Lot
— Fausto Ayson and Lot #1888 — Gloria F. No. 1893 described in Plan AP-9021". DcITaC
Estonante: on the East, by Lot #1464 — Zacarias
Espadilla; and Lot #1466 — Felix Arimado; on the The land registration court, by Decision of July 29, 1975, ordered
South, by Lot #1898 — Silverio Garcia; and on the the registration of private respondent's title over Lots Nos. 1466 and 1893.
West, by Lot #1890 — Eugenia Espedido and Lot
#1892 — Antonio Escobedo. Concrete cements On July 13, 1977, pending issuance of the final decree of
posts are the visible signs of boundary. No registration petitioner filed before the same court a Petition for Review of
permanent improvements thereon. Covered by Lot the July 29, 1975 judgment. He alleged that he is the owner of a portion of
#1893 of Teotimo E. Berosa, and declared under Lot 1893 consisting of 54,818 sq. m. conveyed to him by Teotimo Berosa
Tax No. 13039, valued at P760.00 for the present on January 19, 1967; that he was unaware of the registration proceedings
year in the name of TEOTIMO E. BEROSA." 3 on Lot 1893 due to private respondent's failure to give him notice and post
any notice in the subject lot; and that private respondent fraudulently
On April 26, 1960, Gamos acquired from the heirs of Felix misrepresented herself as the owner of the disputed portion despite her
Arimado, a boundary owner of Lot 1893, a 20,687 sq. m. parcel of land knowledge that another person had acquired the same.
identified as Lot 1466, also in Gubat. It adjoins Lot 1893. On March 28,
1961, Gamos had these two parcels of land under Tax Declaration No. Private respondent opposed the petition alleging that the
13237 and declared it had a total area of 4.0867 hectares. He also had the registration case had long become final and the court no longer had any
property resurveyed by private land surveyor Antonio Tiotangco. In 1967, jurisdiction thereon; and that lack of personal notice to the petitioner of the
Tax Declaration No. 13237 was cancelled by Tax Declaration No. 9032 in registration proceedings did not constitute actual fraud.
Gamos' name.
The trial court, in its Decision 6 dated June 7, 1979, found that
The re-survey plan (AP-9021), of Lots 1466 and 1893 conducted the petition for review was timely filed. It also ruled that the failure of private
on June 16, 1961 for Gamos, showed that the consolidated properties respondent to include a known claimant in her application for registration
constituted "deliberate misrepresentation that the lot subject of her errors. Suffice it to state that Lot 1893 bought by
application is not contested when in fact it was. Private respondent, Teotimo Berosa which he sold to Jose P. Gamos
according to the trial court, should have included in her application at least who in turn sold it to respondent-appellee in 1970
"the person of petitioner's cousin, Elena Domalaon" who had, before was designated by boundaries in such a manner
respondent filed her application for registration, made known to the latter's as to put its identity beyond doubt; that the total
sister her apprehension of "their land" being included in respondent's area of Lot 1893 lot was determined after a
application for registration. This misrepresentation, according to the court, resurvey/relocation was conducted for Gamos in
amounted to fraud within the contemplation of Section 38 of Act 496. 7 The 1961 the result of which is reflected in the
trial court in its decision disposed as follows: Plan approved by the Bureau of Lands also in
1961; that what really defines a piece of land is not
WHEREFORE, judgment is hereby the area but the boundaries therein laid down
rendered: (Peña, Registration of Land Titles and Deeds,
1988 Edition, p. 213); that the "Lot 1893-B" sold to
(a) Setting aside the Decision rendered
petitioner-appellee made no mention of any tax
in the land registration case and revoking the order
declaration covering it, unlike the different deeds
for the issuance of a Decree;
of sale covering Lot 1893, thereby raising the
(b) Declaring petitioner Vicente G. presumption that "Lot 1893-B" was really part
Divina the owner of the portion of the land applied of the Lot 1893 sold by Gamos to respondent-
for containing an area of 54,818 square meters appellant; and that the "subdivision" of Lot 1893
which is described in paragraph 3 of the Petition into Lots A and B, caused to be made by petitioner-
for Review of Judgment; and appellee who claims Lot 1893-B to have been the
"Lot 1893-B" subject of the January 19, 1967 Deed
(c) Allowing this land registration case of Sale in his favor which was registered on
to proceed as to the portion applied for which is November 28, 1968, appears to have been made
outside the limits of the portion herein awarded to in 1977, ten years from the date of said deed of
the petitioner Vicente G. Divina; and sale, a confirmation that there was no such "Lot
1893-B" subject of his purchase in 1967; and in
(d) Ordering a subdivision survey of the any event, as the subdivision survey prepared for
lots applied for, delimiting therein the area not petitioner-appellee was not approved by the
contested and which is registrable in favor of Bureau of Lands, it is not of much value (vide
applicant Vilma Gajo-Sy, and the area herein Flores vs. Director of Lands, 17 Phil. 512
adjudicated to petitioner Vicente G. Divina, as to [1910]). DTCSHA
whom the land registration proceedings shall
likewise be allowed to proceed after he shall have In fine, not all the basic elements for the
adduced such other evidence as are appropriate allowance of the reopening or review of the
in land registration cases. judgment rendered in the land registration case in
respondent-appellant's favor are present. The
SO ORDERED. 8 present appeal is thus meritorious.
Private respondent assailed the decision of the trial court before WHEREFORE, the assailed judgment
the Court of Appeals. It averred that the trial court erred (1) in declaring is hereby REVERSED and set aside and another
petitioner-appellee owner of a portion of Lot 1893, in ordering a subdivision rendered DISMISSING petitioner-appellee's
survey, and allowing petitioner-appellee to proceed with registration after petition at the court a quo. 10
adducing evidence as are appropriate; (2) in declaring respondent-appellant
guilty of "actual fraud" in the land registration case; (3) in taking cognizance Hence, the present petition. Petitioner now assails the reversal
of the petition for review of judgment, setting aside the decision dated June of the Court of Appeals of the trial court decision. In substance, he raises
29, 1975, and revoking the order of the issuance of the final decree in the the primary issue of whether or not, there was deliberate misrepresentation
land registration case; and (4) in not dismissing the petition for review of constituting actual fraud on private respondent's part when she failed to give
judgment with cost. 9 or post notice to petitioner of her application for registration of the contested
land, such that it was error for the trial court to declare private respondent
The CA reversed the trial court and dismissed the petition. It owner of the disputed land.
ruled:
Prefatorily, on the timeliness of the petition for review of
In the case at bar, petitioner-appellee judgment, we have repeatedly said that the adjudication in a registration of
did not indeed appear in the survey plan as an a cadastral case does not become final and incontrovertible until the
adjoining owner of the subject property. Neither expiration of one year after the entry of then final decree. As long as the
was he a known claimant or possessor of the final decree is not issued, and the one year within which it may be revised
questioned portion of Lot 1893 which was found by had not elapsed, the decision remains under the control and sound
the court a quo to be untouched and thickly discretion of the court rendering the decree, which court after hearing may
planted with bigaho. A fortiori, there was no need set aside the decision or decree or adjudicate the land to another
to mention in the application for registration the party. 11 In the present case, a certification was issued by the Land
apprehension or claim of "at least" petitioner- Registration Commission that no final decree of registration had yet been
appellee's cousin Evelyn (sic) Domalaon in the issued and by the order of the trial court dated September 28, 1977, it
application for registration, nor to personally notify restrained the Commission from issuing such a decree. Clearly, the tolling
Elena about registration proceeding. of the one year period has not even began. Thus, the trial court did not err
There could, therefore, have been no when it entertained the petition.
misrepresentation in any form on the part of Now, we consider the crux of the petition. Both the trial and
respondent-appellee. appellate courts found that petitioner's name did not appear in the survey
xxx xxx xxx plan as an adjacent owner, nor claimant nor possessor. However, the trial
and appellate courts differed in their conclusion on whether or not there was
There being no extrinsic or collateral deliberate misrepresentation constituting fraud in private respondent's part
fraud attendant to the registration of the property when it failed to give notice or post notice to potential claimant and include
in the name of respondent-appellee, We find it their names in the application for registration. The trial court said there was,
unnecessary to discuss the rest of the assigned but the appellate court disagreed.
Section 15 of P.D. 1529 12 is explicit in requiring that in the decree of registration shall not be opened, but shall remain
application for registration of land titles, the application "shall also state the in full force and effect forever, subject only to the right of
full names and addresses of all occupants of the land and those of the appeal herein before provided: Provided, however, That no
adjoining owners if known, and if not known, it shall state the extent of the decree or certificate of title issued to persons not parties to
search made to find them."As early as Francisco vs. Court of Appeals, 97 the appeal shall be cancelled or annulled. But any person
SCRA 22 [1980] we emphasized that a mere statement of the lack of aggrieved by such decree in any case may pursue his
knowledge of the names of the occupants and adjoining owners is not remedy by action for damages against the applicant or any
sufficient but "what search has been made to find them is necessary." The other person for fraud in procuring the decree. Whenever the
trial court was correct when it took notice that respondent's sister Lydia phrase "innocent purchaser for value" or an equivalent
Gajo-Añonuevo admitted that she had a conversation with petitioner's phrase occurs in this Act, it shall be deemed to include an
cousin Elena Dumalaon about the latter's apprehension that their land may innocent lessee, mortgagee, or other encumbrancer for
have been included in respondent's application for registration of the value. (As amended by Section 3, Act 3621; and Sec. 1, Act
disputed land. 13 Respondent's omission of this material information No. 3630). [Emphasis ours.]
prevented petitioner from having his day in court. The trial court in its
decision more than amply supported its conclusion with jurisprudence to the ||| (Divina v. Court of Appeals, G.R. No. 117734, [February 22, 2001], 405
effect that it is fraud to knowingly omit or conceal a fact upon which benefit PHIL 161-173)
is obtained to the prejudice of a third person. 14 Such omission can not but
be deliberate misrepresentation constituting fraud, a basis for allowing a
petition for review of judgment under Section 38 of Act No. 496, The Land
Registration Act.

Additionally, it should be noted that petitioner acquired the bigger


portion of Lot 1893 long after the initial survey of Barrio San Ignacio.
Teotimo Berosa sold Lot 1893 to Jose P. Gamos who in turn sold it to
respondent in 1970. Clearly, going by the records, petitioner's name would
not be found on the said survey plan approved by the Bureau of Lands in
1961, years before his purchase of the portion of Lot 1893. Petitioner's claim
is clearly meritorious. SHIcDT

WHEREFORE, the petition is GRANTED. The assailed decision


of the Court of Appeals dated October 24, 1994 is REVERSED and SET
ASIDE. The judgment in LRC Case No. N-147 of the then Court of First
Instance, Branch II in Gubat, Sorsogon is REINSTATED. Costs against
private respondent.
SO ORDERED.
Bellosillo, Mendoza, Buena and De Leon, Jr., JJ., concur.

Footnotes

1.Rollo, pp. 21-32.


2.Id. at 21-22; Record, p. 17.
3.Id. at 22 only.
4.Id. at 23-24.
5.Id. at 24 only.
6.Id. at 36-44.

7.Sec. 38. If the Court after hearing finds that the applicant or adverse
claimant has title as stated in his application or adverse
claim and proper for registration, a decree of confirmation
and registration shall be entered. Every decree of
registration shall bind the land, and quiet title thereto, subject
only to the exceptions stated in the following section. It shall
be conclusive upon and against all persons, including the
Insular Government and all the branches thereof, whether
mentioned by name in the application, notice, or citation, or
included in the general description "To all whom it may
concern." Such decree shall not be opened by reason of the
absence, infancy, or other disability of any person affected
thereby, nor by any proceeding in any court for reversing
judgments or decrees; subject, however, to the right of any
person deprived of land or of any estate or interest therein
by decree of registration obtained by fraud to file in the
competent Court of First Instance a petition for review within
one year after the entry of the decree, provided no innocent
purchaser for value has acquired an interest. Upon the
expiration of said term of one year, every decree or
certificate of title issued in accordance with this section shall
be incontrovertible. If there is any such purchaser, the
SECOND DIVISION Opposing the application, petitioner, through the Office of
the City Prosecutor of Las Piñas City, advanced that the lots sought
to be registered were inalienable lands of the public domain; that
[G.R. No. 175578. August 11, 2010.] neither respondents nor their predecessors-in-interest had been in
prior possession thereof; and that the muniment of title and the tax
declaration submitted to the court did not constitute competent and
REPUBLIC OF THE
sufficient evidence of bona fide acquisition or of prior possession in
PHILIPPINES, petitioner, vs. ZENAIDA GUINTO-
the concept of owner. 17
ALDANA, in her own behalf as Attorney-in-
fact of MA. AURORA GUINTO-COMISO, MA. At the hearing, Zenaida identified her herein co-
LUISA GUINTO-DIONISIO, ALFREDO GUINTO, respondents to be her siblings, nephews and nieces. She likewise
JR., PACITA R. GUINTO, ERNESTO R. identified the adjoining lot owners named in the application and the
GUINTO, NATIVIDAD R. GUINTO and supporting documents attached to the application as well. She testified
ALBERTO R. GUINTO, respondents. that the subject lots had been surveyed at the instance of her family
sometime between 1994 and 1995, and that said survey was
documented in Plan Ccs-007601-000040-D and in the geodetic
engineer's technical description of the lots. She implied that they did
DECISION obtain the original tracing cloth plan of the property, but it was
forwarded to the Land Registration Authority (LRA) by the Las Piñas
RTC in connection with the proceedings in LRC Case No. LP-128.
Notwithstanding this admission, and without objection from the
PERALTA, J p: oppositor, the blueprint of Plan Ccs-007601-000040-D and the
technical description of the property were provisionally marked in
In this petition for review under Rule 45 of the Rules of evidence. 18
Court, the Republic of the Philippines, through the Office of the Furthermore, Zenaida — 61 years old at the time of her
Solicitor General, assails the March 30, 2006 Decision 1 and the testimony — declared that she has known that the subject lots were
November 20, 2006 Resolution, 2 both of the Court of Appeals, in CA- owned by her family since she was 5 years old and from her earliest
G.R. CV No. 80500. The assailed decision reversed and set aside the recollection, she narrated that her grandparents had lived in the
July 10, 2003 judgment 3 of the Regional Trial Court of Las Piñas City, subject lots until the death of her grandmother in 1961. She implied
Branch 199 in LRC Case No. 02-0036, one for original registration of that aside from her predecessors there were other persons,
title, whereas the assailed Resolution denied reconsideration. TcIaHC caretakers supposedly, who had tilled the land and who had lived until
The facts follow. sometime between 1980 and 1990. She remembered her
grandmother having constructed a house on the property, but the
On April 3, 2002, respondents Zenaida Guinto- same had already been destroyed. Also, sometime in 1970, her family
Aldana 4 (Zenaida), Ma. Aurora Guinto-Comiso, Ma. Luisa Guinto- built an adobe fence around the perimeter of the lots and later, in the
Dionisio, Alfredo Guinto, Jr., Pacita R. Guinto, Ernesto R. Guinto, 1990s, they reinforced it with hollow blocks and concrete after an
Natividad R. Guinto and Alberto R. Guinto, filed with the Regional Trial inundation caused by the flood. 19 She claimed that she and her
Court (RTC) of Las Piñas City, Branch 199 an Application for father, Sergio, had been religious in the payment of real estate taxes
Registration of Title 5 over two pieces of land in Talango, Pamplona as shown by the tax declarations and tax receipts which she submitted
Uno, Las Piñas City. These lands, identified as Lot No. 4 and Lot No. to the court and which, following identification, were forthwith marked
5 in Conversion Consolidation Subdivision Plan Ccs-007601-000040- in evidence. 20
D, 6 measure 1,509 square meters and 4,640 square meters,
respectively. 7 Respondents professed themselves to be co-owners Zenaida's claim of prior, open, exclusive and continuous
of these lots, having acquired them by succession from their possession of the land was corroborated by Josefina Luna (Josefina),
predecessors Sergio Guinto (Sergio) and Lucia Rivera-Guinto (Lucia) one of the adjoining lot owners. Josefina, then 73 years old, strongly
— Zenaida's parents — who, in turn, had acquired the property under declared that the subject lots were owned by Zenaida's parents,
a 1969 document denominated as "Kasulatan sa Paghahati ng Lupa Sergio Guinto and Lucia Rivera, since she reached the age of
na Labas sa Hukuman na may Pagpaparaya at Bilihan." Under this understanding, and that she had not come to know of any instance
document, Sergio and Lucia Guinto acquired for a consideration the where a third party had placed a claim on the property. When asked
respective shares on the property of Pastor Guinto, Dionisio Guinto, whether there was anyone residing in the property and whether there
Potenciana Guinto and Marcelina Bernardo who, together with Luisa, were improvements made thereon, she said there was no one residing
had derived the same from Romualdo Guinto. 8 Respondents also therein and that there was nothing standing thereon except for
alleged that until the time of the application, they and their a nipa hut. 21 cCSDTI
predecessors-in-interest have been in actual, open, peaceful, At the close of Josefina's testimony, respondents formally
adverse, exclusive and continuous possession of these lots in the offered their exhibits without the oppositor placing any objection
concept of owner and that they had consistently declared the property thereto. 22 After weighing the evidence, the trial court, on July 10,
in their name for purposes of real estate taxation. 9 2003, rendered its Decision denying the application for registration. It
In support of their application, respondents submitted to found that respondents were unable to establish with certainty the
the court the blueprint of Plan Ccs-007601-000040-D, 10 as well as identity of the lots applied for registration, because of failure to submit
copies of the technical descriptions of each lot, 11 a certification from to the court the original tracing cloth plan as mandated by Presidential
the geodetic engineer 12 and the pertinent tax Decree (P.D.) No. 1529. It likewise noted that the fact of adverse,
declarations, 13 together with the receipts of payment continuous, open, public and peaceful possession in the concept of
therefor. 14 Expressly, they averred that the property's original tracing owner has not been proved by the evidence as Zenaida's and
cloth plan had previously been submitted to the RTC of Las Piñas City, Josefina's respective testimonies did not establish the nature of the
Branch 255 (Las Piñas RTC) in connection with the proceedings in possession of respondents' predecessors. 23 The dispositive portion
LRC Case No. LP-128 — a previous registration case involving the of the Decision reads:
subject property which, however, had been dismissed without WHEREFORE, for failure of the applicants to
prejudice. 15 comply with the requirements of Presidential
The trial court found the application to be sufficient in form Decree No. 1529, the Application for Original
and substance; hence, it gave due course thereto and ordered Registration of Title is hereby DENIED.
compliance with the publication and notification requirements of the
ORDERED. 24
law. 16
Aggrieved, respondents appealed to the Court of Appeals which, on March for land registration as it supplies the means by which to determine
30, 2006, issued the assailed Decision reversing the trial court as follows: the exact metes and bounds of the property. The applicant in that case
was unable to submit the original tracing cloth plan of the land he was
WHEREFORE, premises considered, the claiming because apparently, as in the present case, it was previously
assailed decision is hereby REVERSED and SET transmitted by the clerk of court to the LRA. Yet the Court, deeming it
ASIDE. Accordingly, the instant appeal is hereby the applicant's obligation to retrieve the plan himself and present it in
GRANTED. evidence, denied the application, to wit:
SO ORDERED. 25 The submission in evidence of the original tracing
cloth plan, duly approved by the Bureau of Lands,
Petitioner's motion for reconsideration was in cases for application of original registration of
denied. 26 Hence, it filed the instant petition which attributes error to land is a mandatory requirement. The reason for
the Court of Appeals in reversing the trial court's July 10, 2003 this rule is to establish the true identity of the land
decision. to ensure that it does not overlap a parcel of land
Petitioner principally posits that under Section 17 of P.D. or a portion thereof already covered by a previous
No. 1529, the submission in court of the original tracing cloth plan of land registration, and to forestall the possibility that
the property sought to be registered is a mandatory requirement in it will be overlapped by a subsequent registration
registration proceedings in order to establish the exact identity of the of any adjoining land. The failure to comply with
property. While respondents admitted that the original tracing cloth this requirement is fatal to petitioner's application
plan of Lot Nos. 4 and 5 in this case was in the custody of the LRA as for registration.
a consequence of their first attempt to have the property registered,
Petitioner contends, however, that he had
petitioner, invoking Del Rosario v. Republic of the
submitted the original tracing cloth plan to the
Philippines, 27 believes that respondents, on that score alone, are not
branch clerk of court, but the latter submitted the
relieved of their procedural obligation to adduce in evidence the
same to the LRA. This claim has no merit.
original copy of the plan, because they could have easily retrieved it
Petitioner is duty bound to retrieve the tracing cloth
from the LRA and presented it in court.28
plan from the LRA and to present it in evidence in
Furthermore, petitioner suggests that the blueprint of the the trial court. . . . 33
subdivision plan submitted by respondents cannot approximate
substantial compliance with the requirement of Section 17 of P.D. No. Yet if the reason for requiring an applicant to adduce in
1529. Again, relying on the aforementioned Del Rosario case, evidence the original tracing cloth plan is merely to provide a
petitioner observes that the blueprint in this case, allegedly illegible convenient and necessary means to afford certainty as to the exact
and unreadable, does not even bear the certification of the Lands identity of the property applied for registration and to ensure that the
Management Bureau. 29 Lastly, petitioner attacks respondents' claim same does not overlap with the boundaries of the adjoining lots, there
of prior possession. It notes that there is no clear and convincing stands to be no reason why a registration application must be denied
evidence that respondents and their predecessors-in-interest have for failure to present the original tracing cloth plan, especially where it
been in open, continuous, adverse, public and exclusive possession is accompanied by pieces of evidence — such as a duly executed
of Lot Nos. 4 and 5 for 30 years. 30 blueprint of the survey plan and a duly executed technical description
of the property — which may likewise substantially and with as much
Commenting on the petition, respondents observe that certainty prove the limits and extent of the property sought to be
petitioner's arguments are mere reiterative theses on the issues that registered.
have already been addressed by the Court of Appeals in the assailed
Decision and Resolution, and that there are no new matters raised Thus, sound is the doctrinal precept laid down in Republic
which have not yet been previously passed upon. Accordingly, they of the Philippines v. Court of Appeals, 34 and in the later cases
prayed that the petition be denied. 31 of Spouses Recto v. Republic of the Philippines 35 and Republic of
the Philippines v. Hubilla, 36 that while the best evidence to identify a
We find the petition to be unmeritorious. piece of land for registration purposes is the original tracing cloth plan
issued by the Bureau of Lands (now the Lands Management Services
Section 17 of P.D. No. 1529, otherwise known as The
of the Department of Environment and Natural Resources [DENR]),
Property Registration Decree of 1978, materially provides: DHITCc
blueprint copies and other evidence could also provide sufficient
Section 17. What and where to file. — The identification. Pertinently, the Court in Hubilla,citing Recto,
application for land registration shall be filed with pronounced:
the Court of First Instance of the province or city
While the petitioner correctly asserts that the
where the land is situated. The applicant shall file,
submission in evidence of the original tracing cloth
together with the application, all original
plan, duly approved by the Bureau of Lands, is a
muniments of titles or copies thereof and a survey
mandatory requirement, this Court has recognized
plan of the land approved by the Bureau of Lands.
instances of substantial compliance with this rule.
The clerk of court shall not accept any application In previous cases, this Court ruled that blueprint
unless it is shown that the applicant has furnished copies of the original tracing cloth plan from the
the Director of Lands with a copy of the application Bureau of Lands and other evidence could also
and all annexes. provide sufficient identification to identify a piece
of land for registration purposes. . . . 37
The provision denotes that it is imperative in an application
for original registration that the applicant submit to the court, aside In the case at bar, we find that the submission of the
from the original or duplicate copies of the muniments of title, a copy blueprint of Plan Ccs-007601-000040-D, together with the technical
of a duly approved survey plan of the land sought to be registered. description of the property, operates as substantial compliance with
The survey plan is indispensable as it provides a reference on the the legal requirement of ascertaining the identity of Lot Nos. 4 and 5
exact identity of the property. This begs the question in the instant applied for registration. The blueprint, which is shown to have been
case: Does the blueprint copy of the survey plan suffice for compliance duly executed by Geodetic Engineer Rolando Roxas (Roxas),
with the requirement? In not so many cases, 32 it was held that the attached to the application and subsequently identified, marked, and
non-submission, for any reason, of the original tracing cloth plan is offered in evidence, shows that it proceeded officially from the Lands
fatal to the registration application, since the same is mandatory in Management Services and, in fact, bears the approval of Surveys
original registration of title. For instance, in the Del Rosario case relied Division Chief Ernesto Erive. It also shows on its face that the survey
on by petitioner, the Court ruled that the submission of the original of the property was endorsed by the Community Environment and
copy of the duly approved tracing cloth plan is a mandatory condition Natural Resources Office of the DENR. 38 This, compounded by the
accompanying technical description of Lot Nos. 4 and 5 duly executed Proceeding from this fundamental principle, we find that
and verified also by Roxas, 39 should substantially supply as it did the indeed respondents have been in possession and occupation of Lot
means by which the identity of Lot Nos. 4 and 5 may be Nos. 4 and 5 under abona fide claim of ownership for the duration
ascertained. DEICaA required by law. This conclusion is primarily factual.
Verily, no error can be attributed to the Court of Appeals From the records, it is clear that respondents' possession
when it ruled that respondents were able to approximate compliance through their predecessor-in-interest dates back to as early as 1937.
with Section 17 of P.D. No. 1529. Also telling is the observation made In that year, the subject property had already been declared for
by the Court of Appeals that there was no objection raised by the taxation by Zenaida's father, Sergio, jointly with a certain Toribia
oppositor or by the LRA to the admission of the blueprint of Plan Ccs- Miranda (Toribia). 46 Yet, it also can be safely inferred that Sergio and
007601-000040-D despite the fact that they were well-informed of the Toribia had declared the land for taxation even earlier because the
present proceedings, to wit: 1937 tax declaration shows that it offsets a previous tax
number. 47 The property was again declared in 1979, 48 1985 49 and
In the instant case, the plaintiffs-appellants do not 1994 50 by Sergio, Toribia and by Romualdo. SHaATC
deny that only the blueprint copy of the plan of the
subject lands (Exh. "J") and not the original tracing Certainly, respondents could have produced more proof of
cloth plan thereof was submitted to the court a this kind had it not been for the fact that, as certified by the Office of
quo since they had previously submitted the the Rizal Provincial Assessor, the relevant portions of the tax records
original tracing cloth plan to the Land Registration on file with it had been burned when the assessor's office was razed
Authority. However, despite the failure of the by fire in 1997. 51 Of equal relevance is the fact that with these tax
plaintiffs-appellants to present the original tracing assessments, there came next tax payments. Respondents' receipts
cloth plan, neither the Land Registration Authority for tax expenditures on Lot Nos. 4 and 5 between 1977 and 2001 are
nor the oppositor-appellee question[ed] this likewise fleshed out in the records and in these documents, Sergio,
deficiency. Likewise, when the blueprint copy of Toribia and Romualdo are the named owners of the property with
the plan (Exh. "J") was offered in evidence, the Zenaida being identified as the one who delivered the payment in the
oppositor-appellee did not raise any objection 1994 receipts. 52
thereto. Such silence on the part of the Land
Registration [Authority] and the oppositor-appellee The foregoing evidentiary matters and muniments clearly
can be deemed as an implied admission that the show that Zenaida's testimony in this respect is no less believable.
original tracing cloth plan and the blueprint copy And the unbroken chain of positive acts exercised by respondents'
thereof (Exh. "J") are one and the same, free from predecessors, as demonstrated by these pieces of evidence, yields
all defects and clearly identify the lands sought to no other conclusion than that as early as 1937, they had already
be registered. In this regard . . ., the blueprint copy demonstrated an unmistakable claim to the property. Not only do they
of the plan (Exh. "J"), together with its technical show that they had excluded all others in their claim but also, that such
descriptions (Exhs. "K" and "L"), is deemed claim is in all good faith.
tantamount to substantial compliance with the Land registration proceedings are governed by the rule that
requirements of law. 40 while tax declarations and realty tax payment are not conclusive
evidence of ownership, nevertheless, they are a good indication of
We now proceed to the issue of possession. Petitioner possession in the concept of owner. These documents constitute at
theorizes that not only were respondents unable to identify the lots least proof that the holder has a claim of title over the property, for no
applied for registration; it also claims that they have no credible one in his right mind would be paying taxes for a property that is not
evidence tending to establish that for at least 30 years they and their in his actual or at least constructive possession. The voluntary
predecessors-in-interest have occupied and possessed the property declaration of a piece of property for taxation purposes manifests not
openly, continuously, exclusively and notoriously under a bona only one's sincere and honest desire to obtain title to the property. It
fide claim of ownership since June 12, 1945 or earlier. 41 We do not also announces his adverse claim against the state and all other
agree.
parties who may be in conflict with his interest. More importantly, it
In an original registration of title under Section 14 signifies an unfeigned intention to contribute to government revenues
(1) 42 P.D. No. 1529, the applicant for registration must be able to — an act that strengthens one's bona fide claim of acquisition of
establish by evidence that he and his predecessor-in-interest have ownership. 53
exercised acts of dominion over the lot under a bona fide claim of
Indeed, that respondents herein have been in possession
ownership since June 12, 1945 or earlier. 43 He must prove that for at
of the land in the concept of owner — open, continuous, peaceful and
least 30 years, he and his predecessor have been in open, continuous,
without interference and opposition from the government or from any
exclusive and notorious possession and occupation of the
private individual — itself makes their right thereto unquestionably
land.Republic v. Alconaba 44 well explains possession and
settled and, hence, deserving of protection under the law.
occupation of this character, thus:
WHEREFORE, the petition is DENIED. The March 30, 2006 Decision and
The law speaks of possession and occupation. the November 20, 2006 Resolution of the Court of Appeals, in CA-G.R. CV
Since these words are separated by the No. 80500, are AFFIRMED.
conjunction and, the clear intention of the law is
not to make one synonymous with the other. SO ORDERED.
Possession is broader than occupation because it
includes constructive possession. When, ||| (Republic v. Guinto-Aldana, G.R. No. 175578, [August 11, 2010], 642
therefore, the law adds the word occupation, it PHIL 364-379)
seeks to delimit the all-encompassing effect of
constructive possession. Taken together with
the words open, continuous, exclusive and
notorious, the word occupation serves to
highlight the fact that for an applicant to
qualify, his possession must not be a mere
fiction. Actual possession of a land consists in
the manifestation of acts of dominion over it of
such a nature as a party would naturally
exercise over his own property. 45
SECOND DIVISION Petitioner Republic appealed the MTC Judgment, dated 21 December
[G.R. No. 156117. May 26, 2005.] 1999, to the Court of Appeals. 19 The Court of Appeals, in its Decision, dated 22
November 2002, affirmed the appealed MTC Judgment reasoning thus:
REPUBLIC OF THE PHILIPPINES, petitioner, vs.
JEREMIAS AND DAVID HERBIETO, respondents. In the case at bar, there can be no question
that the land sought to be registered has been classified
DECISION as within the alienable and disposable zone since June
25, 1963. Article 1113 in relation to Article 1137 of the
CHICO-NAZARIO, J p: Civil Code, respectively provides that "All things which
are within the commerce of men are susceptible of
prescription, unless otherwise provided. Property of the
Before this Court is a Petition for Review on Certiorari, under Rule 45 of
State or any of its subdivisions of patrimonial character
the 1997 Rules of Civil Procedure, seeking the reversal of the Decision of the Court
shall not be the object of prescription" and that
of Appeals in CA-G.R. CV No. 67625, dated 22 November 2002, 1 which affirmed the
"Ownership and other real rights over immovables also
Judgment of the Municipal Trial Court (MTC) of Consolacion, Cebu, dated 21
prescribe through uninterrupted adverse possession
December 1999, 2 granting the application for land registration of the respondents.
thereof for thirty years, without need of title or of good
Respondents in the present Petition are the Herbieto brothers, Jeremias faith."2005cdtai
and David, who filed with the MTC, on 23 September 1998, a single application for
As testified to by the appellees in the case
registration of two parcels of land, Lots No. 8422 and 8423, located in Cabangahan,
at bench, their parents already acquired the subject
Consolacion, Cebu (Subject Lots). They claimed to be owners in fee simple of the
parcels of lands, subject matter of this application, since
Subject Lots, which they purchased from their parents, spouses Gregorio Herbieto
1950 and that they cultivated the same and planted it with
and Isabel Owatan, on 25 June 1976. 3 Together with their application for
jackfruits, bamboos, coconuts, and other trees
registration, respondents submitted the following set of documents:
(Judgment dated December 21, 1999, p. 6). In short, it is
(a) Advance Survey Plan of Lot No. 8422, in the name of respondent undisputed that herein appellees or their predecessors-
Jeremias; and Advance Survey Plan of Lot No. 8423, in the in-interest had occupied and possessed the subject land
name of respondent David; 4 openly, continuously, exclusively, and adversely since
1950. Consequently, even assuming arguendo that
(b) The technical descriptions of the Subject Lots; 5 appellees' possession can be reckoned only from June
25, 1963 or from the time the subject lots had been
(c) Certifications by the Department of Environment and Natural classified as within the alienable and disposable zone,
Resources (DENR) dispensing with the need for Surveyor's still the argument of the appellant does not hold
Certificates for the Subject Lots; 6 water. CTacSE

(d) Certifications by the Register of Deeds of Cebu City on the absence As earlier stressed, the subject property,
of certificates of title covering the Subject Lots; 7 being alienable since 1963 as shown by CENRO Report
dated June 23, 1963, may now be the object of
(e) Certifications by the Community Environment and Natural Resources prescription, thus susceptible of private ownership. By
Office (CENRO) of the DENR on its finding that the Subject express provision of Article 1137, appellees are, with
Lots are alienable and disposable, by virtue of Forestry much greater right, entitled to apply for its registration, as
Administrative Order No. 4-1063, dated 25 June 1963; 8 provided by Section 14(4) of P.D. 1529 which allows
individuals to own land in any manner provided by law.
(f) Certified True Copies of Assessment of Real Property (ARP) No. Again, even considering that possession of appellees
941800301831, in the name of Jeremias, covering Lot No. should only be reckoned from 1963, the year when
8422, issued in 1994; and ARP No. 941800301833, in the CENRO declared the subject lands alienable, herein
name of David, covering Lot No. 8423, also issued in appellees have been possessing the subject parcels of
1994; 9 and TEHDIA
land in open, continuous, and in the concept of an owner,
(g) Deed of Definite Sale executed on 25 June 1976 by spouses for 35 years already when they filed the instant
Gregorio Herbieto and Isabel Owatan selling the Subject application for registration of title to the land in 1998. As
Lots and the improvements thereon to their sons and such, this court finds no reason to disturb the finding of
respondents herein, Jeremias and David, for P1,000. Lot the court a quo. 20
No. 8422 was sold to Jeremias, while Lot No. 8423 was sold The Republic filed the present Petition for the review and reversal of the
to David.10 Decision of the Court of Appeals, dated 22 November 2002, on the basis of the
On 11 December 1998, the petitioner Republic of the Philippines following arguments:
(Republic) filed an Opposition to the respondents' application for registration of the First, respondents failed to establish that they and their predecessors-in-
Subject Lots arguing that: (1) Respondents failed to comply with the period of adverse interest had been in open, continuous, and adverse possession of the Subject Lots
possession of the Subject Lots required by law; (2) Respondents' muniments of title in the concept of owners since 12 June 1945 or earlier. According to the petitioner
were not genuine and did not constitute competent and sufficient evidence of bona Republic, possession of the Subject Lots prior to 25 June 1963 cannot be considered
fide acquisition of the Subject Lots; and (3) The Subject Lots were part of the public in determining compliance with the periods of possession required by law. The
domain belonging to the Republic and were not subject to private appropriation. 11 Subject Lots were classified as alienable and disposable only on 25 June 1963, per
The MTC set the initial hearing on 03 September 1999 at 8:30 a.m. 12 All CENRO's certification. It also alleges that the Court of Appeals, in applying the 30-
owners of the land adjoining the Subject Lots were sent copies of the Notice of Initial year acquisitive prescription period, had overlooked the ruling in Republic v.
Hearing. 13 A copy of the Notice was also posted on 27 July 1999 in a conspicuous Doldol, 21 where this Court declared that Commonwealth Act No. 141, otherwise
place on the Subject Lots, as well as on the bulletin board of the municipal building known as the Public Land Act, as amended and as it is presently phrased, requires
of Consolacion, Cebu, where the Subject Lots were located. 14 Finally, the Notice that possession of land of the public domain must be from 12 June 1945 or earlier,
was also published in the Official Gazette on 02 August 1999 15 and The Freeman for the same to be acquired through judicial confirmation of imperfect title.
Banat News on 19 December 1999. 16 Second, the application for registration suffers from fatal infirmity as the
During the initial hearing on 03 September 1999, the MTC issued an subject of the application consisted of two parcels of land individually and separately
Order of Special Default, 17 with only petitioner Republic opposing the application for owned by two applicants. Petitioner Republic contends that it is implicit in the
registration of the Subject Lots. The respondents, through their counsel, proceeded provisions of Presidential Decree No. 1529, otherwise known as the Property
to offer and mark documentary evidence to prove jurisdictional facts. The MTC Registration Decree, as amended, that the application for registration of title to land
commissioned the Clerk of Court to receive further evidence from the respondents shall be filed by a single applicant; multiple applicants may file a single application
and to submit a Report to the MTC after 30 days. only in case they are co-owners. While an application may cover two parcels of land,
it is allowed only when the subject parcels of land belong to the same applicant or
On 21 December 1999, the MTC promulgated its Judgment ordering the applicants (in case the subject parcels of land are co-owned) and are situated within
registration and confirmation of the title of respondent Jeremias over Lot No. 8422 the same province. Where the authority of the courts to proceed is conferred by a
and of respondent David over Lot No. 8423. It subsequently issued an Order on 02 statute and when the manner of obtaining jurisdiction is mandatory, it must be strictly
February 2000 declaring its Judgment, dated 21 December 1999, final and executory, complied with or the proceedings will be utterly void. Since the respondents failed to
and directing the Administrator of the Land Registration Authority (LRA) to issue a comply with the procedure for land registration under the Property Registration
decree of registration for the Subject Lots. 18 Decree, the proceedings held before the MTC is void, as the latter did not acquire
jurisdiction over it.
I A land registration case is a proceeding in rem, 28 and jurisdiction in
rem cannot be acquired unless there be constructive seizure of the land through
Jurisdiction
publication and service of notice. 29
Addressing first the issue of jurisdiction, this Court finds that the MTC
had no jurisdiction to proceed with and hear the application for registration filed by Section 23 of the Property Registration Decree requires that the public
the respondents but for reasons different from those presented by petitioner Republic. be given Notice of the Initial Hearing of the application for land registration by means
of (1) publication; (2) mailing; and (3) posting. Publication of the Notice of Initial
A. The misjoinder of causes of action and parties does not affect the Hearing shall be made in the following manner:
jurisdiction of the MTC to hear and proceed with respondents'
application for registration. 1. By publication. —

Respondents filed a single application for registration of the Subject Lots Upon receipt of the order of the court setting
even though they were not co-owners. Respondents Jeremias and David were the time for initial hearing, the Commissioner of Land
actually seeking the individual and separate registration of Lots No. 8422 and 8423, Registration shall cause a notice of initial hearing to be
respectively. CSEHcT published once in the Official Gazette and once in a
newspaper of general circulation in the
Petitioner Republic believes that the procedural irregularity committed by Philippines: Provided, however, that the publication in
the respondents was fatal to their case, depriving the MTC of jurisdiction to proceed the Official Gazette shall be sufficient to confer
with and hear their application for registration of the Subject Lots, based on this jurisdiction upon the court. Said notice shall be
Court's pronouncement in Director of Lands v. Court of Appeals, 22 to wit: addressed to all persons appearing to have an interest in
the land involved including the adjoining owners so far as
. . . In view of these multiple omissions which known, and "to all whom it may concern." Said notice
constitute non-compliance with the above-cited sections shall also require all persons concerned to appear in
of the Act, We rule that said defects have not invested court at a certain date and time to show cause why the
the Court with the authority or jurisdiction to proceed with prayer of said application shall not be granted. CEDHTa
the case because the manner or mode of obtaining
jurisdiction as prescribed by the statute which is Even as this Court concedes that the aforequoted Section 23(1) of the
mandatory has not been strictly followed, thereby Property Registration Decree expressly provides that publication in the Official
rendering all proceedings utterly null and void. Gazette shall be sufficient to confer jurisdiction upon the land registration court, it still
affirms its declaration in Director of Lands v. Court of Appeals 30 that publication in a
This Court, however, disagrees with petitioner Republic in this regard. newspaper of general circulation is mandatory for the land registration court to validly
This procedural lapse committed by the respondents should not affect the jurisdiction confirm and register the title of the applicant or applicants. That Section 23 of the
of the MTC to proceed with and hear their application for registration of the Subject Property Registration Decree enumerated and described in detail the requirements
Lots. of publication, mailing, and posting of the Notice of Initial Hearing, then all such
requirements, including publication of the Notice in a newspaper of general
The Property Registration Decree 23 recognizes and expressly allows
circulation, is essential and imperative, and must be strictly complied with. In the
the following situations: (1) the filing of a single application by several applicants for
same case, this Court expounded on the reason behind the compulsory publication
as long as they are co-owners of the parcel of land sought to be registered; 24 and
of the Notice of Initial Hearing in a newspaper of general circulation, thus —
(2) the filing of a single application for registration of several parcels of land provided
that the same are located within the same province. 25 The Property Registration It may be asked why publication in a
Decree is silent, however, as to the present situation wherein two applicants filed a newspaper of general circulation should be deemed
single application for two parcels of land, but are seeking the separate and individual mandatory when the law already requires notice by
registration of the parcels of land in their respective names. publication in the Official Gazette as well as by mailing
and posting, all of which have already been complied
Since the Property Registration Decree failed to provide for such a
with in the case at hand. The reason is due process and
situation, then this Court refers to the Rules of Court to determine the proper course
the reality that the Official Gazette is not as widely read
of action. Section 34 of the Property Registration Decree itself provides that, "[t]he
and circulated as newspaper and is oftentimes delayed
Rules of Court shall, insofar as not inconsistent with the provisions of this Decree, be
in its circulation, such that the notices published therein
applicable to land registration and cadastral cases by analogy or in a suppletory
character and whenever practicable and convenient." may not reach the interested parties on time, if at all.
Additionally, such parties may not be owners of
Considering every application for land registration filed in strict neighboring properties, and may in fact not own any
accordance with the Property Registration Decree as a single cause of action, then other real estate. In sum, the all encompassing in
the defect in the joint application for registration filed by the respondents with the MTC rem nature of land registration cases, the consequences
constitutes a misjoinder of causes of action and parties. Instead of a single or joint of default orders issued against the whole world and the
application for registration, respondents Jeremias and David, more appropriately, objective of disseminating the notice in as wide a manner
should have filed separate applications for registration of Lots No. 8422 and 8423, as possible demand a mandatory construction of the
respectively. requirements for publication, mailing and posting. 31

Misjoinder of causes of action and parties do not involve a question of In the instant Petition, the initial hearing was set by the MTC, and was in
jurisdiction of the court to hear and proceed with the case. 26 They are not even fact held, on 03 September 1999 at 8:30 a.m. While the Notice thereof was printed in
accepted grounds for dismissal thereof. 27 Instead, under the Rules of Court, the the issue of the Official Gazette, dated 02 August 1999, and officially released on 10
misjoinder of causes of action and parties involve an implied admission of the court's August 1999, it was published in The Freeman Banat News, a daily newspaper
jurisdiction. It acknowledges the power of the court, acting upon the motion of a party printed in Cebu City and circulated in the province and cities of Cebu and in the rest
to the case or on its own initiative, to order the severance of the misjoined cause of of Visayas and Mindanao, only on 19 December 1999, more than three months after
action, to be proceeded with separately (in case of misjoinder of causes of action); the initial hearing.
and/or the dropping of a party and the severance of any claim against said misjoined
party, also to be proceeded with separately (in case of misjoinder of parties). Indubitably, such publication of the Notice, way after the date of the initial
hearing, would already be worthless and ineffective. Whoever read the Notice as it
The misjoinder of causes of action and parties in the present Petition was published in The Freeman Banat News and had a claim to the Subject Lots was
may have been corrected by the MTC motu propio or on motion of the petitioner deprived of due process for it was already too late for him to appear before the MTC
Republic. It is regrettable, however, that the MTC failed to detect the misjoinder when on the day of the initial hearing to oppose respondents' application for registration,
the application for registration was still pending before it; and more regrettable that and to present his claim and evidence in support of such claim. Worse, as the Notice
the petitioner Republic did not call the attention of the MTC to the fact by filing a itself states, should the claimant-oppositor fail to appear before the MTC on the date
motion for severance of the causes of action and parties, raising the issue of of initial hearing, he would be in default and would forever be barred from contesting
misjoinder only before this Court. respondents' application for registration and even the registration decree that may be
issued pursuant thereto. In fact, the MTC did issue an Order of Special Default on 03
B. Respondents, however, failed to comply with the publication requirements September 1999.
mandated by the Property Registration Decree, thus, the MTC
was not invested with jurisdiction as a land registration court. The late publication of the Notice of Initial Hearing in the newspaper of
general circulation is tantamount to no publication at all, having the same ultimate
Although the misjoinder of causes of action and parties in the present result. Owing to such defect in the publication of the Notice, the MTC failed to
Petition did not affect the jurisdiction of the MTC over the land registration proceeding, constructively seize the Subject Lots and to acquire jurisdiction over respondents'
this Court, nonetheless, has discovered a defect in the publication of the Notice of application for registration thereof. Therefore, the MTC Judgment, dated 21
Initial Hearing, which bars the MTC from assuming jurisdiction to hear and proceed December 1999, ordering the registration and confirmation of the title of respondents
with respondents' application for registration. ASDTEa Jeremias and David over Lots No. 8422 and 8423, respectively; as well as the MTC
Order, dated 02 February 2000, declaring its Judgment of 21 December 1999 final (c) Members of the national cultural minorities who by themselves or
and executory, and directing the LRA Administrator to issue a decree of registration through their predecessors-in-interest have been in open,
for the Subject Lots, are both null and void for having been issued by the MTC without continuous, exclusive and notorious possession and
jurisdiction. occupation of lands of the public domain suitable to
II agriculture whether disposable or not, under abona
Period of Possession fide claim of ownership since June 12, 1945 shall be entitled
to the rights granted in subsection (b) hereof.
Respondents failed to comply with the required period of possession of the
Subject Lots for the judicial confirmation or legalization of imperfect or Not being members of any national cultural minorities, respondents may
incomplete title. only be entitled to judicial confirmation or legalization of their imperfect or incomplete
While this Court has already found that the MTC did not have jurisdiction title under Section 48(b) of the Public Land Act, as amended. Section 48(b), as
to hear and proceed with respondents' application for registration, this Court amended, now requires adverse possession of the land since 12 June 1945 or earlier.
nevertheless deems it necessary to resolve the legal issue on the required period of In the present Petition, the Subject Lots became alienable and disposable only on 25
possession for acquiring title to public land. TAEDcS June 1963. Any period of possession prior to the date when the Subject Lots were
classified as alienable and disposable is inconsequential and should be excluded
Respondents' application filed with the MTC did not state the statutory from the computation of the period of possession; such possession can never ripen
basis for their title to the Subject Lots. They only alleged therein that they obtained into ownership and unless the land had been classified as alienable and disposable,
title to the Subject Lots by purchase from their parents, spouses Gregorio Herbieto the rules on confirmation of imperfect title shall not apply thereto. 41 It is very
and Isabel Owatan, on 25 June 1976. Respondent Jeremias, in his testimony, apparent then that respondents could not have complied with the period of
claimed that his parents had been in possession of the Subject Lots in the concept of possession required by Section 48(b) of the Public Land Act, as amended, to acquire
an owner since 1950. 32 imperfect or incomplete title to the Subject Lots that may be judicially confirmed or
legalized.
Yet, according to the DENR-CENRO Certification, submitted by
respondents themselves, the Subject Lots are "within Alienable and Disposable, The confirmation of respondents' title by the Court of Appeals was based
Block I, Project No. 28 per LC Map No. 2545 of Consolacion, Cebu certified under on the erroneous supposition that respondents were claiming title to the Subject Lots
Forestry Administrative Order No. 4-1063, dated June 25, 1963. Likewise, it is outside under the Property Registration Decree. According to the Decision of the Court of
Kotkot-Lusaran Mananga Watershed Forest Reservation per Appeals, dated 22 November 2002, Section 14(4) of the Property Registration
Presidential Proclamation No. 932 dated June 29, 1992." 33 The Subject Lots are Decree allows individuals to own land in any other manner provided by law. It then
thus clearly part of the public domain, classified as alienable and disposable as of 25 ruled that the respondents, having possessed the Subject Lots, by themselves and
June 1963. through their predecessors-in-interest, since 25 June 1963 to 23 September 1998,
when they filed their application, have acquired title to the Subject Lots
As already well-settled in jurisprudence, no public land can be acquired by extraordinary prescription under Article 1113, in relation to Article 1137, both of
by private persons without any grant, express or implied, from the government;34 and the Civil Code. 42
it is indispensable that the person claiming title to public land should show that his
title was acquired from the State or any other mode of acquisition recognized by The Court of Appeals overlooked the difference between the Property
law. 35 Registration Decree and the Public Land Act. Under the Property Registration
Decree, there already exists a title which is confirmed by the court; while under the
The Public Land Act, as amended, governs lands of the public domain, Public Land Act, the presumption always is that the land applied for pertains to the
except timber and mineral lands, friar lands, and privately-owned lands which State, and that the occupants and possessors only claim an interest in the same by
reverted to the State. 36 It explicitly enumerates the means by which public lands virtue of their imperfect title or continuous, open, and notorious possession. 43 As
may be disposed, as follows: established by this Court in the preceding paragraphs, the Subject Lots respondents
wish to register are undoubtedly alienable and disposable lands of the public domain
(1) For homestead settlement; and respondents may have acquired title thereto only under the provisions of the
(2) By sale; Public Land Act.
(3) By lease;
(4) By confirmation of imperfect or incomplete titles; However, it must be clarified herein that even though respondents may
(a) By judicial legalization; or acquire imperfect or incomplete title to the Subject Lots under the Public Land Act,
(b) By administrative legalization (free their application for judicial confirmation or legalization thereof must be in accordance
patent). 37 with the Property Registration Decree, for Section 50 of the Public Land Act reads —
Each mode of disposition is appropriately covered by separate chapters
SEC. 50. Any person or persons, or their
of the Public Land Act because there are specific requirements and
legal representatives or successors in right, claiming any
application procedure for every mode. 38 Since respondents herein filed their
lands or interest in lands under the provisions of this
application before the MTC, 39 then it can be reasonably inferred that they
chapter, must in every case present an application to the
are seeking the judicial confirmation or legalization of their imperfect or
proper Court of First Instance, praying that the validity of
incomplete title over the Subject Lots.
the alleged title or claim be inquired into and that a
Judicial confirmation or legalization of imperfect or incomplete title to certificate of title be issued to them under the provisions
land, not exceeding 144 hectares, 40 may be availed of by persons identified of the Land Registration Act. 44
underSection 48 of the Public Land Act, as amended by Presidential Decree No.
1073, which reads — Hence, respondents' application for registration of the Subject Lots must
have complied with the substantial requirements under Section 48(b) of the Public
Section 48. The following-described citizens Land Act and the procedural requirements under the Property Registration Decree.
of the Philippines, occupying lands of the public domain
or claiming to own any such lands or an interest therein, Moreover, provisions of the Civil Code on prescription of ownership and
but whose titles have not been perfected or completed, other real rights apply in general to all types of land, while the Public Land
may apply to the Court of First Instance of the province Actspecifically governs lands of the public domain. Relative to one another, the Public
where the land is located for confirmation of their claims Land Act may be considered a special law 45 that must take precedence over the
and the issuance of a certificate of title thereafter, under Civil Code, a general law. It is an established rule of statutory construction that
the Land Registration Act, to wit: between a general law and a special law, the special law prevails —Generalia
specialibus non derogant. 46
(a) [Repealed by Presidential Decree No. 1073].
WHEREFORE, based on the foregoing, the instant Petition is
(b) Those who by themselves or through their predecessors-in-interest GRANTED. The Decision of the Court of Appeals in CA-G.R. CV No. 67625, dated
have been in open, continuous, exclusive, and notorious 22 November 2002, is REVERSED. The Judgment of the MTC of Consolacion, Cebu
possession and occupation of agricultural lands of the public in LRC Case No. N-75, dated 21 December 1999, and its Order, dated 02 February
domain, under a bona fide claim of acquisition of ownership, 2000 are declared NULL AND VOID. Respondents' application for registration is
since June 12, 1945, or earlier, immediately preceding the DISMISSED. EAIcCS
filing of the applications for confirmation of title, except when
prevented by war or force majeure. These shall be SO ORDERED.
conclusively presumed to have performed all the conditions
||| (Republic v. Herbieto, G.R. No. 156117, [May 26, 2005], 498 PHIL 227-247)
essential to a Government grant and shall be entitled to a
certificate of title under the provisions of this
chapter. TIAEac
SECOND DIVISION land applied for registration of title, and that it had registrable title
thereto in accordance with Section 14 of P.D. 1529.
[G.R. No. 172102. July 2, 2010.] On appeal by the State, the judgment of the RTC was
affirmed by the CA via the presently assailed Decision and Resolution.

REPUBLIC OF THE Hence, the instant petition based on the following grounds:
PHILIPPINES, petitioner,vs.HANOVER I
WORLWIDE TRADING
CORPORATION, respondent. THE DEFECTIVE AND/OR WANT OF NOTICE
BY PUBLICATION OF THE INITIAL HEARING OF
THE CASE A QUO DID NOT VEST THE TRIAL
COURT WITH JURISDICTION TO TAKE
DECISION COGNIZANCE THEREOF.
II
DEEDS OF SALE AND TAX
PERALTA, J p: DECLARATIONS/CLEARANCES DID NOT
CONSTITUTE THE "WELL-NIGH
Before the Court is a petition for review on certiorari under INCONTROVERTIBLE" EVIDENCE
Rule 45 of the Rules of Court, seeking the reversal and setting aside NECESSARY TO ACQUIRE TITLE THROUGH
of the Decision 1dated May 6, 2005 of the Court of Appeals (CA) in ADVERSE OCCUPATION. 4
CA-G.R. CV No. 70077, which affirmed the August 7, 1997 Decision
of the Regional Trial Court (RTC) of Mandaue City, Branch 56, in Petitioner claims that the RTC failed to acquire jurisdiction
LAND REG. CASE NO. N-281. Petitioner also assails the CA over the case. It avers that the RTC set the initial hearing of the case
Resolution 2 dated March 30, 2006, denying its Motion for on September 25, 1995 in an Order dated June 13, 1995. Petitioner
Reconsideration. contends, however, that, pursuant to Section 23 of P.D. 1529, the
initial hearing of the case must be not earlier than forty-five (45) days
The facts of the case are as follows: and not later than ninety (90) days from the date of the Order setting
On October 15, 1993, Hanover Worldwide Trading the date and hour of the initial hearing. Since the RTC Order was
Corporation filed an application for Registration of Title over Lot No. issued on June 13, 1995, the initial hearing should have been set not
4488 of Consolacion Cad-545-D (New) under Vs-072219-000396, earlier than July 28, 1995 (45 days from June 13, 1995) and not later
situated in Barrio Sacsac, Consolacion, Cebu, containing an area of than September 11, 1995 (90 days from June 13, 1995).
One Hundred Three Thousand Three Hundred Fifty (103,350) square Unfortunately, the initial hearing was scheduled and actually held on
meters, more or less, pursuant to Presidential Decree (P.D.) No. 1529, September 25, 1998, some fourteen (14) days later than the
otherwise known as the Property Registration Decree.The application prescribed period. cDCIHT
stated that Hanover is the owner in fee simple of Lot No. 4488, its title Petitioner also argues that respondent failed to present
thereto having been obtained through purchase evidenced by a Deed incontrovertible evidence in the form of specific facts indicating the
of Absolute Sale. nature and duration of the occupation of its predecessor-in-interest to
Attached to the petition are: 1) a Verification Survey Plan; prove that the latter has been in possession of the subject lot under
2) a copy of the approved Technical Description of Lot 4488; 3) a copy a bona fide claim of acquisition of ownership since June 12, 1945 or
of the Deed of Sale in favor of Hanover's President and General earlier.
Manager; 4) a copy of a Waiver executed by the President and The petition is meritorious.
General Manager of Hanover in favor of the latter; 5) a Geodetic
Engineer's Certificate attesting that the property was surveyed; 6) a As to the first assigned error, however, the Court is not
Tax Declaration; 7) a tax clearance; 8) a Municipal Assessor's persuaded by petitioner's contention that the RTC did not acquire
Certification stating, among others, the assessed value and market jurisdiction over the case. It is true that in land registration cases, the
value of the property; and 9) a CENRO Certification on the alienability applicant must strictly comply with the jurisdictional requirements. In
and disposability of the property. TDESCa the instant case, though, there is no dispute that respondent complied
with the requirements of the law for the court to acquire jurisdiction
Except for the Republic, there were no other oppositors to over the case.
the application. The Republic contended, among others, that neither
Hanover nor its predecessors-in-interest are in open, continuous, With respect to the setting of the initial hearing outside the
exclusive and notorious possession and occupation of the land in 90-day period set forth under Section 23 of P.D. 1529, the Court
question since June 12, 1945 or prior thereto; the muniments of title, agrees with the CA in ruling that the setting of the initial hearing is the
tax declarations and receipts of tax payments attached to or alleged duty of the land registration court and not the applicant.
in the application do not constitute competent and sufficient evidence Citing Republic v. Manna Properties, Inc., 5 this Court held
of a bona fide acquisition of the lands applied for; Hanover is a private in Republic v. San Lorenzo Development Corporation 6 that:
corporation disqualified under the Constitution to hold alienable lands The duty and the power to set the hearing date lie
of the public domain; the parcels of land applied for are portions of the with the land registration court. After an applicant
public domain belonging to the Republic and are not subject to private has filed his application, the law requires the
appropriation. issuance of a court order setting the initial hearing
The case was then called for trial and respondent date. The notice of initial hearing is a court
proceeded with the presentation of its evidence. The Republic was document. The notice of initial hearing is signed by
represented in the proceedings by officers from the Office of the the judge and copy of the notice is mailed by the
Solicitor General (OSG) and the Department of Environment and clerk of court to the LRA [Land Registration
Natural Resources (DENR). Authority].This involves a process to which the
party-applicant absolutely has no participation. ...
On August 7, 1997, the RTC rendered its
Decision 3 approving Hanover's application for registration of the xxx xxx xxx
subject lot. It held that from the documentary and oral evidence
presented by Hanover, the trial court was convinced that Hanover and ...a party to an action has no control over the
its predecessors-in-interest had been in open, public, continuous, Administrator or the Clerk of Court acting as a land
notorious and peaceful possession, in the concept of an owner, of the court; he has no right to meddle unduly with the
business of such official in the performance of his of the application for confirmation of
duties. A party cannot intervene in matters within title except when prevented by war
the exclusive power of the trial court. No fault is or force majeure.These shall be
attributable to such party if the trial court errs on conclusively presumed to have
matters within its sole power. It is unfair to punish performed all the conditions essential
an applicant for an act or omission over which the to a Government grant and shall be
applicant has neither responsibility nor control, entitled to a certificate of title under the
especially if the applicant has complied with all the provisions of this chapter. 8HDITCS
requirements of the law.
As the law now stands, a mere showing of possession and
Moreover, it is evident in Manna Properties, Inc. occupation for 30 years or more is not sufficient. Therefore, since the
that what is more important than the date on which effectivity of P.D. 1073 on January 25, 1977, it must now be shown
the initial hearing is set is the giving of sufficient that possession and occupation of the piece of land by the applicant,
notice of the registration proceedings via by himself or through his predecessors-in-interest, started on June 12,
publication. ... ITESAc 1945 or earlier. This provision is in total conformity with Section 14 (1)
of P.D. 1529. 9
In the instant case, there is no dispute that sufficient notice
of the registration proceedings via publication was duly made. Thus, pursuant to the aforequoted provisions of law,
applicants for registration of title must prove: (1) that the subject land
Moreover, petitioner concedes (a) that respondent should forms part of the disposable and alienable lands of the public domain,
not be entirely faulted if the initial hearing that was conducted on and (2) that they have been in open, continuous, exclusive and
September 25, 1995 was outside the 90-day period set forth under notorious possession and occupation of the same under a bona
Section 23 of Presidential Decree No. 1529, and (b) that respondent fide claim of ownership since June 12, 1945, or earlier.
substantially complied with the requirement relating to the registration
of the subject land. It is true, as respondent argues, that an examination of
these requisites involve delving into questions of fact which are not
Hence, on the issue of jurisdiction, the Court finds that the proper in a petition for review on certiorari.Factual findings of the
RTC did not commit any error in giving due course to respondent's court a quo are generally binding on this Court, except for certain
application for registration. recognized exceptions, 10 to wit:
The foregoing notwithstanding, the Court agrees with (1) When the conclusion is a finding grounded
petitioner on the more important issue that respondent failed to entirely on speculation, surmises and conjectures;
present sufficient evidence to prove that it or its predecessors-in-
interest possessed and occupied the subject property for the period (2) When the inference made is manifestly
required by law. mistaken, absurd or impossible;
Section 14 (1) of P.D. 1529, as amended, provides: (3) Where there is a grave abuse of discretion;
SEC. 14. Who may apply. — The following (4) When the judgment is based on a
persons may file in the proper Court of First misapprehension of facts;
Instance an application for registration of title to
land, whether personally or through their duly (5) When the findings of fact are conflicting;
authorized representatives:
(6) When the Court of Appeals, in making its
(1) Those who by themselves or findings, went beyond the issues of the case and
through their predecessors-in-interest the same is contrary to the admissions of both
have been in open, continuous, appellant and appellee;
exclusive and notorious possession
and occupation of alienable and (7) When the findings are contrary to those of the
disposable lands of the public domain trial Court;
under a bona fide claim of
(8) When the findings of fact are conclusions
ownership since June 12, 1945, or
without citation of specific evidence on which
earlier.7
they are based;
Likewise, Section 48 (b) of Commonwealth Act 141, as
(9) When the facts set forth in the petition as well
amended by Section 4 of P.D. 1073, states:
as in the petitioners' main and reply briefs are not
Section 48.The following described citizens of the disputed by the respondents; and CacEID
Philippines, occupying lands of the public domain
or claiming to own any such lands or an interest (10) When the findings of fact of the Court of
therein, but whose titles have not been perfected Appeals are premised on the supposed absence
or completed, may apply to the Court of First of evidence and contradicted by the evidence on
Instance [now Regional Trial Court] of the province record. 11
where the land is located for confirmation of their The Court finds that the instant case falls under the third
claims and the issuance of a certificate of title and ninth exceptions.
therefor, under the Land Registration Act, to wit:
A careful reading of the Decisions of the RTC and the CA
xxx xxx xxx will show that there is neither finding nor discussion by both the trial
and appellate courts which would support their conclusion that
(b) Those who by themselves or
respondent's predecessors-in-interest had open, continuous,
through their predecessors-in-interest
exclusive and notorious possession and occupation of the disputed
have been in open, continuous,
parcel of land since June 12, 1945 or earlier.
exclusive and notorious possession
and occupation of agricultural lands of No testimonial evidence was presented to prove that
the public domain, under a bona respondent or its predecessors-in-interest had been possessing and
fide claim of acquisition of occupying the subject property since June 12, 1945 or earlier.
ownership, since June 12, 1945, or Hanover's President and General Manager testified only with respect
earlier,immediately preceding the filing to his claim that he was the former owner of the subject property and
that he acquired the same from the heirs of a certain Damiano testified regarding its contents. Hence, the RTC should not have
Bontoyan; that he caused the payment of realty taxes due on the accepted the contents of the Certification as proof of the facts stated
property; that a tax declaration was issued in favor of Hanover; that therein. The contents of the Certification are hearsay, because
Hanover caused a survey of the subject lot, duly approved by the Hanover's President and General Manager was incompetent to testify
Bureau of Lands; and that his and Hanover's possession of the on the truth of the contents of such Certification. Even if the subject
property started in 1990. 12 Certification is presumed duly issued and admissible in evidence, it
has no probative value in establishing that the land is alienable and
The pieces of documentary evidence submitted by disposable. 20
respondent neither show that its predecessor's possession and
occupation of the subject land is for the period or duration required by Moreover, the CENRO is not the official repository or legal
law. The earliest date of the Tax Declarations presented in evidence custodian of the issuances of the DENR Secretary declaring the
by respondent is 1965, the others being 1973, 1980, 1992 and 1993. alienability and disposability of public lands. 21 Thus, the CENRO
Respondent failed to present any credible explanation why the realty Certification should have been accompanied by an official publication
taxes due on the subject property were only paid starting in 1965. of the DENR Secretary's issuance declaring the land alienable and
While tax declarations are not conclusive evidence of ownership, they disposable. HcTIDC
constitute proof of claim of ownership. 13 In the present case, the
payment of realty taxes starting 1965 gives rise to the presumption Respondent, however, failed to comply with the foregoing
that respondent's predecessors-in-interest claimed ownership or requirements.
possession of the subject lot only in that year. WHEREFORE,the petition is GRANTED.The May 6, 2005
Settled is the rule that the burden of proof in land Decision and March 30, 2006 Resolution of the Court of Appeals in
registration cases rests on the applicant who must show by clear, CA-G.R. CV No. 70077 and the August 7, 1997 Decision of the
positive and convincing evidence that his alleged possession and Regional Trial Court of Mandaue City, Branch 56 in Land Registration
occupation of the land is of the nature and duration required by Case No. N-281 are SET ASIDE.Respondent Hanover Worldwide
law. 14 Unfortunately, as petitioner contends, the pieces of evidence Trading Corporation's application for registration of Lot No. 4488 of
presented by respondent do not constitute the "well-nigh Consolacion Cad-545-D (New),under Vs-072219-000396, Barrio
incontrovertible" proof necessary in cases of this nature. Sacsac, Consolacion, Cebu, is DENIED.

Lastly, the Court notes that respondent failed to prove that SO ORDERED.
the subject lot had been declared alienable and disposable by the ||| (Republic v. Hanover Worldwide Trading Corp., G.R. No. 172102, [July
DENR Secretary. HEDSIc 2, 2010], 636 PHIL 739-753)
The well-entrenched rule is that all lands not appearing to
be clearly of private dominion presumably belong to the
State. 15 The onus to overturn, by incontrovertible evidence, the
presumption that the land subject of an application for registration is
alienable and disposable rests with the applicant. 16
In the present case, to prove the alienability and
disposability of the subject property, Hanover submitted a Certification
issued by the Community Environment and Natural Resources Offices
(CENRO) attesting that "lot 4488, CAD-545-D, containing an area of
ONE HUNDRED THREE THOUSAND THREE HUNDRED FIFTY
(103,350) square meters, more or less, situated at Sacsac,
Consolacion, Cebu" was found to be within "Alienable and Disposable
Block-1, land classification project no. 28, per map 2545 of
Consolacion, Cebu." However, this certification is not sufficient.
In Republic v. T.A.N. Properties, Inc. 17 this Court held
that it is not enough for the Provincial Environment and Natural
Resources Offices (PENRO) or CENRO to certify that a land is
alienable and disposable, thus:
...The applicant for land registration must prove
that the DENR Secretary had approved the land
classification and released the land of the public
domain as alienable and disposable, and that the
land subject of the application for registration falls
within the approved area per verification through
survey by the PENRO or CENRO. In addition, the
applicant for land registration must present a copy
of the original classification approved by the DENR
Secretary and certified as a true copy by the legal
custodian of the official records. These facts must
be established to prove that the land is alienable
and disposable ....18

In the instant case, even the veracity of the facts stated in


the CENRO Certification was not confirmed as only the President and
General Manager of respondent corporation identified said
Certification submitted by the latter. It is settled that a document or
writing admitted as part of the testimony of a witness does not
constitute proof of the facts stated therein. 19 In the present case,
Hanover's President and General Manager, who identified the
CENRO Certification, is a private individual. He was not the one who
prepared the Certification. The government official who issued the
Certification was not presented before the RTC so that he could have
FIRST DIVISION Petitioners questioned the inconsistencies in the dates and requested
[G.R. No. 146262. January 21, 2005.] the LRA to recall the decrees. The LRA Administrator denied the request and
explained the inconsistencies in the dates in a letter 13 dated 1 December 1998. The
HEIRS OF EUGENIO LOPEZ, SR., petitioners, vs. entire letter states:
HON. ALFREDO R. ENRIQUEZ, in his capacity as
Administrator of the Land Registration Authority Republic of the Philippines
and the REGISTER OF DEEDS OF MARIKINA Department of Justice
CITY, respondents. LAND REGISTRATION AUTHORITY
Quezon City
DECISION
CARPIO, J p: 1 December 1998
The Case
Atty. Crisostomo A. Quizon
This is a petition for review 1 to reverse the Decision 2 dated 29
Quiason Makalintal Barot Torres & Ibarra Law Offices
November 2000 of the Court of Appeals ("appellate court") in CA-G.R. SP No. 55993.
2nd Floor Benpres Building
The appellate court affirmed the Resolution 3 dated 21 May 1999 issued by the Land
Exchange Road corner Meralco Ave.
Registration Authority ("LRA") in Consulta No. 2879. The LRA ruled that a notice of lis Ortigas Center, Pasig City
pendens based on a motion is not registrable.
The Facts Sir:

Alfonso Sandoval ("Sandoval") and Roman Ozaeta, Jr. ("Ozaeta") filed This concerns your letter requesting the
an application for registration of title before the Regional Trial Court of Pasig City, recall of Decree Nos. N-217643 and N-217644 issued in
Branch 152 ("land registration court"), docketed as Case No. 2858, Land Registration Land Registration Case No. N-2858, LRC Record No. N-
Case No. N-18887 ("LRC No. N-18887"). The land registration court issued an order 18887, both in the names of Alfonso Sandoval and his
of general default and hearings on the application followed. On 31 May 1966, the land wife, Rosa Ruiz, and Roman Ozaeta, Jr., and his wife,
registration court granted the application. The decision became final and executory, Ma. Salome Lao.
and the land registration court issued a certificate of finality dated 8 March 1991. 4
Records of this Authority show that
The National Land Titles and Deeds Administration (now LRA) issued on aforesaid decrees of registration were prepared on
20 October 1977 Decree Nos. N-217643 and N-217644 in the names of Sandoval October 20, 1977 pursuant to the decision of the court
and his wife Rosa Ruiz, and Ozaeta and his wife Ma. Salome Lao. 5 dated May 31, 1966 and the order for issuance of decree
dated August 24, 1993. Said decrees were forwarded to
On 16 July 1997, petitioners Eugenio Lopez, Jr., Manolo Lopez, Oscar the Office of the Administrator on August 8, 1998 and
Lopez, and Presentacion L. Psinakis ("petitioners"), heirs of Eugenio Lopez, Sr., filed was [sic] released therefrom on August 13, 1998.
a motion 6 in LRC No. N-18887. The motion alleged that Sandoval and Ozaeta sold Consequently, said decrees were signed sometime
the lots subject of the application to the late Eugenio Lopez, Sr. on 23 September between August 8 and 13 1998 and definitely not on
1970. Petitioners prayed that the court consider in the land registration case the Deed October 20, 1997 as what is reflected thereon because
of Absolute Sale 7 over the lots executed by Sandoval and Ozaeta and their the undersigned Administrator assumed office only on
respective spouses in favor of Eugenio Lopez, Sr. Invoking Section 22 of Presidential July 8, 1998. Apparently, at the time the decrees were
Decree No. 1529 ("PD 1529"), 8 petitioners also prayed that the court issue the signed it was not noticed, through oversight, that they
decree of registration in their names as the successors-in-interest of Eugenio Lopez, were dated October 20, 1977. It is therefore hereby
Sr. clarified that Decree Nos. N-217643 and N-217644 were
actually issued sometime between August 8 and 13 1998
The land registration court gave due course to the motion and conducted and not on October 20, 1997.
hearings. 9
Regarding the claim that these decrees
The Register of Deeds of Marikina City issued the corresponding OCT were prematurely issued as the motion for the issuance
Nos. O-1603 and O-1604 in favor of Sandoval and Ozaeta and their spouses only on of the decrees in favor of the Heirs of Eugenio Lopez, the
18 August 1998. 10 The pertinent entries 11 in the Decrees read: properties involved having been sold to him by the
applicants, is still pending with the court, it is informed
This Decree is issued pursuant to the
that no copy of said motion nor of the order directing this
Decision dated 31st day of May, 1966 of the Hon. Pedro
Office to comment thereon appears on file in the records
C. Navarro, Judge of [Court of First Instance of Rizal,
of the case. Hence, these matters could not have been
Branch II, Pasig, Rizal], and the Honorable Briccio C.
taken into consideration in the issuance of the decrees.
Ygaña, this 3rd day of July, 1998.
Had the Administration been apprised of these incidents,
Issued at the National Land Titles and perhaps the issuance of the decrees could have been
Deeds Registration Administration, Quezon City, held in abeyance until the court has resolved the same.
this 20th day of October, in the year of Our Lord nineteen
As to the recall of the decrees of registration,
hundred and ninety-seven at 8:01 a.m. HIACEa
we regret to inform you that since the certificates of title
(signed) transcribed pursuant to said decrees have already been
ALFREDO R. ENRIQUEZ issued and released by the Registrar of Deeds
ADMINISTRATOR concerned, it is now beyond our authority to recall them
National Land Titles and unless duly authorized by the court.
Deeds Registration
We hope that we have satisfactorily
Administration
disposed of the concerns raised in your letter.
Entered in the "Registration Book" for
Very truly yours,
Marikina, pursuant to the provisions of section
39 of PD No. 1529, on the 18th day of August (signed)
nineteen hundred and ninety-eight, at 1:16 ALFREDO R. ENRIQUEZ
p.m. Administrator
(signed) On 25 November 1998, petitioners filed with the Register of Deeds of
EDGAR D. SANTOS Marikina City an application to annotate the notice of lis pendens at the back of OCT
Register of Deeds (Emphasis added) Nos. O-1603 and O-1604 on the ground that petitioners have filed with the land
registration court a motion to declare OCT Nos. O-1603 and O-1604
Petitioners filed another motion on 25 November 1998 to declare void
void.14 Petitioners attached to the application a copy of the 25 November 1998
Decree Nos. N-217643 and N-217644 and Original Certificate of Title ("OCT") Nos.
motion and the pertinent OCTs. CASaEc
O-1603 and O-1604. Petitioners pointed out that the OCTs show that incumbent
Administrator Alfredo R. Enriquez signed the Decrees on 20 October 1997, before he In a letter 15 dated 15 December 1998, the Register of Deeds of
assumed office on 8 July 1998 and even before Hon. Briccio C. Ygaña issued the Marikina City denied the application to annotate the notice of lis pendens. The entire
Order of 3 July 1998. 12 letter states:
Republic of the Philippines error and grave abuse of discretion on the part of the LRA Administrator when he
Department of Justice ruled in Consulta No. 2879 that the notice of lis pendens is not registrable. caHIAS
LAND REGISTRATION AUTHORITY
Registry of Deeds, Marikina City The appellate court dismissed the petition for lack of merit. The appellate
court reiterated the LRA's ruling that only a party to a case has the legal personality
15 December 1998 to file a notice of lis pendens. Petitioners have no legal personality because they
failed to file a motion to lift the order of general default in the land registration case.
Atty. Crisostomo A. Quizon
2nd Floor, Benpres Bldg. Issues
Exchange Road cor. Meralco Avenue
Petitioners present the following issues for resolution of this Court:
Pasig City
1. WHETHER PETITIONERS' MOTION TO DECLARE
Sir:
VOID THE DECREES ISSUED BY THE
This is in connection to [sic] your application LAND REGISTRATION AUTHORITY IS A
to have a Notice of Lis Pendens [annotated] at the back PROPER BASIS FOR FILING THE
of OCT Nos. O-1603 and O-1604 issued in the name of NOTICE OF LIS PENDENS, and
ALFONSO SANDOVAL AND SPOUSE.
2. WHETHER PETITIONERS CAN FILE THE MOTION
Pursuant to Sec. 76, PD No. 1529[,] the TO DECLARE VOID THE DECREES
contents of the notice are the name[s] of the parties, the ISSUED BY THE LAND REGISTRATION
court where the action is pending, the date the action was COURT IN LRC CASE NO. N-18887
instituted and a copy of the compalint [sic] in order to DESPITE THE FACT THAT THE COURT
determine if the person named in the title is impleaded. HAS NOT LIFTED THE GENERAL
ORDER OF DEFAULT. 18
We regret to inform you that the application,
bereft of the original petition or compaint [sic] upon which The Ruling of the Court
this office will base its action, is DENIED. The petition has no merit.
If you do not agree with our findings, you We agree with the observation of the appellate court that the pleadings
can, without withdrawing the documents you submitted, filed by petitioners, public respondents and the Office of the Solicitor General cite
elevate the matter en consulta five (5) days from receipt "more or less the same provisions of the laws as applicable in support of their
hereof to the Office of the Administrator, Land respective contentions but differ . . . only with respect to their interpretation
Registration Authority, East Avenue cor. NIA Road, thereof." 19 With this observation in mind, we quote the pertinent provisions of the
Quezon City. 1997 Rules of Civil Procedure and of PD 1529.
Very truly yours, Section 14, Rule 13 of the 1997 Rules of Civil Procedure provides:
(signed) SECTION 14. Notice of lis pendens. — In an
EDGAR D. SANTOS action affecting the title or the right of possession of real
Register of Deeds property, the plaintiff and the defendant, when affirmative
relief is claimed in his answer, may record in the office of
the registry of deeds of the province in which the property
On 14 January 1999, three days after receipt of the letter, petitioners is situated a notice of the pendency of the action. Said
elevated the denial in consulta to the LRA. The case was docketed as Consulta No. notice shall contain the names of the parties and the
2879. object of the action or defense, and a description of the
property in that province affected thereby. Only from the
The Ruling of the Land Registration Authority time of filing such notice for record shall a purchaser, or
encumbrancer of the property affected thereby, be
In its resolution 16 dated 21 May 1999, the LRA stated that the sole deemed to have constructive notice of the pendency of
question for resolution is whether a notice of lis pendens is registrable based on a the action, and only of its pendency against the parties
motion to declare void the decrees and titles. The LRA agreed with the Register of designated by their real names.
Deeds that a notice of lis pendens based on a motion is not registrable. Relying on
Section 24, Rule 14 of the Rules of Court, the LRA ruled that only a party to a case The notice of lis pendens hereinabove
has the legal personality to file a notice of lis pendens relative to the pending case. mentioned may be cancelled only upon order of the
court, after proper showing that the notice is for the
The LRA focused on petitioners' standing in LRC No. N-18887. The LRA purpose of molesting the adverse party, or that it is not
declared that petitioners are not parties in LRC No. N-18887. Since a land registration necessary to protect the rights of the party who caused it
case is a proceeding in rem, an order of general default binds the whole world as a to be recorded.
party in the case. Petitioners are mere movants whose personality the court has not
admitted. Based on Section 26 of PD 1529, the LRA ruled that petitioners should Section 76 of PD 1529 states:
have filed a motion to lift the order of general default. Pertinent portions of the LRA
decision read: SECTION 76. Notice of lis pendens. — No
action to recover possession of real estate, or to quiet
Until and after the Order of General Default title thereto, or to remove clouds upon the title thereof, or
in LRC Case No. 18887 is lifted, petitioners cannot be for partition or other proceedings of any kind in court
clothed with personality as oppositors in said land directly affecting the title to land or the use or occupation
registration case by merely filing a motion after a thereof or the buildings thereon, and no judgment, and
judgment has been rendered. Such being the case, a no proceeding to vacate or reverse any judgment, shall
notice of lis pendens on the basis of the motion filed by have any effect upon registered land as against persons
petitioners cannot be admitted for registration. To rule other than the parties thereto, unless a memorandum or
otherwise would preempt the judgment of the Court in so notice stating the institution of such action or proceeding
far as the personalities of the movants as oppositors in and the court wherein the same is pending, as well as
the land registration case is concerned. the date of the institution thereof, together with a
reference to the number of the certificate of title, and an
adequate description of the land affected and the
registered owner thereof, shall have been filed and
WHEREFORE, premises considered, this
registered. CcHDSA
Authority is of the opinion and so holds that the notice
of lis pendens is not registrable. Notice of Lis Pendens
SO ORDERED. 17 Lis pendens literally means a pending suit. The doctrine of lis
pendens refers to the jurisdiction, power or control which a court acquires over
The Ruling of the Court of Appeals property involved in a suit, pending the continuance of the action, and until final
Undaunted, petitioners filed before the appellate court a petition for judgment. 20
review of the LRA's decision. Petitioners filed the petition on the ground of manifest
The purposes of lis pendens are (1) to protect the rights of the party a motion to declare Original Certificates of Title Nos. O-
causing the registration of the lis pendens, and (2) to advise third persons who 1603 and O-1604 null and void;
purchase or contract on the subject property that they do so at their peril and subject
to the result of the pending litigation. 21 7.2.2 It contains the name of the court
wherein the motion is pending which is "the registration
The filing of a notice of lis pendens has a two-fold effect. First, it keeps court, Regional Trial Court, Branch 152, Pasig City." The
the subject matter of the litigation within the power of the court until the entry of the date of the filing of the motion is shown on the motion
final judgment to prevent the defeat of the final judgment by successive alienations. itself wherein the receipt of said motion by the land
Second, it binds a purchaser, bona fide or not, of the land subject of the litigation to registration court on November 25, 1998 is duly
the judgment or decree that the court will promulgate subsequently. However, the stamped;
filing of a notice of lis pendens does not create a right or lien that previously did not
exist. 22 7.2.3 The numbers of the Original
Certificates of Title Nos. O-1603 and O-1604 are clearly
Without a notice of lis pendens, a third party who acquires the property indicated in the notice;
after relying only on the certificate of title is a purchaser in good faith. Against such
third party, the supposed rights of a litigant cannot prevail, because the former is not 7.2.4 There is adequate description of the
bound by the property owner's undertakings not annotated in the transfer certificate land affected in the Notice of Lis Pendens;
of title. 23 Thus, we have consistently held that —
7.2.5 The names of the registered owners
The notice of lis pendens . . . is ordinarily are indicated in Paragraph 4 of the Motion attached to
recorded without the intervention of the court where the the Notice;
action is pending. The notice is but an incident in an
action, an extrajudicial one, to be sure. It does not affect 7.2.6 A copy of the motion to declare OCT
the merits thereof. It is intended merely to constructively Nos. O-1603 and O-1604 null and void, dated November
advise, or warn, all people who deal with the property that 25, 1998 upon which the Register of Deeds of the
they so deal with it at their own risk, and whatever rights Province of Rizal will base its action is attached as Annex
they may acquire in the property in any voluntary "A" of the Notice of Lis Pendens. (Emphasis in the
transaction are subject to the results of the action, and original) 29
may well be inferior and subordinate to those which may
Petitioners' enumeration readily reveals that they have not complied with
be finally determined and laid down therein. The
the requisites. Both the LRA and the appellate court denied the application for a notice
cancellation of such a precautionary notice is therefore
of lis pendens because petitioners are mere movants, and not original parties, in LRC
also a mere incident in the action, and may be ordered
No. N-18887. As petitioners are not parties to an action as contemplated in Section
by the Court having jurisdiction of it at any given time.
76 of PD 1529, they failed to present the requisite pleading to the Register of Deeds
And its continuance or removal . . . is not contingent on
of Marikina City. We hold that the Register of Deeds correctly denied the application
the existence of a final judgment in the action, and for a notice of lis pendens. AECcTS
ordinarily has no effect on the merits thereof. 24
Reconveyance
A notice of lis pendens may involve actions that deal not only with title or
possession of a property, but also with the use or occupation of a property. 25The Petitioners committed a fatal procedural error when they filed a motion
litigation must directly involve a specific property which is necessarily affected by the in LRC No. N-18887 on 16 July 1997. The remedy of petitioners is an action for
judgment. Magdalena Homeowners Association, Inc. v. Court of reconveyance against Sandoval, Ozaeta and their spouses. Reconveyance is based
Appeals 26 enumerated the cases where a notice of lis pendens is appropriate: on Section 55 of Act No. 496, as amended by Act No. 3322, which states that ". . . in
all cases of registration procured by fraud the owner may pursue all his legal and
[A] notice of lis pendens is proper in the following equitable remedies against the parties to such fraud, without prejudice, however, to
cases, viz: the rights of any innocent holder for value of a certificate of title . . . ."
a) An action to recover possession of real estate;

b) An action to quiet title thereto; An action for reconveyance is an action in personam available to a
person whose property has been wrongfully registered under the Torrens system in
c) An action to remove clouds thereon; another's name. Although the decree is recognized as incontrovertible and no longer
open to review, the registered owner is not necessarily held free from liens. As a
d) An action for partition; and
remedy, an action for reconveyance is filed as an ordinary action in the ordinary
e) Any other proceedings of any kind in Court directly courts of justice and not with the land registration court. 30Reconveyance is always
affecting the title to the land or the use or available as long as the property has not passed to an innocent third person for value.
occupation thereof or the buildings thereon. A notice of lis pendens may thus be annotated on the certificate of title immediately
upon the institution of the action in court. The notice of lis pendens will avoid transfer
On the other hand, the doctrine of lis pendens has no application in the to an innocent third person for value and preserve the claim of the real owner. 31
following cases:
Necessity of a Motion to Lift the Order of General Default
a) Preliminary attachments;
In its comment, 32 the LRA states that under Section 26 of PD 1529 the
b) Proceedings for the probate of wills; order of default includes petitioners. Therefore, petitioners' failure to move to lift the
default order did not give them standing in the case. As long as the court does not lift
c) Levies on execution; the order of general default, petitioners have no legal standing to file the motion to
declare void the decrees of registration issued to the applicant. Section 26 of PD
d) Proceedings for administration of estate of deceased 1529 provides thus:
persons; and
Sec. 26. Order of default; effect. — If no
e) Proceedings in which the only object is the recovery person appears and answers within the time allowed, the
of a money judgment. 27 court shall, upon motion of the applicant, no reason to
the contrary appearing, order a default to be recorded
As decreed by Section 76 of PD 1529, a notice of lis pendens should
and require the applicant to present evidence. By the
contain a statement of the institution of an action or proceeding, the court where the
description in the notice "To All Whom It May Concern",
same is pending, and the date of its institution. A notice of lis pendens should also
all the world are made parties defendant and shall be
contain a reference to the number of the certificate of title of the land, an adequate
concluded by the default order.
description of the land affected and its registered owner.
Where an appearance has been entered
The Register of Deeds denied registration of the notice of lis
and an answer filed, a default order shall be entered
pendens because "the application was bereft of the original petition or complaint upon
against persons who did not appear and answer.
which this office will base its action." 28 In consulta to the LRA, petitioners pointed
out that they have complied with the requirements for the registration of the notice Petitioners' justification for filing a motion to annul the decrees and titles,
of lis pendens, as follows: as opposed to filing a motion to lift the order of general default, rests on two related
assumptions. First, with the filing of the 16 July 1997 motion and giving of due course
7.2.1 The Notice of Lis Pendens contains a
to the motion by the land registration court, petitioners assert that they acquired legal
statement of the filing by the Heirs of Eugenio Lopez of
standing in the registration proceedings. Second, buyer Eugenio Lopez, Sr. stepped
into the shoes of the sellers-applicants Sandoval and Ozaeta when applicants sold issued a certificate of finality on 8 March 1991. Petitioners filed their motion on 16
the property to him. As successors-in-interest of the buyer, petitioners contend that July 1997. Thus, even if petitioners filed a motion to lift the order of general default,
they are not strangers to the proceedings. the order of default could not be set aside because the motion was filed out of time.

To justify their two assumptions, petitioners traced the antecedent of In Lim Toco v. Go Fay, 37 this Court explained the effect of an order of
Section 22 of PD 1529 to Section 29 of Act 496 33 and its judicial interpretation default to the party defaulted. A party declared in default loses his standing in court.
inMendoza v. Court of Appeals. 34 As a result of his loss of standing, a party in default cannot appear in court, adduce
evidence, be heard, or be entitled to notice. A party in default cannot even appeal
Section 22 of PD 1529 provides: from the judgment rendered by the court, unless he files a motion to set aside the
order of default under the grounds provided in what is nowSection 3, Rule 9 of the
SECTION 22. Dealings with land pending 1997 Rules of Civil Procedure.
original registration. — After the filing of the application
and before the issuance of the decree of registration, the Indeed, in its comment before this Court, the LRA stated thus:
land therein described may still be the subject of dealings
in whole or in part, in which case the interested party Under Section 26, PD 1429, petitioners are
shall present to the court the pertinent instruments deemed to have been included by the default order.
together with the subdivision plan approved by the Those who did not file an answer should be considered
Director of Lands in case of transfer of portions thereof, as having lost their standing in court from that stage
and the court, after notice to the parties, shall order such (Republic v. Dela Rosa, 173 SCRA 12) except when they
land registered subject to the conveyance or file a motion to set aside the order [of] default on the
encumbrance created by said instruments, or order that grounds mentioned in Section 3, Rule 18 of the Rules of
the decree of registration be issued in the name of the Court (Toco v. Fay, 80 Phil. 166). EIaDHS
person to whom the property has been conveyed by said
instruments. ADCIca In land registration cases (as in the said
LRC No. N-18887), an order of general default was
The pertinent portion of Section 29 of Act 496 provides: deemed to have been issued based on the presumption
of regularity in judicial proceedings (Pascual, et
SECTION 29. After the filing of the al. v. Ortega, et al., 58 O.G. 12 March 1962 C.A.).
application and before the issuance of the decree of title Petitioners failed to adduce any evidence showing that
by the Chief of the General Land Registration Office, the the order of general default was lifted. Records disclosed
land therein described may be dealt with and instruments that without first filing a motion to lift the order of general
relating thereto shall be recorded in the office of the default, petitioners filed a motion to declare as null and
register of deeds at any time before issuance of the void the decrees and titles. Until the order of general
decree of title, in the same manner as if no application default is lifted by the court, petitioner could not be
had been made. The interested party may, however, considered as a party to the action. They are deemed
present such instruments to the Court of First Instance movants whose personality as far as the case is
instead of presenting them to the office of the Register of concerned is not yet admitted by the court considering
Deeds, together with a motion that the same be that the order of default has not been lifted. 38
considered in relation with the application, and the court,
after notice to the parties shall order such land registered One should be careful, however, to distinguish between movants as
subject to the encumbrance created by said instruments, mere interested parties prescribed under Section 22 of PD 1529 and movants as
or order the decree of registration issued in the name of intervenors-oppositors to the land registration proceedings. It is only in the latter case
the buyer or of the person to whom the property has been that a motion to lift the order of general default is required. It is only in the latter case
conveyed by said instruments. . . . that the doctrine pronounced in Serrano v. Palacio, 39 as repeatedly invoked by the
LRA and OSG, is applicable:
Mendoza v. Court of Appeals 35 explains the procedure in cases of
conveyance of the land subject of a registration proceeding by an instrument . . . [P]etitioners committed an error of
executed between the time of filing of the application for registration and the issuance procedure when they filed a motion to intervene in the . .
of the decree of title. . land registration case for the proper procedure would
have been for them to ask first for the lifting of the order
The law does not require that the of general default, and then, if lifted, to file an opposition
application for registration be amended by substituting to the application of the applicants. This is so because
the "buyer" or the "person to whom the property has been proceedings in land registration are in rem, and not in
conveyed" for the applicant. Neither does it require that personam, the sole object being the registration applied
the "buyer" or the "person to whom the property has been for, and not the determination of any right not connected
conveyed" be a party to the case. He may thus be a total with the registration (Estiva vs. Alvero, 37 Phil. 498).
stranger to the land registration proceedings. The only
requirements of the law are: (1) that the instrument be
presented to the court by the interested party together
with a motion that the same be considered in relation with Petitioners are not mere interested parties in this case. By filing their motion
the application; and (2) that prior notice be given to the to have the decrees and the corresponding certificates of title declared void,
parties to the case . . . . 36 they took the role of oppositors to the application for land registration.
The appellate court stated that "in as much as it would want to oblige to
Petitioners also assert that they do not dispute the judgment of the land
the plea of petitioners to hasten or expedite the proceedings and to avoid further
registration court. However, this position is in conflict with their 25 November 1998
expenses on the part of the petitioners, however[,] (it) could not." 40 Indeed, it
motion to have the decree and the titles declared void. Petitioners now assume the
requires a delicate balancing act between the objective of the Rules of Court to
roles of both successors-in-interest and oppositors. This confusion of roles brought
about petitioners' grave error in procedure. secure a just, speedy and inexpensive disposition of every action and
proceeding 41 and the strict requirements for a notice of lis pendens. The facts in this
The land registration court granted the application in LRC No. N-18887 case show that petitioners have not complied with the requirements.
on 31 May 1966 and issued a certificate of finality dated 8 March 1991. Petitioners
WHEREFORE, we DENY the petition. We AFFIRM the Decision of the
filed their motion to consider the deed of sale in the registration on 16 July 1997.
Court of Appeals in CA-G.R. SP No. 55993 dated 29 November 2000.
Petitioners filed their motion to have the decrees and the corresponding certificates
of title declared void on 25 November 1998. Petitioners filed both motions long after SO ORDERED.
the decision in LRC No. N-18887 became final and executory. Neither petitioners nor
even the applicants from whom they base their claim presented the Deed of Sale Quisumbing, Ynares-Santiago and Azcuna, JJ., concur.
before the land registration court while the action was pending.
Davide, Jr., C.J., is on leave.
Considering the facts and arguments as presented above, we hold that
the motion filed by petitioners is insufficient to give them standing in the land ||| (Heirs of Lopez, Sr. v. Enriquez, G.R. No. 146262, [January 21, 2005], 490 PHIL
registration proceedings for purposes of filing an application of a notice of lis 74-95)
pendens. However, we disagree with the LRA and the appellate court's observation
that petitioners need to file a motion to lift the order of general default. A motion to lift
the order of general default should be filed before entry of final judgment. The land
registration court granted the application for registration of title on 31 May 1966 and
FIRST DIVISION land in question subject of this petition as an owner . . ."
[G.R. No. 125154. September 28, 2001.] which motion was opposed by the petitioners herein.
DIGNA VERGEL, EDUARDO SALVACRUZ, BEATRIZ "On October 12, 1995, respondent filed with
MANACOP, FELICISIMA FLORES, GENEROSO the trial court a reply to the opposition interposed by the
SALVACRUZ, BLANDINO SALVACRUZ, MILAGROS petitioners and, at the same time, filed an "Urgent
SALVACRUZ and THE HEIRS OF CORAZON Motion" praying for the suspension of the proceedings.
SANTIAGO, petitioners, vs. COURT OF APPEALS "On October 18, 1995, the trial court issued
and DOROTEA-TAMISIN GONZALES,respondents. the first assailed order, the dispositive portion of which is
quoted, as follows:
Jose F. Manacop for petitioners. 'WHEREFORE, the motion to
Leopoldo P. dela Rosa and Tabalingcos & Associates Law Offices for set aside the Order of default as well as the
private respondents. motion to suspend the proceedings filed by
SYNOPSIS the movant through counsel is hereby
denied for lack of merit.'
Petitioners filed an application for land registration of a parcel of "On October 20, 1995, petitioners filed with
land in Los Baños. The trial court subsequently issued an order of general the trial court a "Motion to Strike Out Urgent Motion to
default against the whole world except the Republic of the Philippines. Suspend Proceeding."
Respondent filed a motion to set aside the order of general default, claiming "On November 21, 1995 respondent filed
that she failed to timely file her opposition because the application was filed with the trial court a "motion for reconsideration" of the
by petitioners in bad faith, surreptitiously and without notice to her. The trial order denying the motion to set aside the order of general
court denied the motion. On appeal, however, the Court of Appeals set aside default, which motion petitioners opposed.
the order of general default. "On November 28, 1995, the trial court
issued its second questioned order, the dispositive
The Supreme Court held that the Court of Appeals acted portion of which reads as follows:
arbitrarily when it set aside the trial court's order of general default without 'WHEREFORE, in view of the
factual basis. Since the Supreme Court is not a trier of facts, the case was foregoing, the motion for reconsideration,
remanded to the Court of Appeals for it to make finding of fact constituting dated November 16, 1995, is hereby denied
fraud, accident or excusable neglect sufficient for the court to lift the order of for lack of merit.'
general default in the land registration case involved. "On December 13, 1995, respondent filed
with the Court of Appeals 6 a petition
SYLLABUS for certiorari alleging that the trial court judge "acted
capriciously and without or in excess of his jurisdiction
1. REMEDIAL LAW; APPEAL; ARBITRARILY SETTING ASIDE and gravely abused the exercise of his discretion" in
DEFAULT ORDER WITHOUT BASIS IS ERRONEOUS; CASE AT BAR. — The issuing the two aforementioned orders." 7
Court of Appeals arbitrarily set aside the trial court's order of general default without On April 02, 1996, the Court of Appeals promulgated a decision annulling
factual basis save for its own gut feeling, ipse dixit. . . . In respondent's motion to set the trial court's orders dated October 18, 1995 and November 28, 1995, and
aside order of general default, she alleged that petitioners were aware of her claim of consequently, setting aside the trial court's order of general default dated December
ownership over the subject property, but did not give her personal notice of the filing 15, 1994, in Land Registration Case No. 88-94-C with respect to respondent. 8
of the application. She learned about the application by accident. In the petition
for certiorari she filed with the Court of Appeals, respondent alleged that petitioners Hence, this appeal. 9
filed the application in bad faith, surreptitiously and without notice to her. The Court
of Appeals did not make a finding on this. Hence, we find that the appellate court The Issue
erred in setting aside the order of general default in the Land Registration Case No. The issue presented is whether the Court of Appeals erred in setting
88-94-C, without making a specific finding of fraud, accident or excusable neglect that aside the trial court's order of general default in the land registration case involved
prevented respondent from timely opposing the application. without making a specific finding of fraud, negligence, accident or excusable mistake
but relying on its view that substantial justice and speedy determination of the
2. ID.; ID.; SUPREME COURT IS NOT A TRIER OF FACTS; REMAND controversy would be better attained in lifting the order of general default, to enable
OF CASE TO THE COURT OF APPEALS, PROPER. — We are not a trier of facts. a claimant to oppose and to establish a case of ownership in herself.
Consequently, we have to remand the case to the Court of Appeals for it to make
findings of fact constituting fraud, accident or excusable neglect sufficient for the court The Court's Ruling
to lift the order of general default in the land registration case involved.
We grant the petition. The Court of Appeals arbitrarily set aside the trial
court's order of general default without factual basis save for its own gut feeling,ipse
DECISION
dixit. 10 Respondent's failure to file timely opposition to the application for land
PARDO, J p:
registration because she missed reading the publication of the notice in the Official
The Case Gazette 11 or in the newspaper "Malaya" issue of August 8, 1994, 12 in itself may
not be considered excusable negligence.
The case is an appeal via certiorari from the decision of the Court of
Appeals 1 setting aside the orders 2 of the trial court that denied petitioners' motion In respondent's motion to set aside order of general default, she alleged
to set aside the order of general default 3 in an application for registration of a parcel that petitioners were aware of her claim of ownership over the subject property, but
of land, consisting of one thousand, one hundred seventy six (1,176), situated in did not give her personal notice of the filing of the application. She learned about the
barrio Batong Malake, municipality of Los Baños, province of Laguna. 4 EaIcAS application by accident. In the petition for certiorari she filed with the Court of
Appeals, respondent alleged that petitioners filed the application in bad faith,
The Facts
surreptitiously and without notice to her. 13 The Court of Appeals did not make a
The facts, as found by the Court of Appeals, 5 are as follows:
finding on this.
"On May 26, 1994, Digna Vergel, Eduardo
Salvacruz, Beatriz Mañacop, Felicisima Flores, Hence, we find that the appellate court erred in setting aside the order of
Generoso and Blandino Salvacruz, Milagros Evangelista general default in the Land Registration Case No. 88-94-C, without making a specific
and the heirs of Corazon Santiago, namely: Leocadio, Jr. finding of fraud, accident or excusable neglect that prevented respondent from timely
and Concepcion Santiago (petitioners herein) filed with opposing the application.
the Regional Trial Court, Calamba, Laguna an
application for registration of a parcel of land (for titling We are not a trier of facts. Consequently, we have to remand the case
purposes). to the Court of Appeals for it to make findings of fact constituting fraud, accident or
"On July 20, 1994, the Republic of the excusable neglect sufficient for the court to lift the order of general default in the land
Philippines represented by the Director of Lands filed an registration case involved.
opposition to the application for registration.
"On December 15, 1994, the trial court The Fallo
issued "an order of general default against the whole WHEREFORE, the Court SET ASIDE the decision of the Court of
world with the exception of Republic of the Philippines . . Appeals in CA-G.R. SP No. 39239.
. ." Let the case be remanded to the Court of Appeals for further proceedings
"On October 3, 1995, respondent Dorotea with instructions to determine whether there exists facts warranting the lifting of the
Tamisin Gonzales filed with the trial court an "Urgent order of general default in LRC Case No. 88-94-C of the trial court.
Motion to Set Aside the Order of General Default" No costs.
alleging, inter alia, in her affidavit that she "is claiming the SO ORDERED.
||| (Vergel v. Court of Appeals, G.R. No. 125154, [September 28, 2001], 418 PHIL
474-479)

You might also like